Sie sind auf Seite 1von 186

1.

Historian: Newton developed mathematical concepts and techniques that are fundamental to modern

calculus. Leibniz developed closely analogous concepts and techniques. It has traditionally been thought that these discoveries were independent. Researchers have, however, recently discovered notes of Leibniz that discuss one of Newton s boo!s on mathematics. "everal scholars have argued that since the boo! includes a presentation of Newton s calculus concepts and techniques, and since the notes were written before Leibniz own development of calculus
concepts and techniques, it is virtually certain that the traditional view is false. # more cautious conclusion than this is called for, however. Leibniz notes are limited to early sections of Newtons book, sections that precede the ones in which Newtons calculus concepts and techniques are presented .

In the historian s reasoning, the two boldfaced portions play which of the following roles$ #. %he first provides evidence in support of the overall position that the historian defends& the second is evidence that has been used to support an opposing position. '. %he first provides evidence in support of the overall position that the historian defends& the second is that position. (. %he first provides evidence in support of an intermediate conclusion that is drawn to provide support for the overall position that the historian defends& the second provides evidence against that intermediate conclusion. ). %he first is evidence that has been used to support a conclusion that the historian criticizes& the second is evidence offered in support of the historian s own position.
*. %he first is evidence that has been used to support a conclusion that the historian criticizes& the second is further information that substantiates that evidence.

#nswer+ ) ,not confirmed.. /hich of the following best completes the passage below$ # primary factor in perpetuating the low salaries of women wor!ers has been their segregation in the so0 called pin!0collar occupations, such as nursing, teaching, library science, and secretarial wor!. 1artly because these 2obs have traditionally been held by women, their salary levels have been depressed, and, despite increased attempts to unionize these wor!ers in recent years, their pay continues to lag. 3oreover, although a large percentage of women than ever before are now entering and remaining in the 2ob mar!et, most continue to gravitate toward the pin!0collar fields, despite the lower salaries. It seems clear, therefore, that if the average salaries of women wor!ers are to approach those of men, 444444

,#- labor unions must redouble their efforts to improve the lot of wor!ing women
,'- society s perception of pin!0collar 2obs as less important and less demanding than other 2obs must be changed ,(- more men must be encouraged to enter fields traditionally occupied by women ,)- the number of 2obs in the pin!0collar fields relative to the size of the wor! force as a whole must be mar!edly increased ,*- more women must enter occupations other than those traditionally reserved for them #nswer+ * 5. *very political philosopher of the early twentieth century who was either a socialist or a communist was influenced by Rosa Lu6emburg. No one who was influenced by Rosa Lu6emburg advocated a totalitarian state. If the statements above are true, which one of the following must on the basis of them also is true$

,#- No early0twentieth0century socialist political philosopher advocated a totalitarian state. ,'- *very early0twentieth0century political philosopher who did not advocate a totalitarian state was influenced by Rosa Lu6emburg. ,(- Rosa Lu6emburg was the only person to influence every early0twentieth0century political philosopher who was either socialist or communist. ,)- *very early0twentieth0century political philosopher who was influenced by Rosa Lu6emburg and was not a socialist was a communist. ,*- *very early0twentieth0century political philosopher who did not advocate a totalitarian state was either socialist or communist. #nswer+ # 7. "ome environmentalists question the prudence of e6ploiting features of the environment, arguing that there are no economic benefits to be gained from forests, mountains, or wetlands that no longer e6ist. 3any environmentalists claim that because nature has intrinsic value it would be wrong to destroy such features of the environment, even if the economic costs of doing so were outweighed by the economic costs of not doing so. /hich one of the following can be logically inferred from the passage$ ,#- It is economically imprudent to e6ploit features of the environment. ,'- "ome environmentalists appeal to a noneconomic 2ustification in questioning the defensibility of e6ploiting features of the environment. ,(- 3ost environmentalists appeal to economic reasons in questioning the defensibility of e6ploiting features of the environment. ,)- 3any environmentalists provide only a noneconomic 2ustification in questioning the defensibility of e6ploiting features of the environment. ,*- *ven if there is no economic reason for protecting the environment, there is a sound noneconomic 2ustification for doing so. #nswer+ ' E planation+ the 2ustification advanced by 8many environmentalists8 in the last sentence is clearly non0 economic, since it runs directly counter to economic principles ,quote+ ... even if the economic costs of doing so were outweighed by the economic costs of not doing so-. i9ll e6plain why ( and ) are wrong. ,c- /hile there is an economic 2ustification in the passage 0 specifically, the 2ustification mentioned in the first sentence 0 this 2ustification is limited to ":3* environmentalists. this choice is wrong, then, since it claims 8most8 and is thus overreaching. ,d- %his choice is wrong because of the 8only8. %he passage claims that certain environmentalists have advanced a non0economic 2ustification, but never says that this is the :NL; 2ustification advanced by those environmentalists. In fact, note that, R*<#R)L*"" := %>* ?@*"%I:N, this (#NN:% be the correct answer I= ,b- is also an available option. ,reason+ if ,d- is true, then ,b- 3@"% also be true, since it9s a wea!er claim about the same thing. therefore, if ,d- is true, then ,b- is a fortiori also true. since you can9t have two correct answers, ,dcan9t possibly be correct.-

A.

Economist+ In the interaction between producers and consumers, the only obligation that all parties have to act in the best interests of their own side. #nd distribution of information about product defects is in the best interests of the consumer, "o consumers are always obligated to report product defects they discover, while producers are never obligated to reveal them.

/hich one of the following is an assumption required by the economist9s argument$ #. It is never in the best interests of producers for a producer to reveal a product defect. '. No one e6pects producers to act in a manner counter to their own best interests. (. #ny product defect is li!ely to be discovered by consumer ). # product defect is more li!ely to be discovered by a consumer than by a producer *. %he best interests of consumers never coincide with the best interests of producers

#nswer+ # E planation+ #rgument+In the interaction between producers and consumers, the only obligation that all parties have to act in the best interests of their own side. (onclusion + "o consumers are always obligated to report product defects they discover, while producers are never obli!ated to reveal them" #ssumption + It is never in the best interests of producers for a producer to reveal a product defect.

B.

%he wor!ers at 'ell 3anufacturing will shortly go on stri!e unless the management increases their wages. #s 'ell s president is well aware, however, in order to increase the wor!er s wages, 'ell would have to sell off some of its subsidiaries. "o, some of 'ell s subsidiaries will be sold. %he conclusion above is properly drawn if which one of the following is assumed$ ,#- 'ell 3anufacturing will begin to suffer increased losses. ,'- 'ell s management will refuse to increase its wor!er s wages. ,(- %he wor!ers at 'ell 3anufacturing will not be going on stri!e. ,)- 'ell s president has the authority to offer the wor!ers their desired wage increase. ,*- 'ell s wor!ers will not accept a pac!age of improved benefits in place of their desired wage increase.

#nswer+ (

C.

#dvertisement+ #nyone who e6ercises !nows from firsthand e6perience that e6ercise leads to better performance of such physical organs as the heart and lungs, as well as to improvement in muscle tone. #nd since your brain is a physical organ, your actions can improve its performance, too. #ct now. "ubscribe to "timulus+ read the magazine that e6ercises your brain. %he #dvertisement employs which one of the following argumentative strategies$ ,#- It cites e6perimental evidence that subscribing to the product being advertised has desirable consequences. ,'- It ridicules people who do not subscribe to "timulus by suggesting that they do not believe that e6ercise will improve brain capacity. ,(- It e6plains the process by which the product being advertised brings about the result claimed for its use. ,)- It supports its recommendation by a careful analysis of the concept of e6ercise. ,*- It implies that brains and muscle are similar in one respect because they are similar in another respect.

#nswer+ * E planation+ since the ad states that wor!ing out your brain will improve your brain much in the same way that wor!ing out your physical organs improve their performance

D.

/hen #licia <reen borrowed a neighbor s car without permission, the police merely gave her a warning. >owever, when 1eter =oster did the same thing, he was charged with automobile theft. 1eter came to the attention of the police because the car he was driving was hit by a speeding ta6i. #licia was stopped because the car she was driving had defective taillights. It is true that the car 1eter too! got damaged and the car #licia too! did not, but since it was the ta6i that caused the damage this difference was not due to any difference in the blameworthiness of their behavior. %herefore, #licia should also have been charged with automobile theft. If all of the claims offered in support of the conclusion are accurate, each of the following could be true *E(*1%+ ,#- %he interests of 2ustice would have been better served if the police had released 1eter =oster with a warning. ,'- #licia <reen had never before driven a car belonging to someone else without first securing the owner s permission. ,(- 1eter =oster was hit by the ta6i while he was running a red light, whereas #licia <reen drove with e6tra care to avoid drawing the attention of the police to the car she had ta!en. ,)- #licia <reen barely missed hitting a pedestrian when she sped through a red light ten minutes before she was stopped by the police for driving a car that had defective taillights. ,*- 1eter =oster had been cited for speeding twice in the preceding month, whereas #licia <reen had never been cited for a traffic violation.

#nswer+ ( E planation+ %he part of the argument that9s important is+ but since it was the ta6i that caused the damage this difference was not due to any difference in the blameworthiness of their behavior.. %his part of the argument states that neither driver was at fault or to blame. >owever, choice ( states that 1eter ran a red light, which would imply that he was at least partially to blame. F. @nder the influence of today s computer0oriented culture, publishing for children has ta!en on a flashy new loo! that emphasizes illustrations and graphic design& the resulting lac! of substance leads to boo!s that are short0lived items covering mainly trendy sub2ects. %he changes also include more humorous content, simplification of difficult material, and a narrower focus on specific topics. /hich one of the following is most strongly supported by the information above$

#. %he inclusion of humorous material and a narrower focus detract from the substance of a children s
boo!.

'. %he substance of a children s boo! is important to its longevity. (. (hildren of the computer generation cannot concentrate on long, unbro!en sections of prose. ). (hildren 2udge boo!s primarily on the basis of graphic design. *. %he lac! of substance of a children s boo! is unli!ely to be important to its popularity.

#nswer+ ' E planation+ %he prompt says that 8the resulting lac! of substance leads to boo!s that are short0lived.8 If lac! of substance leads to lac! of longevity, then we can conclude that substance is necessary for longevity.

1G. =urther evidence of a connection between brain physiology and psychological states has recently been uncovered in the form of a correlation between electroencephalograph patterns and characteristic moods. # study showed that participants who suffered from clinical depression e6hibited less left frontal lobe activity than right, while, conversely, characteristically good0natured participants e6hibited greater left lobe activity. %hus one s general disposition is a result of the activity of one s frontal lobe. *ach of the following, if true, wea!ens the argument *E(*1%+

A. 3any drugs prescribed to combat clinical depression act by causing increased left lobe activity. B. *6cessive sleep, a typical consequence of clinical depression, is !nown to suppress left lobe activity. C. =rontal lobe activity is not sub2ect to variation the way general disposition is. D. *arlier studies indicated that frontal lobe activity and emotive states are both caused by activity in the
brain s limbic activity.

E. "ocial interaction of the !ind not engaged in by most clinically depressed people is !nown to stimulate
left lobe activity.

#nswer+ # E planation: %he author is assuming because low left lobe levels and depression are correlated, the former causes the latter. /e can wea!en this causal relationship by establishing that 1- )epression causes a loss of left lobe activity ,b, e-, .- "ome other root cause is responsible for both effects ,d-, or 5- the findings are simply coincidence and the relationship isn9t reliable ,c-. #, meanwhile, indicates that depression can be fought by raising left0lobe levels, strengthening the conclusion that lobe levels control disposition.

11. /e ought to pay attention only to the intrinsic properties of a wor! of art. Its other, e6trinsic properties are irrelevant to our aesthetic interactions with it. =or e6ample, when we loo! at a painting we should consider only what is directly presented in our e6perience of it. /hat is really aesthetically relevant, therefore, is not what a painting symbolizes, but what it directly presents to e6perience.

%he conclusion follows logically if which one of the following is added to the premises$ #. '. /hat an art wor! symbolizes involves only e6trinsic properties of that wor!. %here are certain properties of our e6periences of artwor!s that can be distinguished as symbolic properties. :nly an artwor! s intrinsic properties are relevant to our aesthetic interactions with it. It is possible in theory for an artwor! to symbolize nothing. #n intrinsic property of an artwor! is one that related the wor! to itself.

(. ). *.

#nswer+ # E planation: %he authors conclusion is that what is important to a painting is what it presents to e6perience rather than what it symbolizes ,we !now this by the !eyword therefore-. >is evidence is that e6trinsic

properties are unimportant, and intrinsic properties are the !ey. 'ecause he9s shifting from irrelevant e6trinsic properties in his evidence to irrelevant symbolism in his conclusion, it 3@"% be true the symbolism is e6trinsic& if any symbolism is intrinsic, then his logic is flawed.

1.. %he 'oard of %rustees of the =ederici #rt 3useum has decided to sell some wor!s from its collection in order to raise the funds necessary to refurbish its galleries. #lthough this may seem li!e a drastic remedy, the curator had long maintained that among the paintings that the late 3s. =ederici collected for the museum were several unsuccessful immature wor!s by Renoir and (ezanne that should be sold because they are of inferior quality and so add nothing to the overall quality of the museum s collection. >ence, the board s action will not detract from the museum s collection. /hich one of the following, if true, most wea!en the argument$ ,#- %he directors of an art museum can generally raise funds for refurbishing the building in which the museum s collection is housed by means other than selling parts of its collection. ,'- %he quality of an art collection is determined not 2ust by the quality of its paintings, but by what development of the artistic talent and ideas of the artists represented. ,(- %he immature wo!s by Renoir and (ezanne that were purchased by 3s. =ederici were at that time thought by some critics to be unimportant 2uvenile wor!s. ,)- %hose people who speculate in art by purchasing artwor!s merely to sell them at much higher prices welcome inflation in the art mar!et, but curators of art museum regret the inflation in the art mar!et. ,*- %he best wor! of a great artist demands much higher prices in the art mar!et than the worst wor! of that same artist.

#nswer+ '

15. In a certain municipality, a 2udge overturned a suspect s conviction for possession of an illegal weapon. %he suspect had fled upon seeing police and subsequently discarded the illegal weapon after the police gave chase. %he 2udge reasoned as follows+ the only cause for the police giving chase was the suspect s flight& by itself, flight from the police does not create a reasonable suspicion of a criminal act& evidence collected during an illegal chase is inadmissible& therefore, the evidence in this case was inadmissible.

/hich one of the following principles, if valid, most helps to 2ustify the 2udge s decision that the evidence was inadmissible$ ,#- =light from the police could create a reasonable suspicion of a criminal act as long as other significant factors are involved. ,'- 1eople can legally flee from the police only when those people are not involved in a criminal act at the time. ,(- 1olice can legally give chase to a person only when the person s actions have created a reasonable suspicion of a criminal act. ,)- =light from the police should not itself be considered a criminal act. ,*- In all cases in which a person s actions have created a reasonable suspicion of a criminal act, police can legally give chase to that person

#nswer+ (

E planation: 'ecause the 2udge concludes the evidence is inadmissible due to the illegal chase. #nd the chase is illegal ,according to the 2udge- because there were no grounds for reasonable suspicion. #gain, by designating it an illegal chase, the 2udge must believe that police can legally give chase only when reasonable suspicion has been established. In logical terms, this is the contra0positive to the 2udge9s initial statement.

17. #nthropolo!ist+ Hiolence is an e6treme form of aggression, and is distinct from the self0e6pression sufficient for survival under normal conditions. >uman beings in certain situations react to unpleasant stimuli with violence but only because they are conditioned by their culture to react in this manner.

*ach of the following can be logically inferred from the anthropologist s statements *E(*1%+ ,#- Not all aggression is violent. ,'- %he self0e6pression required for survival is generally nonaggressive. ,(- "ome behaviors are influenced by the cultures in which human beings live. ,)- In normal circumstance, human beings can survive by responding nonviolently. ,*- Hiolent behavior is a product of one s cultural environment.

#nswer+ '

1A. $ommissioner: I have been incorrectly criticized for having made my decision on the power plant issue prematurely. I based my decision on the report prepared by the neighborhood association and although I have not studied it thoroughly, I am sure that the information it contains is accurate. 3oreover, you may recall that when I received input from the neighborhood association on 2ail relocation, I agreed with its recommendation.

%he commissioner s argument is L*#"% vulnerable to which one of the following criticism$ ,#- It ta!es for granted that the association s information is not distorted by bias. ,'- It draws a conclusion about the recommendations of the association from incomplete recollections. ,(- It ta!es for granted that the association s report is the only direct evidence that needed to be considered. ,)- It hastily concludes that the association s report is accurate without having studied it in detail. ,*- It ta!es for granted that agreeing with the associations past recommendation helps 2ustify agreeing with its current recommendation

#nswer+ ' E planation: the commissioner9s conclusion on the recommendation is based on the trust he has on its accuracy IJ the commissioner cannot be faulted for incomplete recollections ,spotty memory#ll other options are valid criticisms that can be made on the commissioner s argument.

1B. # reason Larson cannot do the assignment is that she has an unavoidable scheduling conflict. :n the other hand, a reason =ran!s cannot do the assignment is that he does not quite have the assertiveness the tas! requires. "o, the tas! must be assigned to 1ar!er, the only supervisor in the shipping department other than Larson and =ran!s.

%he argument depends on assuming which one of the following$ ,#- Larson has the assertiveness the tas! requires. ,'- %he tas! cannot be assigned to anyone other than a supervisor in the shipping department. ,(- =ran!s would be assigned the tas! if =ran!s had the assertiveness the tas! requires. ,)- %he tas! cannot be assigned to anyone who has any !ind of scheduling conflict. ,*- No one who is not a supervisor in the shipping department has the assertiveness this tas! requires.

#nswer+ )

1C. %hilosopher+ %he rational pursuit of happiness is quite different from always doing what one most strongly desires to do. %his is because the rational pursuit of happiness must include consideration of long0term consequences, whereas our desires are usually focused on the short term. 3oreover, desires are sometimes compulsions, and while ordinary desires result in at least momentary happiness when their goals are attained, compulsions strongly drive a person to pursue goals that offer no happiness even when reached.

If all of the philosopher9s statements are true, each of the following could be true *E(*1%+ ,#- the ma2ority of people do not have compulsions. ,'- #ttaining the goal of any desire results in momentary happiness. ,(- 3ost people do not pursue happiness rationally. ,)- 3ost people want more than their own personal happiness. ,*- #ll actions have long0term consequences.

#nswer+ ' E planation+ ' says K#ttaining the goal of any desire results in momentary happinessL while the premise says 9desires are sometimes compulsions9 "o ' can be put as K#ttaining the goal of any compulsion results in momentary happinessL %his negates another premise that says compulsions strongly drive a person to pursue goals that offer no happiness even when reached

1D. #nger in response to insults is unreasonable, for insults are merely assertions that someone has undesirable characteristics. If such an assertion is false, the insulted party ought to pity the ignorance prompting the insult. If it is true, the insulted party should be grateful for such useful information.

/hich one of the following, if assumed, enables the argument s conclusion to be properly drawn$

A. #ctions prompted by ignorance do not warrant hostile reactions. B. #nger is an unreasonable response to useful information. C. #nger is an unreasonable response to any action that should prompt pity or gratitude.

D. <ratitude and pity are reasonable responses to some forms of hostile or insensitive behavior. E. 1ity is the only reasonable reaction to people with undesirable characteristics.

#nswer+ ( E planation+ insult 000000000J #nger insult I assertion of bad character if assertion I false IIJ pityMignorance if assertion I true IIJ gratefulMuseful info so either one should be pitiful or grateful for insult... >ence (

1F. %he city government should invest surplus funds in improving the city9s transportation networ!. &ost of the network was put in place at a time when the city was much smaller in both area and population" %he subway system is outdated and understaffed. %he buses rarely run on schedule and their routes are inconvenient. 'f the city does not make chan!es soon to the network, it will see many of its prized industries relocate to more convenient cities and, as a result, the city(s financial health will be )eopardized" In the argument above, the two portions in boldface play which of the following roles$ #. %he first is an e6planation of a current state of affairs& the second is a prediction based on that state of affairs. '. %he first is a statement of fact in opposition to the author9s conclusion& the second is that conclusion. (. %he first emphasizes an e6isting problem& the second offers a proposal to solve that problem. ). %he first is information the author suggests has been overloo!ed in the situation at hand& the second describes that situation. *. %he first is a 2ustification of an impending problem& the second describes the consequences of that problem.

#nswer+ #

.G. *eacher+ Nournalists who conceal the identity of the sources they quote sta!e their professional reputations on what may be called the logic of anecdotes. %his is so because the statements reported by such 2ournalists are dissociated from the precise circumstances in which they were made and thus will be accepted for publication only if the statements are high in plausibility or originality or interest to a given audience0precisely the properties of a good anecdote. +tudent+ 'ut what you are saying, then, is that the 2ournalist need not bother with sources in the first place. "urely, any reasonably resourceful 2ournalist can invent plausible, original, or interesting stories faster than they can be obtained from unidentified sources. I. %he student9s response contains which one of the following reasoning flaws$ ,#- confusing a marginal 2ournalistic practice with the primary wor! done by 2ournalists ,'- ignoring the possibility that the teacher regards as a prerequisite for the publication of an unattributed statement that the statement have actually been made ,(- confusing the characteristics of reported statements with the characteristics of the situations in which the

statements were made ,)- 2udging the merits of the teacher9s position solely by the most e6treme case to which the position applies ,*- falsely concluding that if three criteria, met 2ointly, assure an outcome, then each criterion, met individually, also assures that outcome

#nswer+ ' E planation+ the student assumes that the 2ournalist can reproduce a anecdote without having to gather statements from a source and ma!e it believable. >owever the student doesn9t ta!e into account that the reported statements ,which result in an anecdote- could be a product of the circumstances in which they are given ,via a source-. "o without a source, you don9t get the reported statements thus you cannot produce the anecdote. II. /hich one of the following, if true, most strengthens the teacher9s argument$ ,#- # 2ournalist undermines his or her own professional standing by submitting for publication statements that, not being attributed to a named source, are re2ected for being implausible, unoriginal, or dull. ,'- "tatements that are attributed to a fully identified source ma!e up the ma2ority of reported statements included by 2ournalists in stories submitted for publication. ,(- Reported statements that are highly original will often seem implausible unless submitted by a 2ournalist who is !nown for solid, reliable wor!. ,)- Reputable 2ournalists sometimes do not conceal the identity of their sources from their publishers but insist that the identity of those sources be concealed from the public. ,*- Nournalists who have special access to sources whose identity they must conceal are greatly valued by their publishers.

#nswer+ # E planation+ %eacher9s argument is what reporter do such as conceal the identity is not important. 'ecause statements reported by such 2ournalists are dissociated from the precise circumstances and will be publish only if the statements are high in plausibility or originality or interest to a given audience. # strengthen the later part of argument.

.1. =ares on the city0run public buses in <reenville are subsidized by city ta6 revenues, but among the beneficiaries of the low fares are many people who commute from outside the city to 2obs in <reenville. "ome city councilors argue that city ta6es should be used primarily to benefit the people who pay them, and therefore that bus fares should be raised enough to cover the cost of the service.

*ach of the following, if true, would wea!en the argument advanced by the city councilors *E(*1%+ #. 3any businesses whose presence in the city is beneficial to the city s ta6payers would relocate outside the city if public0transit fares were more e6pensive. 'y providing commuters with economic incentives to drive to wor!, higher transit fares would worsen air pollution in <reenville and increase the cost of maintaining the city s streets.

'.

(.

Increasing transit fares would disadvantage those residents of the city whose law incomes ma!e them e6empt from city ta6es, and all city councilors agree that these residents should be able to ta!e advantage of city0run services. Hoters in the city, many of whom benefit from the low transit fares, are strongly opposed to increasing local ta6es. 1eople how wor! in <reenville and earn wages above the nationally mandated minimum all pay the city wage ta6 of A percent.

).

*.

#nswer+ ) E planation+ %he issue here isn9t an increase in local ta6es. %he councilors simply want to discontinue using city ta6es to subsidize the bus system, and ma!e up for the lost money by raising the bus fare. %his, by itself, doesn9t imply that city ta6es will go up, so the fact that city voters oppose a potential ta6 hi!e ,big surprise there- has no bearing on the argument.

... ,overnment -fficial+ (learly, censorship e6ists if we, as citizens, are not allowed to communicate what we are ready to communicate at our own e6pense or if other citizens are not permitted access to our communications at their own e6pense. 1ublic unwillingness to provide funds for certain !inds of scientific, scholarly, or artistic activities cannot, therefore, be described as censorship.

%he flawed reasoning in the government official s argument is most parallel to that in which one of the following$

#. #ll actions that cause unnecessary harm to others are un2ust& so if a 2ust action causes harm to others,
that action must be necessary.

'. "ince there is more to good manners than simply using polite forms of address, it is not possible to say
on first meeting a person whether or not that person has good manners.

(. #crophobia, usually defined as a morbid fear of heights, can also mean a morbid fear of sharp ob2ects.
"ince both fears have the same name, they undoubtedly have the same origin.

). %here is no doubt that a deed is heroic if the doer ris!s his or her own life to benefit another person.
%hus an action is not heroic if the only thing it endangers is the reputation of the doer.

*. 1erception of beauty in an ob2ect is determined by past and present influences on the mind of the
beholder. %hus no ob2ect can be called beautiful, since not everyone will see beauty in it,

#nswer+ ) E planation+ Oey to solve questions of these forms is to separate the premise and the conclusion and loo! at the way how the author arrives to the conclusion.

.5. %he Napanese hai!u is defined as a poem of three lines with five syllables in the first line, seven syllables in the second line, and five syllables in the third line. *nglish poets tend to ignore this fact. )isregarding syllable count, they generally call any three0line *nglish poem with a Khai!u feelL a hai!u. %his demonstrates that *nglish poets have little respect for foreign traditions, even those from which some of their own poetry derives.

%he reasoning is flawed because it

A. (onfuses matters of ob2ective fact with matters of sub2ective feeling B. )raws a conclusion that is broader in scope than is warranted by the evidence advanced C. Relies on stereotypes instead of presenting evidence D. :verloo!s the possibility that the case it cites is not unique E. =ails to ac!nowledge that ignoring something implies a negative 2udgment about that thing

#nswer+ ' E planation+ premise+ how *nglish poets classify poems as hai!u ,Napanese tradition- poems. (onclusion+ *nglish poets disregard foreign tradition. IJthe conclusion ma!es a sweeping generalization. ' fits in fine

.7. # physician who is too thorough in conducting a medical chec!up is li!ely to sub2ect the patient to the discomfort and e6pense of unnecessary tests. :ne who is not thorough enough is li!ely to miss some serious problem and therefore give the patient a false sense of security. It is difficult for physicians to 2udge e6actly how thorough they should be. %herefore, it is generally unwise for patients to have medical chec!ups when they do not feel ill.

/hich one of the following, if true, would most seriously wea!en the argument in the passage$

A. "ome serious diseases in their early stages have symptoms that physicians can readily detect, although
patients are not aware of any problem.

B. @nder the pressure of reduced reimbursements, physicians have been reducing the average amount of
time they spend on each medical chec!up.

C. 1atients not medically trained are unable to 2udge for themselves what degree of thoroughness is
appropriate for physicians in conducting medical chec!ups.

D. 3any people are financially unable to afford regular medical chec!ups. E. "ome physicians sometimes e6ercise e6actly the right degree of thoroughness in performing a medical
chec!up.

#nswer+ # E planation+ If the patients are not aware of any problem and don9t feel ill, physicians can definitely discover the symptoms of the serious disease at its early stage. %his wea!ens the conclusion

.A. %here is relatively little room for growth in the overall carpet mar!et, which is tied to the size of the population. 3ost who purchase carpet do so only once or twice, first in their twenties or thirties, and then perhaps again in their fifties or si6ties. %hus as the population ages, companies producing carpet will be able to gain mar!et share in the carpet mar!et only through purchasing competitors, and not through more aggressive mar!eting.

/hich one of the following, if true, casts the most doubt on the conclusion above$

A. 3ost of the ma2or carpet producers mar!et other floor coverings as well. B. 3ost established carpet producers mar!et several different brand names and varieties, and there is no
remaining niche in the mar!et for new brands to fill.

C. %wo of the three mergers in the industry s last ten years led to a decline in profits and revenues for the
newly merged companies.

D. 1rice reductions, achieved by cost0cutting in production, by some of the dominant firms in the carpet
mar!et are causing other producers to leave the mar!et altogether.

E. %he carpet mar!et is unli!e most mar!ets in that consumers are becoming increasingly resistant to new
patterns and styles.

#nswer+ ) E planation+ ) !ives an alternate way of increasin! market share

.B. "ome flowering plant species, entirely dependent on bees for pollination, lure their pollinators with abundant nectar and pollen, which are the only source of food for bees. :ften the pollinating species is so highly adapted that it can feed from P and thus pollinate P only a single species of plant. "imilarly, some plant species have evolved flowers that only a single species of bee can pollinate P an arrangement that places the plant species at great ris! of e6tinction. If careless applications of pesticides destroy the pollinating bee species, the plant species itself can no longer reproduce.

%he information above, if true, most strongly supports which one of the following$

A. %he earliest species of flowering plants appeared on *arth contemporaneously with the earliest bee
species.

B. If the sole pollinator of a certain plant species is in no danger of e6tinction, the plant species it pollinates
is also unli!ely to become e6tinct.

C. "ome bees are able to gather pollen and nectar from any species of plant. D. %he blossoms of most species of flowering plants attract some species of bees and do not attract
others.

E. %he total destruction of the habitat of some plant species could cause some bee species to become
e6tinct.

#nswer+ * E planation+ * states that the total destruction of some plant species could cause some bee species to become e6tinct. %his is the right answer as in the argument some pollinating species can feed only from a sole class of plants. %hus the e6tinction of such plants will probably cause the e6tinction of pollinating species. ' confuses necessity and sufficiency. /hile the 8sole pollinator8 species may be necessary to the plant9s survival, it9s not necessarily sufficient to guarantee its survival. %he bees could be alive and well but some other factor, li!e a drought, for e6ample, could still !ill the flowers

.C. In the (enterville 'otanical <ardens, all tulip trees are older than any maples. # ma2ority, but not all, of the garden s sycamores are older than any of its maples. #ll the garden s maples are older than any of its dogwoods.

If the statements above are true, which one of the following must also be true of trees in the (enterville 'otanical <ardens$

A. "ome dogwoods are as old as the youngest tulip trees. B. "ome dogwoods are as old as the youngest sycamores. C. "ome sycamores are not as older as the oldest dogwoods. D. "ome tulip trees are not as old as the oldest sycamores. E. "ome sycamores are not as old as the youngest tulip trees.

#nswer+ * E planation+ /e !now+ tulips J maples some sycamores J maples maples J some sycamores all maples J dogwoods

so we !now tulips M QsomeR sycamores J maples J dogwoods M QsomeR sycamores Important+ /e have no info on the relation of tulips to sycamores or dogwoods to sycamores. #- %his is impossible. If all tulips are older than maples and all dogwoods are younger than maples, no dogwoods could be as old as tulip trees. *liminate. '- %his is possible but we are loo!ing for a must be true. %he oldest dogwood could be 1 year old and the youngest sycamore B years old, and we could still fulfill the terms of the question. *liminate. (- #gain, this is possible, but it doesn9t need to be true to fulfill the terms of the problem. *liminate. )- :nce again, this is possible. "ome tulip trees could be younger than the oldest sycamore, we have no info as to the relation between tulips and sycamores. 'ut it is equally possible that all tulip trees are older than the oldest sycamore. *liminate. *- %his is the only one that must be true. "ince the youngest tulip trees are still older than maple trees, and there are some sycamores that are younger than maple trees, this condition must be true according to the passage.

.D. "ince 3ayor )rabble always repays her political debts as soon as possible, she will almost certainly appoint Lee to be the new head of the arts commission. Lee has wanted that 2ob for a long time, and )rabble owes Lee a lot for his support in the last election.

/hich one of the following is an assumption on which the argument depends$

A. 3ayor )rabble has no political debt that is both of longer standing than the one she owes to Lee and
could as suitably be repaid by an appointment to be the new head of the arts commission.

B. %here is no one to whom 3ayor )rabble owes a greater political debt for support in the last election
than the political debt she owes to Lee.

C. Lee is the only person to whom 3ayor )rabble owes a political debt who would be willing to accept an
appointment from her as the new head of the arts commission.

D. /hether Lee is qualified to head the arts commission is irrelevant to 3ayor )rabble s decision E. %he only way that 3ayor )rabble can adequately repay her political debt to Lee is by appointing him to
head the arts commission.

#nswer+ #

.F. /hen glass products are made from recycled glass, the resulting products can be equal in quality to glass products made from quartz sand, the usual raw material. /hen plastics are recycled, however, the result is inevitably a plastic of a lower grade than the plastic from which it is derived. 3oreover, no applications have been found for grades of plastic that are lower than the currently lowest commercial grade.

/hich one of the following is a conclusion that can be properly drawn from the statements above$

A. 1roducts cannot presently be made out of plastic recycled entirely from the currently lowest commercial
grade.

B. It is impossible to ma!e glass products from recycled glass that is equal in quality to the best glass
products made from the usual raw material.

C. <lass products made from recycled glass are less e6pensive than comparable products made from
quartz sand.

D. @nless recycled plastic bears some symbol revealing its origin, not even materials scientists can
distinguish it from virgin plastic.

E. %he difference in quality between different grades of glass is not as great as that between different
grades of plastic.

#nswer+ #

5G. # company plans to develop a prototype weeding machine that uses cutting blades with optical sensors and microprocessors that distinguish weeds from crop plants by differences in shade of color. %he inventor of the machine claims that it will reduce labor costs by virtually eliminating the need for manual weeding. /hich of the following is a consideration in favor of the company9s implementing its plan to develop the prototype$ #. %here is a considerable degree of variation in shade of color between weeds of different species. '. %he shade of color of some plants tends to change appreciably over the course of their growing season. (. /hen crops are weeded manually, overall size and leaf shape are ta!en into account in distinguishing crop plants from weeds. ). "election and genetic manipulation allow plants of virtually any species to be economically bred to have a distinctive shade of color without altering their other characteristics. *. =arm laborers who are responsible for the manual weeding of crops carry out other agricultural duties at times in the growing season when e6tensive weeding is not necessary.

#nswer+ ) E planation+ If this were true, even if the plant and weed have same color, plant can be modified to have a different color and thus enable the use of prototype.

51. #ll actors are e6uberant people and all e6uberant people are e6troverts, but nevertheless it is true that some shy people are actors.

If the statements above are true, each of the following must also be true *E(*1%+

A. "ome shy people are e6troverts. B. "ome shy e6troverts are not actors.

C. "ome e6uberant people who are actors are shy. D. #ll people who are not e6troverts are not actors. E. "ome e6troverts are shy.

#nswer+ '

5.. .rieda+ Lightening causes fires and damages electronic equipment. "ince lightening rods can prevent any ma2or damage, every building should have one. Erik+ ;our recommendation is pointless. It is true that lightning occasionally causes fires, but faulty wiring and overloaded circuits cause far more fires and damage to equipment than lightening does.

*ri! s response fails to establish that =rieda s recommendation should not be acted on because his response

#. )oes not show that the benefits that would follow from =rieda s recommendation would be offset by any
disadvantages.

B. )oes not offer any additional way of lessening the ris! associated with lightening. C. #ppeals to =rieda s emotions rather than to her reason. D. Introduces an irrelevant comparison between overloaded circuits and faulty wiring E. (onfuses the notion of preventing damage with that of causing inconvenience.

#nswer+ # E planation+ *ri!9s response simply sidesteps =rieda9s reasoning, accepting the fact that lightning causes fires, but saying that other things cause a lot of fires, too. %hat9s hardly a reason for not doing something about the lightning damage. *ri!9s problem is that he gives absolutely no reason for not following Now why ' is wrong+ It9s true that *ri! fails to offer any additional way of combating lightning damage, but that9s not the point. *ven if he did have other ideas, that wouldn9t indicate that =rieda9s recommendation also shouldn9t be acted on.

55. >ousehold indebtedness, which some theorists regard as causing recession, was high preceding the recent recession, but so was the value of assets owned by households. #dmittedly, if most of the assets were owned by quite affluent households, and most of the debt was owed by low0income households, high household debt levels could have been the cause of the recession despite high asset values+ low0income households might have decreased spending in order to pay off debts while quite affluent ones might simply

have failed to increase spending. 'ut, in fact, quite affluent people must have owed most of the household debt, since money is not lent to those without assets. %herefore, the real cause must lie elsewhere.

%he argument is structured to lead to which one of the following conclusions$

A. >igh levels of household debt did not cause the recent recession. B. Low0income households succeeded in paying off their debts despite the recent recession. C. #ffluent people probably increased their spending levels during the recent recession. D. >igh levels of household debt have little impact on the economy. E. /hen people borrowed money prior to the recent recession, they did not use it to purchase assets.

#nswer+ # E planation+ %he author begins by stating the theory that household indebtedness causes recession, and then e6amines whether that theory applies to 8the recent recession.8 >e considers a hypothetical scenario in which the theory might apply, but then says that this scenario did not obtain during the recent recession. >is conclusion is that the theory doesn9t apply+ >ousehold indebtedness was not the cause of the recent recession.

57. <overnment0subsidized insurance available to homeowners ma!es it feasible for anyone to build a house on a section of coastline regularly struc! by hurricanes. *ach ma2or storm causes billions of dollars worth of damage in such coastal areas, after which owners who have insurance are able to collect an amount of money sufficient to recoup a high percentage of their losses.

%he passage provides the most support for an argument against a government bill proposing

A. %hat power companies be required to bury power lines in areas of the coastline regularly struc! by
hurricanes.

B. #n increase in funding of weather service programs that provides a hurricane watch and warning
system for coastal areas.

C. Renewal of federal funding for emergency life0support programs in hurricane0stric!en areas. D. *stablishment of an agency committed to managing coastal lands in ecologically responsible ways. E. *stablishment of a contingency fund protecting owners of uninsured houses in the coastal areas from
catastrophic losses due to the hurricane damage.

#nswer+ *

5A. /ane+ #ccording to an article in this newsmagazine, children s hand0eye coordination suffers when they spend a great amount of time watching television. %herefore, we must restrict the amount of time Nacqueline and 3ildred are allowed to watch television. #lan+ RubbishS %he article says that only children under three are affected in that way. Nacqueline is ten and 3ildred is eight. %herefore, we need not restrict their television viewing.

#lan s argument against Nane s conclusion ma!es which one of the following errors in reasoning$

#. It relies on the same source that Nane cited in support of her conclusion. '. It confuses undermining an argument in support of a given conclusion with showing that the conclusion
itself is false.

(. It does not address the main point of Nane s argument and focuses instead on a side issue. ). It ma!es an irrelevant appeal to an authority. *. It fails to distinguish the consequences of a certain practice from the causes of the practice.

#nswer+ ' E planation+ Nane9s argument is certainly undermined0 that is, the lin! between evidence and conclusion is wea!ened if not outright severed 0when #lan shows that her evidence ,the article about infants under 5- is inapplicable to their older !ids. (ertainly, the article isn9t adequate evidence for the conclusion that these !ids9 %H viewing should be restricted. Nevertheless, the conclusion itself could still be true 0 it still could be a good idea to restrict Nacqueline and 3ildred9s %H, albeit for different reasons 0 yet #lan denies that.

5B. =or similar cars and drivers, automobile insurance for collision damage has always cost more in <reatport than in =airmont. 1olice studies, however, show that cars owned by <reatport residents are, on average, slightly less li!ely to be involved in a collision than cars in =airmont. (learly, therefore, insurance companies are ma!ing a greater profit on collision0damage insurance in <reatport than in =airmont. /hich of the following is an assumption on which the argument depends$ #. Repairing typical collision damage does not cost more in <reatport than in =airmont. '. %here are no more motorists in <reatport than in =airmont. (. <reatport residents who have been in a collision are more li!ely to report it to their insurance company than =airmont residents are. ). =airmont and <reatport are the cities with the highest collision0damage insurance rates. *. %he insurance companies were already aware of the difference in the li!elihood of collisions before the publication of the police reports.

#nswer+ #

E planation+ If collision insurance costs more in <reatport, then collision damage either costs more to repair or the argument9s conclusion is correct. %hus, the argument must assume that repair costs are N:% higher in <reatport than in =airmont. 0hy other options are wron! : %he absolute number of motorists in either city doesn9t matter, since insurance premiums amount to an average of the amount that the insurance companies pay for collison repair for all their accident0prone customers, plus the companies9 profits. =rom this relationship, we can see that there are only two ways ,or a combination, thereof- that can account for higher insurance premiums in one city relative to the other. *ither the cost of repair is higher in one city ,insurance companies have to pay out more to cover their liabilities- or the insurance companies9 profits are higher in that city. %he former is a statement of the converse of # and the latter is a restatement of the argument9s conclusion. If you put the former statement into the argument as a premise, it contradicts the conclusion and wea!ens the argument, which means it9s an assumption critical to the argument.

5C. ;easts capable of leavening bread are widespread, and in the many centuries during which the ancient *gyptians made only unleavened bread, such yeasts must frequently have been mi6ed into bread doughs accidentally. %he *gyptians, however, did not discover leavened bread until about 5GGG '.(. %hat discovery roughly coincided with the introduction of a wheat variety that was preferable to previous varieties because its edible !ernel could be removed from the hus! without first toasting the grain. /hich of the following, if true, provides the strongest evidence that the two developments were causally related$ #. *ven after the ancient *gyptians discovered leavened bread and the techniques for reliably producing it were well !nown, unleavened bread continued to be widely consumed. '. :nly when the *gyptians stopped the practice of toasting grain were their stonelined grain0toasting pits available for ba!ing bread. (. >eating a wheat !ernel destroys its gluten, a protein that must be present in order for yeast to leaven bread dough. ). %he new variety of wheat, which had a more delicate flavor because it was not toasted, was reserved for the consumption of high officials when it first began to be grown. *. 'ecause the hus! of the new variety of wheat was more easily removed, flour made from it required less effort to produce.

#nswer+ ( E planation+ %he question stem as!s us to establish a causal relationship between the discovery of leavened bread by the egyptians and the introduction of the wheat variety. If heating the !ernel of this wheat destroys the gluten then yeast cannot possibly be growing on this wheat and will not be responsible for leavened bread. >owever this is not certain because the egyptians would have continued to grow other varieties of wheat where yeast could grow and be responsible for leavening. 'ut this is the closest option to chose amongst all because none of the other options made much sense to me.

5D. Ecolo!ist+ %he "cottish >ighlands were once the site of e6tensive forests, but these forests have mostly disappeared and been replaced by peat bogs. %he common view is that the >ighlands9 deforestation was caused by human activity, especially agriculture. >owever, a!riculture be!an in the Hi!hlands less than 1,222 years a!o" 1eat bogs, which consist of compressed decayed vegetable matter, build up by only about one foot per 1,GGG years and, throu!hout the Hi!hlands, remains of trees in peat bo!s are almost all at depths !reat than four feet" "ince climate changes that occurred between C,GGG and 7,GGG years ago

favored the development of peat bogs rather than the survival of forests, the deforestation was more li!ely the result of natural processes than of human activity. In the ecologist9s argument the two portions in boldface play which of the following roles$ #. %he first is evidence that has been used in support of a position that the ecologist re2ects& the second is a finding that the ecologist uses to counter that evidence. '. %he first is evidence that, in light of the evidence provided in the second, serves as grounds for the ecologist9s re2ection of a certain position. (. %he first is a position that the ecologist re2ects& the second is evidence that has been used in support of that position. ). %he first is a position that the ecologist re2ects& the second provides evidence in support of that re2ection. *. %he first is a position for which the ecologist argues& the second provides evidence to support that position.

#nswer+ ' E planation+ %he first is evidence that, in light of the evidence provided in the second, serves as grounds for the ecologist9s re2ection of a certain position.

5F. In e6periments in which certain !inds of bacteria were placed in a generous supply of nutrients, the populations of bacteria grew rapidly, and genetic mutations occurred at random in the populations. %hese e6periments show that all genetic mutation is random.

/hich one of the following, if true, enables the conclusion to be properly drawn$

A. *ither all genetic mutations are random or none are random. B. %he bacteria tested in the e6periments were of e6tremely common forms. C. If all genetic mutations in bacteria are random, then all genetic mutations in every other life form are
random also.

D. %he !ind of environment in which genetic mutation ta!es place has no effect on the way genetic
mutation occurs.

E. %he nutrients used were the same as those that nourish the bacteria in nature.

#nswer+ # E planation+ %his question is a perfect e6ample of scope0shift. %he evidence is that, in one particular environment, some types of bacteria e6perienced random mutations. %he conclusion is that all genetic mutation is random. %he credited response needs to lin! this specific e6ample to the general conclusion. (ombine the stimulus with ,#-. %he stimulus states that some genetic mutations are random, and ,#- states that either all mutations are random or none are. If some mutations are random, then all mutations must be

random& the alternative option isn9t possible. If ,#- is true, all genetic mutations are random, which is our conclusion. "ince the stimulus doesn9t tell us that all genetic mutation in bacteria is random ,it only mentions certain !inds of bacteria-, answer choice ,(- doesn9t allow us to conclude anything. ,(-9s conclusion would only be true if all genetic mutations in bacteria were random.

7G. *ach )ecember 51 in (ountry ?, a tally is made of the country s total available coal supplies P that is, the total amount of coal that has been mined throughout the country but not consumed. In 1FF1 that amount was considerably lower than it had been in 1FFG. =urthermore, (ountry ? has nor imported or e6ported coal since 1FCG.

If the statements above are true, which one of the following must also be true on the basis of them$

A. In (ountry ?, more coal was mined in 1FFG than was mined in 1FF1. B. In (ountry ?, the amount of coal consumed in 1FF1 was greater than the amount of coal mined in 1FF1. C. In (ountry ?, the amount of coal consumed in 1FFG was greater than the amount of coal consumed in
1FF1.

D. In (ountry ?, the amount of coal consumed in 1FF1 was greater than the amount of coal consumed in
1FFG.

E. In (ountry ?, more coal was consumed during the first half of 1FF1 than was consumed during the first
half of 1FFG.

#nswer+ ' E planation+ =irst loo! at the wrong choices+ ,#-, ,(-, ,)-, ,*- %he tally represents a comparison between the amount of coal mined and consumed. #ny change in the tally from year to year therefore represents a change in this comparison. #ll of the wrong choices offer unsupported comparisons between one of the elements across years& in other words, mining in FG vs. mining in F1, or consumption in FG vs. consumption in F1. 'ut all we have is information on the tally, a number that includes both factors. /ithout raw numbers, we can9t tell how the consumption numbers compared from 1FFG to 1FF1& we need to !now the amount mined before ma!ing the 2udgments in ,(and ,)-. %he opposite holds for ,#-& we need the consumption figures before concluding how much was mined in FG as opposed to F1. ,*- has an even greater problem+ it introduces a new concept0the first half of FG and F10 which we !now nothing about. 'mportant tip: 0e must assume that the term 3total available coal supplies3 refers to the cumulative stockpile carried over from year to year" *hat way, we can ri!htly infer 456 !iven the decrease in the tally from 7882 to 7887

71. "pectroscopic analysis has revealed the e6istence of frozen nitrogen, methane and carbon mono6ide on the surface of 1luto. "uch ices have a tendency to vaporize, producing an atmosphere. "ince the proportion of any gas in such an atmosphere depends directly on how readily the corresponding ice vaporizes, astronomers have concluded that the components of 1luto s atmosphere are nitrogen, carbon mono6ide and methane, in order of decreasing abundance.

%he astronomer s argument relies on which one of the following assumptions$

A. %here is no more frozen nitrogen on the surface of 1luto than there is either frozen carbon mono6ide or
methane.

B. @ntil space probes reach 1luto, direct analysis of the atmosphere is impossible. C. %here is no frozen substance on the surface of 1luto that vaporizes more readily than methane but less
readily than carbon mono6ide.

D. Nitrogen is found in the atmosphere of a planet only if nitrogen ice is found on the surface of that planet. E. # mi6ture of nitrogen, carbon mono6ide and methane is characteristic of the substances from which the
solar system formed.

#nswer+ ( E planation+ %remise9.act+ %he e6istence of frozen nitrogen, methane, and carbon mono6ide on 1luto9s surface :ule+ %he proportion of any gas in such an atmosphere as 1luto9s depends directly on how readily the corresponding ice vaporizes. $onclusion+ 1luto9s atmosphere is made up of nitrogen, carbon mono6ide, and methane in decreasing order of abundance. In order for them to conclude that 1luto9s atmosphere is made up of )ust these gases in )ust this order, they must be assuming that there aren9t any other frozen substances on 1luto9s surface that would vaporize and 2oin the ran!s of gases. (IJ "ays this all.

7.. New types of washing machines designed to consume less energy also e6tract less water from laundry during their final spin cycles than do washing machines that consume somewhat more energy. %he wetter the laundry, the more energy required to dry it in an automatic dryer. %hus using these new types of washing machines could result in an overall increase in the energy needed to wash and dry a load of laundry. In which one of the following is the pattern of reasoning most parallel to that in the argument above$ ,#- %he more s!ill required to operate a machine, the harder it is to find people able to do it, and thus the more those people must be paid. %herefore, if a factory installs machines that require highly s!illed operators, it must be prepared to pay higher wages. ,'- %here are two routes between (enterville and 3apletown, and the scenic route is the longer route. %herefore, a person who is not concerned with how long it will ta!e to travel between (enterville and 3apletown will probably ta!e the scenic route. ,(- %he more people who wor! in the library9s reading room, the noisier the room becomes& and the noisier the wor!ing environment, the less efficiently people wor!. %herefore, when many people are wor!ing in the

reading room, those people are wor!ing less efficiently. ,)- 1ine is a less e6pensive wood than cedar but is more susceptible to rot. :utdoor furniture made from wood susceptible to rot must be painter with more e6pensive paint. %herefore, building outdoor furniture from pine rather than cedar could increase the total cost of building and painting the furniture. ,*- %he more weights added to an e6ercise machine, the greater the muscle strength needed to wor! out on the machine. @p to a point, using more muscle strength can ma!e a person stronger. %hus an e6ercise machine with more weights can, but does not necessarily, ma!e a person stronger

#nswer+ ) E planation+ 1. 1ine is susceptible to rot but is less e6pensive ,less e6pensive is the benefit to ta!e not off I equivalent of lowering energy consumption in the stimulus. .. :utdoor furniture made from wood susceptible to rot must be painted with more e6pensive paint ,opposite of less e6pensive goal for going with 1ine in the first place-. %herefore, building outdoor furniture from pine rather than cedar could increase the total cost of building and painting the furniture. ,'ingo 0 the original intent is eventually negated because of a factor inherent in the original intent-

75. %he petrochemical industry claims that chemical waste dumps pose no threat to people living near them. If this is true, then why do they locate the plants in sparsely populated regions$ 'y not locating the chemical dumps in densely populated areas the petrochemical industry tacitly admits that these chemicals are potentially dangerous to the people living nearby. /hich of the following, if true, would most wea!en the author9s argument$ ,#- =unding through the environmental "uper =und to clean up poorly run waste dumps is reserved for rural areas only. ,'- @ntil chemical dumps are proven 1GGT safe, it would be imprudent to locate them were they could potentially do the most harm. ,(- Locating the dumps in sparsely populated areas is less e6pensive and involves less government red tape. ,)- %he potential for chemicals to leach into the water table has in the past been underestimated. ,*- 1eople in cities are more li!ely to sue the industry if their health is harmed by the dumps. #nswer: ,(E planation: %he suppressed false premise of the argument is that all things being equal there is no reason to prefer locating the sites in sparsely populated areas. %o wea!en the argument, we need to show it is not true that all things are equal. In other words, there are advantages other than safety in locating the sites in sparsely populated areas. (hoice ,(- gives two possible advantages00cost and ease. >ence ,(- is the answer. 77. %he news media is often accused of being willing to do anything for ratings. >owever, recent action by a television networ! indicates that the news media is sometimes guided by moral principle. %his networ! had discovered through polling voters on the east coast that the Republican candidate for 1resident had garnered enough votes to ensure victory before the polls closed on the west coast. >owever, the networ! withheld this information until the polls on the west coast closed so that the information would not affect the outcome of !ey congressional races. /hich one of the following most strengthens the argument$ ,#- %he networ! had endorsed the Republican candidate for 1resident. ,'- %he networ! e6pected its ratings to increase if it predicted the winner of the presidential race, and to decrease if did not predict the winner. ,(- # rival networ! did predict a winner of the presidential race before the polls on the west coast closed. ,)- %he networ! believed that it would receive higher ratings by not predicting the winner of the presidential race.

,*- %he networ! feared that predicting the winner of the presidential race could so anger (ongress that it might enact legislation preventing all future polling outside of voting centers. #nswer: ,'E planation: %he suppressed premise in this argument is that the networ! hurt itself by not predicting the winner of the presidential race, or at least did not help itself. %o strengthen the argument, we need to show that this assumption is true. (hoice ,'- implies that this is the case by stating that the networ! e6pected to lose ratings if it did not predict a winner. >ence the answer is ,'-. 7A. %o avoid economic collapse, Russia must increase its <N1 by .GT. >owever, due to the structure of its economy, if the .GT threshold is reached, then a 7GT increase in <N1 is achievable. #ssuming that the above statements are true, which one of the following must also be true$ ,#- If ethnic strife continues in Russia, then a .GT increase in <N1 will be unattainable. ,'- If a 7GT increase in Russia9s <N1 is impossible, its economy will collapse. ,(- If Russia9s <N1 increases by 7GT, its economy will not collapse. ,)- If the .GT threshold is reached, then a 7GT increase in <N1 is achievable and a BGT increase is probable. ,*- If Russia9s economy collapses, then it will not have increased its <N1 by 7GT. #nswer: ,'E planation: )iagramming will show this seemingly difficult problem to be simply an application of the contrapositive rule of logic+ in an if0then statement, negating the conclusion also negates the premise. %he sentence 8%o avoid economic collapse, Russia must increase its <N1 by .GT8 can be reworded as 8if Russia does not increase its <N1 by .GT, its economy will collapse.8 %his in turn can be symbolized as not12;<=$ollapse /here the arrow, 0J, stands for 8if ..., then ....

Ne6t, symbolize the clause 8if the .GT threshold is reached, then a 7GT increase is achievable8 as 12;<=>2; #pplying the contrapositive to this statement yields not>2;<=not12;

@sing the transitive property ,If a I b and b I c, then a I c- to combine this with the first symbol statement yields not>2;<=$ollapse In other words, if a 7GT increase in <N1 is unattainable, the economy will collapse. %his is precisely what choice ,'- states. %he answer is ,'-. 7B. :ebecca+ /hen I went hi!ing in the mountains the other day, every bird that scolded me was a "teller9s Nay, and every "teller9s Nay I saw scolded me. /hich one of the following statements can be inferred from Rebecca9s observations$ ,#- %he only 2ays that Rebecca saw while hi!ing were "teller9s Nays. ,'- %here were no <ray Nays in the area where Rebecca hi!ed. ,(- /hile she was hi!ing, no <ray Nays scolded Rebecca. ,)- #ll the 2ays that Rebecca saw scolded her. ,*- Rebecca did not see any <ray Nays while she was hi!ing.

#nswer: ,(E planation: %he passage contains an embedded if0then statement. 8*very bird that scolded me was a "teller9s Nay8 can be transformed into+ If the bird scolded me, then it was a "teller9s Nay. %his can be diagrammed as 5+<=+/ /here the arrow, 0J, stands for 8if ..., then ....

Oeep this diagram in mind as you consider the answer selections. ,#-. No. %he passage indicates that every bird that scolded Rebecca was a "teller9s Nay. "tating it another way, a bird scolded Rebecca if and only if it was a "teller9s Nay. %he passage doesn9t preclude the possibility that Rebecca saw other types of 2ays that didn9t scold her. ,'- No. Remember the diagram above, '"0J"N. <ray Nays are not in the equation, but the equation indicates that if Rebecca saw any <ray Nays, they didn9t scold her. ,(- ;es. Review the diagram again, '"0J"N. If a particular bird scolded Rebecca, then it must have been a "teller9s Nay, not a <ray Nay. Let9s apply the contrapositive rule of logic to the diagram ,In an if0then statement, negating the conclusion also negates the premise-+ not +/<=not 5+

# <ray Nay is not a "teller Nay. %he hypothesis of the if0then contrapositive statement, not +/<=not 5+, is thus supported. #s a result, the conclusion not 5+, must follow. No <ray Nays scolded Rebecca. ,)- No. @nless all the 2ays Rebecca saw were "teller9s Nays ,which we do not !now-, this statement does not follow. %his statement is not supported by the diagram, which is limited to "teller Nays. ,*- No. #gain, consider the diagram, '"0J"N. It does not e6clude <ray Nays, but it does not allow them to scold Rebecca. "o again, Rebecca could have seen <ray Nays, but they didn9t scold her as she hi!ed.

7C. )emocracy is the best form of government yet created. %herefore, we must be vigilant in its defense& that is, we must be prepared to defend the right to freedom. 'ecause this right is fundamental to any progressive form of government, it is clear that democracy is better than any other form of government. /hich one of the following illustrates the same flawed reasoning as found in the passage$ ,#- I never get a headache when I eat only (hinese food, nor when I drin! only wine. 'ut when I eat (hinese food and drin! wine, I get a headache. "o the combination of the two must be the cause of my headaches. ,'- %he two times I have gone to that restaurant something bad has happened. %he first time the waiter dropped a glass and it shattered all over the table. #nd after the second time I went there, I got sic!. "o why should I go there again00something bad will 2ust happen again. ,(- I would much rather live a life dedicated to helping my fellow man than one dedicated to gaining material possessions and seeing my fellow man as a competitor. #t the end of each day, the satisfaction of having helped people is infinitely greater than the satisfaction of having achieved something material. ,)- I9m obsessed with volleyball& that9s why I play it constantly. I train seven days a wee!, and I enter every tournament. "ince I9m always playing it, I must be obsessed with it. ,*- In my academic studies, I have repeatedly changed ma2ors. I decide to ma2or in each new sub2ect that I9m introduced to. Nust as a bee lights from one flower to the ne6t, tasting the nectar of each, I 2ump from one sub2ect to the ne6t getting 2ust a taste of each. #nswer: ,)E planation: %he argument in the passage is circular ,and filled with non0sequiturs-. It is incumbent on the writer to give evidence or support for the conclusion. In this argument, though, the writer first states that democracy is the best government, the rest is merely 8noise,8 until he restates the conclusion.

(hoice ,#- is a reasonably valid causation argument00eliminate. ,'- argues by generalization. #lthough it is of questionable validity, it is not circular because the conclusion, 8it will happen again,8 is not stated, nor is it implicit in the premises00eliminate. ,(- is not circular because the conclusion is mentioned only once00 eliminate. ,)- begins by stating, 8I9m obsessed with volleyball.8 It does not, however, provide compelling evidence for that claim+ training seven days a wee!, rather than indicating obsession, may be required for, say, members of the :lympic Holleyball %eam. =urthermore, the argument repeats the conclusion at the end. "o it is circular in the same manner as the original. >ence ,)- is our answer. 7D. *ither restrictions must be placed on freedom of speech or certain subversive elements in society will use it to destroy this country. "ince to allow the latter to occur is unconscionable, we must restrict freedom of speech. %he conclusion above is unsound because ,#- subversives do not in fact want to destroy the country ,'- the author places too much importance on the freedom of speech ,(- the author fails to consider an accommodation between the two alternatives ,)- the meaning of 8freedom of speech8 has not been defined ,*- subversives are a true threat to our way of life #nswer: ,(E planation: %he arguer offers two options+ either restrict freedom of speech, or lose the country. >e hopes the reader will assume that these are the only options available. %his is unwarranted. >e does not state how the so0called 8subversive elements8 would destroy the country, nor for that matter why they would want to destroy it. %here may be a third option that the author did not mention& namely, that society may be able to tolerate the 8subversives8& it may even be improved by the diversity of opinion they offer. %he answer is ,(-. 7F. *ight years ago hunting was banned in <reenfield (ounty on the grounds that hunting endangers public safety. Now the deer population in the county is si6 times what it was before the ban. )eer are invading residential areas, damaging property and causing motor vehicle accidents that result in serious in2ury to motorists. "ince there were never any hunting0related in2uries in the county, clearly the ban was not only unnecessary but has created a danger to public safety that would not otherwise e6ist.

/hich one of the following, if true, provides the strongest additional support for the conclusion above$

A. In surrounding counties, where hunting is permitted, the size of the deer population has not increased in
the last eight years.

B. 3otor vehicle accidents involving deer often result in damage to the vehicle, in2ury to the motorist, or
both.

C. /hen deer populations increase beyond optimal size, disease and malnutrition become more
widespread among the deer herds.

D. In residential areas in the county, many residents provide food and salt for deer. E. )eer can cause e6tensive damage to ornamental shrubs and trees by chewing on twigs and saplings.

#nswer+ # E planation+ =or e6ample, here we have the conclusion that, the hunting ban is unnecessary and the deer have become a menace to public safety. %his conclusion is based on the premise that hunting ban000J overgrowth in deer population 000J causing problems.

If suppose (hoice # was true, it would tell us that hunting had !ept the population growth under control. 'y !eeping the population under control, we can infer that not much danger was caused to the public. %his in turn would support the author9s conclusion that hunting ban was unnecessary and, removing the ban would limit the problems caused by the deer.

AG. 3any ma2or scientific discoveries of the past were the product of serendipity, the chance discovery of valuable findings that investigators had not purposely sought. Now, however, scientific research tends to be so costly that investigators are heavily dependent on large grants to fund their research. 'ecause such grants require investigators to provide the grant sponsors with clear pro2ections of the outcome of the proposed research, investigators ignore anything that does not directly bear on the funded research. %herefore, under the prevailing circumstances, serendipity can no longer play a role in scientific discovery.

/hich one of the following is an assumption on which the argument depends$

A. :nly findings that an investigator purposely see!s can directly bear on that investigator s research. B. In the past few scientific investigators attempted to ma!e clear predictions of the outcome of their
research.

C. )ependence on large grants is preventing investigators from conducting the type of scientific research
that those investigators would personally prefer.

D. #ll scientific investigators who provide grant sponsors with clear pro2ections of the outcome of their
research receive at least some of the grants for which they apply.

E. In general the most valuable scientific discoveries are the product of serendipity.

#nswer+ #

A1. +tudent representative+ :ur @niversity, in e6pelling a student who verbally harassed his roommate, has erred by penalizing the student for doing what he surely has a right to do+ spea! his mindS ?ean of students+ 'ut what you re saying is that our university would endorse verbal harassment. ;et surely if we did that, we would threaten the free flow of ideas that is the essence of university life.

/hich one of the following is a questionable technique that the dean of students uses in attempting to refute the student representative$ #. '. (. (hallenging the student representative s !nowledge of the process by which the student was e6pelled. Invo!ing a fallacious distinction between speech and other sorts of behavior. 3isdescribing the student representative s position, thereby ma!ing it easier to challenge.

D. ?uestioning the motives of the student representative rather than offering reasons for the conclusion
defended

E. Relying on a position of power to silence the opposing viewpoint with a threat.

#nswer+ (

A.. #dvertisement+ :f the many over0the0counter medications mar!eted for the relief of sinus headache. "ine*ase costs the least per dose. #nd "ine*ase is as effective per dose as the most effective of those other medications. "o for relief from sinus headaches, "ine*ase is the best buy. /hich of the following, if true, most seriously wea!ens the argument above$ ,#- 3ost of the over0the0counter medications mar!eted for the relief of sinus headache are equally effective per dose in providing such relief. ,'- 3any of the over0the0counter medications mar!eted for the relief of sinus headache contain the same active ingredient as "ine*ase. ,(- 1eople who suffer from frequent sinus headaches are strongly advised to consult a doctor before ta!ing any over0the0counter medication. ,)- #n over0the0counter medication that is mar!eted for the relief of symptoms of head cold is identical in composition to "ine*ase but costs less per dose. ,*- %he per dose price for any given over0the0counter medication mar!eted for the relief of sinus headache is higher for smaller pac!ages than it is for larger pac!ages.

#nswer+ ) E planation+ %he conclusion in the stimulus is that "ine*ase is the best buy, but in answer choice ), it suggests that head cold medicine has the e6act same composition but at a lesser price, which would most seriously wea!en the given conclusion. "ine*ase is the cheapest per dose in 8medications mar!eted for the relief of sinus headache.8 %he conclusion specifically says 8=or relief from sinus headaches, "ine*ase is the best buy.8 If you can show that another medicine will relieve sinus headaches 2ust as effectively but for a cheaper price, such as ), then it wea!ens the conclusion. It doesn9t matter that it9s a head cold medicine because the stimulus concludes that it9s the best value for sinus headaches period, not 2ust within medicines mar!eted for sinus headache.

A5. 3ost students are bored by history courses as they are usually taught, primarily because a large amount of time is spent teaching dates and statistic. %he best way to teach history, therefore, is to spend most class time recounting the lives of historical figures and very little on dates and statistics. *ach of the following is an assumption on which the argument depends *E(*1%+ ,#- :ne should avoid boring one9s students when teaching a history course. ,'- It is not incompatible with the attainable goals of teaching history to spend very little class time on dates and statistics. ,(- It is possible to recount the lives of historical figures without referring to dates and statistics. ,)- It is compatible with the attainable goals of teaching history to spend most class time recounting the lives of historical figures. ,*- "tudents are more bored by history courses as they are usually taught than they would be by courses that spend most class time recounting the lives of historical figures.

#nswer+ ( E planation+ the stimulus advocates teaching history by spending most class time recounting the lives of historical figures and very little on dates and statistics. it doesn t say the teaching has to be able to spend most class time recounting lives of historical figures without referring to the dates.

A7. :n completing both the course in e6perimental design and the developmental psychology course, #ngela will have earned a degree in psychology. "ince e6perimental design, which must be completed before ta!ing developmental psychology, will not be offered until ne6t term, it will be at least two terms before #ngela gets her psychology degree.

If the statements above are all true, which one of the following must also be true$ ,#- %he developmental psychology course #ngela needs to ta!e requires two terms to complete. ,'- %he course in e6perimental design is an easier course than the course in developmental psychology. ,(- %here are no prerequisites for the course in e6perimental design. ,)- #nyone who earns a degree in psychology from the university #ngela attends will have completed the course in e6perimental design. ,*- :nce #ngela completes the developmental psychology course, she will have earned a degree in psychology.

#nswer+ )

AA. It is probably within the reach of human technology to ma!e the climate of 3ars inhabitable. It might be several centuries before people could live there, even with breathing apparatuses, but some of the world s great temples and cathedrals too! centuries to build. Research efforts now are 2ustified if there is even a chance of ma!ing another planet inhabitable. 'esides, the intellectual e6ercise of understanding how the 3artian atmosphere might be changed could help in understanding atmospheric changes inadvertently triggered by human activity on *arth.

%he main point of the argument is that

A. It is probably technologically possible for human!ind to alter the climate of 3ars. B. It would ta!e several centuries to ma!e 3ars even marginally inhabitable. C. 3a!ing 3ars inhabitable is an effort comparable to building a great temple or cathedral. D. Research efforts aimed at discovering how to change the climate of 3ars are 2ustified. E. *fforts to change the climate of 3ars could facilitate understanding of the *arth s climate.

#nswer+ )

AB. %he new perfume #urora smells worse to Noan than any comparable priced perfume, and none of her friends li!es the smell of #urora as much as the smell of other perfumes. >owever, she and her friends must have a defect in their sense of smell, since 1rofessor Nameson prefers the smell of #urora to that of any other perfume and she is one of the world s foremost e6perts on the physiology of smell.

%he reasoning is flawed because it

A. (alls into question the truthfulness of the opponent rather than addressing the point at issue. B. Ignores the well0!nown fact that someone can prefer one thing to another without li!ing either very
much.

C. =ails to establish that there is widespread agreement among the e6perts in the field. D. 3a!es an illegitimate appeal to the authority of an e6pert. E. 3isrepresents the position against which it is directed.

#nswer+ )

AC. # group of children of various ages was read stories in which people caused harm, some of those people doing so intentionally and some accidentally. /hen as!ed about appropriate punishments for those who had caused harm, the younger children, unli!e the older ones, assigned punishments that did not vary according to whether the harm was done intentionally or accidentally. ;ounger children, then, do not regard people9s intentions as relevant to punishment.

/hich of the following, if true, would most seriously wea!en the conclusion above$ ,#- In interpreting these stories, the listeners had to draw on a relatively mature sense of human psychology in order to tell whether harm was produced intentionally or accidentally. ,'- In these stories, the severity of the harm produced was clearly stated. ,(- ;ounger children are as li!ely to produce harm unintentionally as are older children. ,)- %he older children assigned punishment in a way that closely resembled the way adults had assigned punishment in a similar e6periment. ,*- %he younger children assigned punishments that varied according to the severity of the harm done by the agents in the stories.

#nswer+ #

AD. %he advanced technology of s!i boots and bindings has brought a dramatic drop in the incidence of in2uries that occur on the slopes of s!i resorts+ from F in2uries per 1,GGG s!iers in 1FAG to 5 in 1FDG. #s a result, the remainder of s!i0related in2uries, which includes all in2uries occurring on the premises of a s!i resort but not

on the slopes, rose from 1G percent of all s!i0related in2uries in 1FAG to .A percent in 1FDG. %he incidence of these in2uries, including accidents such as falling down steps, increases with the amount of alcohol consumed per s!ier. I. /hich one of the following can be properly inferred from the passage$ ,#- #s the number of s!i in2uries that occur on the slopes decreases, the number of in2uries that occur on the premises of s!i resorts increases. ,'- %he amount of alcohol consumed per s!ier increased between 1FAG and 1FDG. ,(- %he technology of s!i boots and bindings affects the incidence of each type of s!i0related in2ury. ,)- If the technology of s!i boots and bindings continues to advance, the incidence of s!i0related in2uries will continue to decline. ,*- In2uries that occurred on the slopes of s!i resorts made up a smaller percentage of s!i0related in2uries in 1FDG than in 1FAG.

#nswer+ * E planation+ /e9re told that, from 1FAG to 1FDG, the percentage of all s!i0related in2uries at s!i resorts that did not occur on the slopes increased from 1GT to .AT. %hat must mean that the percentage of s!i0related in2uries at s!i resorts that did occur on the slopes must have decreased over that same period. %hin! about it this way+ there are only two possible alternatives for a s!it related in2ury+ either it occurs on the slopes or it does not occur on the slopes. "o if the share of s!i0related in2uries represented by one type of in2ury goes up, the share represented by the other type 3@"% go down. %hat9s choice ,*-.

II. /hich one of the following conflicts with information in the passage$ ,#- %he number of s!i in2uries that occurred on the slopes was greater in 1FDG than in 1FAG. ,'- # s!ier was less li!ely to be in2ured on the slopes in 1FAG than in 1FDG. ,(- %he reporting of s!i in2uries became more accurate between 1FAG and 1FDG. ,)- %he total number of s!iers dropped between 1FAG and 1FDG. ,*- "ome s!i0related in2uries occurred in 1FDG to people who were not s!iing.

#nswer+ ' E planation+ /e9re told that the incidence of on0slope in2uries decreased from F in2uries per 1GGG s!iers in 1FAG, to 5 in2uries per 1GGG s!iers in 1FDG. %hat means a s!ier was much less li!ely to suffer an on0slope in2ury in 1FDG than in 1FAG. (learly, ,'- is in direct contradiction with this.

AF. In two months, the legal minimum wage in the country of Oirlandia will increase from five Oirlandic dollars,O)A.GG- 1er hour to O)A.AG per hour. :pponents of this increase have argued that the resulting rise in wages will drive the inflation rate up. In fact its impact on wages will probably be negligible, since only a very small proportion of all Oirfandic wor!ers are currently receiving less than O)A.AG per hour. /hich of the following, if true, most seriously wea!ens the argument$ #. 3ost people in !irlandia who are currently earning the minimum wage have been employed at their current 2obs for less than a year. '. "ome firms in Oirlandia have paid wor!ers considerably less than O)A.GG per hour, in violation of !irlandic employment regulations. (. 3any businesses hire trainees at or near the minimum wage but must reward trained wor!ers by !eeping their paylevels above the pay level of trainees. ). %he greatest growth in Oirlandia9s economy in recent years has been in those sectors where wor!ers

earn wages that tend to be much higher than the minimum wage. * %he current minimum wage is insufficient for a wor!er holding only one 2ob to earn enough to support a family, even when wor!ing full time at that 2ob.

#nswer+ ' E planation+ %he argument is about the impact on wages ,in view of the increase- being very less and hence would not actually cause an increase in inflation. ' is the only option that actually says the impact on wages is considerable. "ince some wor!ers were receiving wages considerably below UA ,say, maybe U1- 0 the impact on the wages of these wor!ers is pretty high. #nd probably will cause inflation.

BG. /hen 1GG people who have not used cocaine are tested for cocaine use, on average only A will test positive. 'y contrast, of every 1GG people who have used cocaine FF will test positive. %hus, when a randomly chosen group of people is tested for cocaine use, the vast ma2ority of those who test positive will be people who have used cocaine.

# reasoning error in the argument is that the argument ,#- attempts to infer a value 2udgment from purely factual premises ,'- attributes to every member of the population the properties of the average member of the population ,(- fails to ta!e into account what proportion of the population have used cocaine ,)- ignores the fact that some cocaine users do not test positive ,*- advocates testing people for cocaine use when there is no reason to suspect that they have used cocaine.

#nswer+ (

B1. # poor farmer was fond of telling his children+ KIn this world, you are either rich or poor, and you are either honest or dishonest. #ll poor farmers are honest. %herefore, all rich farmers are dishonest.L

%he farmer s conclusion is properly drawn if the argument assumes that

A. *very honest farmer is poor B. *very honest person is a farmer C. *veryone who is dishonest is a rich farmer D. *veryone who is poor is honest E. *very poor person is a farmer

#nswer+ #

B.. (riticism that the press panders to public sentiment neglects to consider that the press is a profit0ma!ing institution. Li!e other private enterprises, it has to ma!e money to survive. If the press were not profit0 ma!ing, who would support it$ %he only alternative is subsidy and, with it, outside control. It is easy to get subsidies for propaganda, but no one will subsidize honest 2ournalism.

It can be properly inferred from the passage that if the press is

A. Not subsidized, it is in no danger of outside control B. Not subsidized, it will not produce propaganda C. Not to be subsidized, it cannot be a profit0ma!ing institution. D. %o produce honest 2ournalism, it must be a profit0ma!ing institution E. %o ma!e a profit, it must produce honest 2ournalism.

#nswer+ ) E planation+ %here is no other alternative to profit ma!ing other than subsidy. No one will subsidize honest 2ournalism.

B5. (ertain instruments used in veterinary surgery can be made either of stainless steel or of nylon. In a study of such instruments, AG complete sterilizations of a set of nylon instruments required 5.7 times the amount of energy used to manufacture that set of instruments, whereas AG complete sterilizations of a set of stainless steel instruments required ..1 times the amount of energy required to manufacture that set of instruments.

If the statements above are true, each of the following could be true *E(*1%+

A. %he AG complete sterilizations of the nylon instruments used more energy than did the AG complete
sterilizations of the stainless steel.

B. 3ore energy was required for each complete sterilization of the nylon instruments than was required to
manufacture the nylon instruments.

C. 3ore nylon instruments than stainless steel instruments were sterilized in the study. D. 3ore energy was used to produce the stainless steel instruments than was used to produce the nylon
instruments.

E. %he total cost of AG complete sterilizations of the stainless steel instruments was greater than the cost
of manufacturing the stainless steel instruments.

#nswer+ ' E planation+ =irst, note that the question as!s for the one thing that cannot be true. Ne6t, draw out the information the stimulus gives 0 and doesn9t give. %here are two sets of instruments& the same procedures are done to each& and the procedures yield similar results. It ta!es more energy to sterilize a set of these instruments than it does to manufacture them. 'ut we don9t !now how many instruments ma!e up each set, nor do we !now e6actly how much energy it ta!es to manufacture either set. It could ta!e more to ma!e nylon tools, or it could ta!e more to ma!e stainless steel, or it could ta!e equal amounts of energy for both. /e 2ust don9t !now. @nfortunately there9s no way to predict the answer here. It9s 2ust a matter of slogging through the choices until you find the right one.

B7. %o suit the needs of corporate clients, advertising agencies have successfully modified a strategy originally developed for political campaigns. %his strategy aims to provide clients with free publicity and air time by designing an advertising campaign that is controversial, thus drawing prime0time media coverage and evo!ing public comment by officials.

%he statements above, if true, most seriously undermine which one of the following assertions$

A. %he usefulness of an advertising campaign is based on solely on the degree to which the campaign s
advertisements persuade their audiences.

B. :nly a small percentage of eligible voters admit to being influenced by advertising campaigns in
deciding how to vote.

C. (ampaign managers have transformed political campaigns by ma!ing increasing use of strategies
borrowed from corporate advertising campaigns.

D. (orporations are typically more concerned with maintaining public recognition of the corporate name
than with enhancing goodwill toward the corporation.

E. #dvertising agencies that specialize in campaigns for corporate clients are not usually chosen for
political campaigns.

#nswer+ # E planation+ %his question presents an interesting twist+ It9s a 8wea!en the argument8 question, but the argument to be wea!ened is in the answer choices, and the wea!ener is the stimulus. %here we find out that an advertising strategy developed for and used by political campaigns has now successfully been applied to corporate accounts. %he strategy is to design controversial ads that will become news, generate media attention, and evo!e public responses from officials. %hus, the companies get a lot more e6posure than they pay for. /e don9t have to loo! very far for the choice that9s incompatible with this notion& choice ,#- directly violates the 8get something for nothing8 principle behind the strategy. %his strategy flies in the face of the assertion in ,#-, which says that the usefulness of an ad campaign is based solely on the degree to which the ads themselves persuade people. ,#- does not consider e6tra media coverage or public comment by officials to be relevant to an ad9s effectiveness. If the statements in the stimulus are true, then ,#- is

seriously wea!ened by the fact that some ads are successful than!s to a factor besides persuading the public0 namely because they ma!e the news and generate free publicity for the client.

BA. =amous personalities found guilty of many types of crimes in well0publicized trials are increasingly sentenced to the performance of community service, though un!nown defendants convicted of similar crimes almost always serve prison sentences. >owever, the principle of equality before the law rules out using fame and publicity as relevant considerations in the sentencing of convicted criminals.

%he statements above, if true, most strongly support which one of the following conclusions$

A. %he principle of equality before the law is rigorously applied in only a few types of criminal trials. B. %he number of convicted celebrities sentenced to community service should equal the number of
convicted un!nown defendants sentenced to community service.

C. %he principle of equality before the law can properly be overridden by other principles in some cases. D. %he sentencing of celebrities to community service instead of prison constitutes a violation of the
principle of equality before the law in many cases.

E. %he principle of equality before the law does not allow for leniency in sentencing.

BB. $orporate -fficer+ Last year was an unusually poor one for our chemical division, which has traditionally contributed about BG percent of the corporation9s profits. It is therefore encouraging that there is the following evidence that the pharmaceutical division is growing stronger+ it contributed 7A percent of the corporation9s profits, up from .G percent the previous year. :n the basis of the facts stated which of the following is the best critique of the evidence presented above$ ,#- %he increase in the pharmaceutical division9s contribution to corporation profits could have resulted largely from the introduction of single, important new product. ,'- In multidivisional corporations that have pharmaceutical divisions, over half of the corporation9s profits usually come from the pharmaceuticals. ,(- %he percentage of the corporation9s profits attributable to the pharmaceutical division could have increased even if that division9s performance had not improved. ,)- %he information cited does not ma!e it possible to determine whether the .G percent share of profits cited was itself an improvement over the year before. ,*- %he information cited does not ma!e it possible to compare the performance of the chemical and pharmaceutical divisions in of the percent of total profits attributable to each.

#nswer+ ( E planation+ I find the phrasing of the original question unusual 0 it as!s for a 9critique of the evidence presented9. *vidence is factual& you can9t offer a critique of it. ;ou can offer a critique of the interpretation of that evidence, or of a conclusion drawn from that evidence. I assume that the question is really as!ing us to find a flaw in the conclusion here, rather than as!ing us to find a 9critique of the evidence9, something which doesn9t ma!e sense.

/ith that interpretation, ( is certainly correct. /e may have had the following+ 1revious year+ 1harmaceutical )ivision+ U.Gm profit (ompany %otal+ U1GGm profit 1harmaceutical )ivision+ .GT of total profit Last year+ 1harmaceutical )ivision+ U7.AG profit (ompany %otal+ U1G profit 1harmaceutical )ivision+ 7AT of total profit =rom the above e6ample, we can see that the 1harmaceutical )ivision may have performed much worse last year than the previous year, while still accounting for a greater percentage of the overall profits of the company. %he information provided is not necessarily 9encouraging9 news about the 1harmaceutical )ivision at all.

BC. "ome philosophers of science claim that no serious scientific theory can be tested e6perimentally without ta!ing for granted some other body of scientific beliefs, the operation of the instruments0for we cannot interpret the e6perimental results without appealing to such beliefs. If this is true, then which of the following conclusions seems most li!ely$

#- #ny particular scientific theory can be consistently retained, even in the face of apparently incompatible evidence, if we are willing to give up certain other scientific beliefs. '- *6perimental evidence is really irrelevant to scientific theorizing. (- *6perimental evidence is more relevant to the testing of scientific theories than to their initial formulation. )- *6perimental evidence is more relevant to the initial formulation of scientific theories than to their testing. *- %he best scientific theories are those which are formulated in such a way as to be sub2ect to conclusive e6perimental refutation.

#nswer+ #

BD. 3arine biology had hypothesized that lobsters !ept together traps eat one another in response to hunger. 1eriodic chec!ing of lobster traps, however, has revealed instances of lobsters sharing traps together for wee!s. *ight lobsters even shared one trap together for two months without eating one another. %he marine biologists9 hypothesis, therefore, is clearly wrong. %he argument against the marine biologists9 hypothesis is based on which one of the following assumptions$ #- Lobsters not caught in lobster traps have been observed eating one another. b- %wo months is the longest !nown period during which eight or more lobsters have been trapped together. c- It is unusual to find as many as eight lobsters caught together in one single trap. d- 3embers of the other marine species sometimes eat their own !ind when no other food source are available e- #ny food that the eight lobsters in the trap might have obtained was not enough to ward off hunger.

#nswer+ *

BF. Lou observes that if flight 7GF is canceled, then the manager could not possibly arrive in time for the meeting. 'ut flight 7GF was not canceled. %herefore, Lou concludes, the manager will certainly be on time. *velyn replies that even if Lou9s premises are true, his argument is fallacious. #nd therefore, she adds, the manager will not arrive on time after all.

/hich of the following is the strongest thing that we can properly say about this discussion$ #- *velyn is mista!en in thin!ing Lou9s argument to be fallacious, and so her own conclusion is unwarranted. '- *velyn is right about Lou9s argument, but nevertheless her own conclusion is unwarranted. (- "ince *velyn is right about Lou9s argument, her own conclusion is well0supported. )- "ince *velyn is mista!en about Lou9s argument, her own conclusion must be false. *- *velyn is right about Lou9s argument, but nevertheless her own conclusion is false.

#nswer+ '

CG. #ccording to a survey of consumers conducted one wee! before the end of a national call0in campaign to decide the newest flavor of =rea! (ola, more of those surveyed responded that they en2oyed the (itrusea "wirl flavor than responded that they en2oyed any other flavor choice. Regardless of the survey results, a different flavor, Hanilla 'onanza, was the national favorite by a considerable percentage. *ach of the following, if true, contributes to a resolution of the discrepancy described above *E(*1%+ #- =rea! (ola made an announcement four days before the end of the call0in campaign stating that the third option, Heri'lue'eri, was being removed from consideration as the new flavor choice. '- %he survey was conducted only on the east coast, and the percentage of people thought to li!e Hanilla 'onanza on the east coast is much smaller than the percentage who li!ed (itrusea "wirl. (- 3ore than BGT of those responding to the survey in favor of Hanilla 'onanza stated they were li!ely to call in and vote, whereas only .AT of those supporting (itrusea "wirl claimed they would be calling in their votes. )- # smaller percentage of those favoring the Hanilla 'onanza flavor !new the call0in number than did those favoring (itrusea "wirl. *- %he entire survey was developed and conducted by members of the =rea! (ola design team that created (itrusea "wirl.

#nswer+ ) E planation+ #- H'' ,V5- removed from competition before it was over. (ompetition still going on, so H'' people might9ve voted for their second favorite instead, which could9ve been H'. "o that might e6plain discrepancy. >mm. # little wea! M tempting 0 leave in for now but cross off it find something better. '- survey was limited in geographical size. (ampaign was national. %his could definitely e6plain the discrepancy, esp. given the stats listed in the rest of this choice. *liminate. (- survey BGT of H' fans said they9d vote& only .AT of (" fans said they9d vote. #lso could definitely e6plain the discrepancy. *liminate. )- "maller T of H' fans than (" fans !new the call0in number. 3ore people ended up voting for H', so the 8smaller T of H'8 stat could still leave more people voting for H'. %his choice still allows the given circumstances to be true, yes, but it does not specifically e6plain why more survey respondents chose (", even though H' won the call0in contest. #nd that9s our tas! here 0 to resolve the parado6, not 2ust to show

that the given stats could be true. <o bac! and eliminate # 0 this is better. *- survey made by (" people 0 so could be biased. )efinitely could e6plain 0 eliminate.

C1. # study of National football League "tatistics over the last ten years reveals that the loosing team threw more interceptions than did the winning team in D. percent of the games played. %his statistics clearly indicate that interceptions contribute greatly to team losses. %he conclusion in the above argument depends on which of the following assumptions$ #. =umbles do not hurt a team9s chances of winning a game. '. # team9s chances of winning a game are greatly reduced if it throws any interceptions during a game. (. # team that throws more interceptions than its opponent does and still wins the game must have superior players. ). Interceptions do not result from a team9s falling behind in the game. *. Interceptions are harmfull primarily because they ma!e it easy for the other team to score points.

#nswer+ ) E planation+ %he conclusion is+ interceptions contribute greatly to team losses. :r E leads to ;

:ne of the assumption types is+ #n assumption that states ; does not lead to E in other words Losing a game ):*" N:% lead to Interception ). Interceptions do not result from a team9s falling behind in the game.

C.. "ome types of organisms originated through endosymbiosis, the engulfing of one organism by another so that a part of the former becomes a functioning part of the latter. #n unusual nucleomorph, a structure that contains )N# and resembles a cell nucleus, has been discovered within a plant !nown as a chlorarachniophyte. %wo versions of a particular gene have been found in the )N# of this nucleomorph, and one would e6pect to find only a single version of this gene if the nucleomorph were not the remains of an engulfed organism9s nucleus. /hich one of the following is most strongly supported by the information above$ ,#- :nly organisms of types that originated through endosymbiosis contain nucleomorphs. ,'- # nucleomorph within the chlorarachniophyte holds all of the genetic material of some other organism. ,(- Nucleomorphs originated when an organism endosymbiotically engulfed a chlorarachniophyte. ,)- %wo organisms will not undergo endosymbiosis unless at least one of them contains a nucleomorph. ,*- (hlorarachniophytes emerged as the result of two organisms having undergone endosymbiosis.

#nswer+ * E planation+

,#- Not supported by the argument. It s a strong statement to ma!e. ,'- #gain not supported. #rgument says nucleomorph contains the plants )N# not all genetic info of the other organism. ,(- :rigination of nucleomorphs is not mentioned. ,)- Not supported. ,*- "upported in the argument. (hec! the last line.

C5. %he recently negotiated North #merican =ree %rade #greement among (anada, 3e6ico, and the @nited "tates is misnamed, because it would not result in truly free trade. #dam "mith, the economist who first articulated the principles of free trade, held that any obstacle placed in the way of the free movement of goods, investment, or labor would defeat free trade. "o since under the agreement wor!ers would be restricted by national boundaries from see!ing the best conditions they could find, the resulting obstruction of the flow of trade would, from a free0trade perspective, be harmful. %he argument proceeds by ,#- ruling out alternatives ,'- using a term in two different senses ,(- citing a non0representative instance ,)- appealing to a relevant authority ,*- responding to a different issue from the one posed

#nswer+ )

C7. #ccording to a recent magazine article, of those office employees who typically wor! D hours at the office each day but sometimes say that they will wor! at home on a particular day, .A percent actually wor! less than one hour. #t the same time, over FG percent of those same office employees believe they are more productive wor!ing at home than wor!ing in their office. %he statements above, if true, best support which of the following conclusions about the office employees discussed in the article$ #. :n average, the office employees wor!ing at home for a day wor! fewer hours than office employees wor!ing at the office. '. 1G percent of the office employees are less productive wor!ing from home than wor!ing in their office. (. #t least 1A percent of the office employees do not define productivity e6clusively in terms of the number of hours wor!ed. ). #t least .A percent of the office employees can complete the same amount of wor! in one hour at home as in D hours at the office. *. "ome of the office employees ma!e statements regarding their productivity that are not in fact true. #nswer+ ( E planation+ 7@;0 A N-* 4define work in terms of hours6 =rom the premises conditional statements. ,5- states that FGT productive at home and ,.- states that .AT wor! for one hour. ,It means 1GG 0 FG W .A-T/ are doing same amount of wor! in less hour ,i.e. 1 hour or less-. >ence, 1AT are not tal!ing productivity in terms of wor!ing hours. >ence (:RR*(%. (hoose this choice. %o simplify further, we !now that FGT productive at home and we don9t !now anything about rest 1GT. #lso,

we !now that .AT wor!ed for less than 1 hour at home& those supposed as to be less productive. "o if we subtract 1GT ,@n!nown production value- from these .AT ,!nown wor!ing timing- then we can conclude that 1AT from these .AT ,wor!ing for less than 1 hour- completed their wor! within that time at home. %hat means, they are not considering time spend I production value. >ence, choose this choice.

CA. 3ost disposable plastic containers are now labeled with a code number ,from 1 to F- indicating the type or quality of the plastic. 1lastics with the lowest code numbers are the easiest for recycling plants to recycle and are thus the most li!ely to be recycled after use rather than dumped in landfills. 1lastics labeled with the highest numbers are only rarely recycled. (onsumers can ma!e a significant long0term reduction in the amount of waste that goes unrecycled, therefore, by refusing to purchase those products pac!aged in plastic containers labeled with the highest code numbers. /hich one of the following, if true, most seriously undermines the conclusion above$ ,#- %he cost of collecting, sorting, and recycling discarded plastics is currently higher than the cost of manufacturing new plastics from virgin materials. ,'- 3any consumers are unaware of the codes that are stamped on the plastic containers. ,(- # plastic container almost always has a higher code number after it is recycled than it had before recycling because the recycling process causes a degradation of the quality of the plastic. ,)- 1roducts pac!aged in plastics with the lowest code numbers are often more e6pensive than those pac!aged in the higher0numbered plastics. ,*- (ommunities that collect all discarded plastic containers for potential recycling later dump in landfills plastics with higher0numbered codes only when it is clear that no recycler will ta!e them.

#nswer+ ( E planation+ 4#6 *liminate+ :ut of scope. /ho cares about the cost, we9re tal!ing about harming the environment. 456 *liminate+ "till consistent with info given and thus irrelevant. If 8most8 people are aware, they could reduce their consumption. 4$6 "hould people !eep purchasing the lowered numbered plastics, there would be a net increase in higher numbered plastics as more lowered numbered plastics are bought and recycled. %herefore, the environment is still harmed. ( is correct. 4?6 *liminate+ :ut of scope. #gain, costs. 4E6 *liminate+ "till consistent with info given, thus irrelevant. If consumers are not purchasing the higher0 numbered plastics, there won9t be much to dump anyways.

CB. No senator spo!e at the convention unless he or she was a )emocrat. No )emocrat both spo!e at the convention and was a senator.

/hich one of the following conclusions can be correctly drawn from the statements above$ ,#- No one but senators spo!e at the convention. ,'- No )emocrat spo!e at the convention. ,(- :nly )emocrats spo!e at the convention. ,)- No senator spo!e at the convention. ,*- "ome )emocrat senators spo!e at the convention.

#nswer+ ) E planation+ %he first sentence states that only )emocrat "enators could spea! at the convention. %he second sentence states that one could only spea! at the convention or be a "enator at the convention, but one can9t do both. %herefore, no )emocratic "enators could spea! at the convention. CC. %he report released by the interior ministry states that within the past A years the national land0reclamation program has created a 1F percent increase in arable land within the country. If these figures are accurate, the program has been a huge success. "enator (o6, a distinguished mathematician and a woman of brilliance, maintains, however, that the reclamation program could not possibly have been successful. (learly, therefore, the figures cited in the report cannot be accurate. %he argument above e6hibits an erroneous pattern of reasoning most similar to that e6hibited by which one of the following$ #. #lbert9s father claims that #lbert does not !now where the spare car !eys are hidden. ;esterday however, #lbert reported that he had discovered the spare car !eys in the garage toolbo6, so his father9s claim cannot be true. '. <loria9s drama teacher claims that her policy is to give each student the opportunity to act in at least one play during the year but, since <loria, who attended every class, reports that she was not given such an opportunity the teacher9s claim cannot be true. (. #mos claims that he can hold his breath under water for a full hour. )r. %reviso, a cardiopulmonary specialist, has stated that humans are physiologically incapable of holding their breath for even half that long& so #mos9 claim cannot be true. ). *velyn reports that she got home before midnight. Robert, who always !nows the time, insists that she did not. If Robert is right, *velyn could not possibly have listened to the late news& since she admits not having listened to the late news, her report cannot be true. *. 3oira, after observing the finish of the BG0!ilometer bicycle race, reports that Lee won with #dams a distant third. Lomas, a bicycle engineering e6pert, insists, however, that Lee could not have won a race in which #dams competed& so 3oira9s report cannot be true. #nswer+ * E planation+ * is a report of an action similar to the original argument. "o something happened and she reported it. %here is a proof that it happened and 3oira watched it. %he e6pert is saying that it is wrong and cannot happen. 1robably 3oira screwed up 2ust li!e the people in original report might have. CD. In a nature reserve in India, people are sometimes attac!ed by tigers. It is believed that the tigers will only attac! people from behind. "o for the past few years many wor!ers in the reserve have started wearing mas!s depicting a human face on the bac! of their heads. /hile many area residents remain s!eptical, no wor!er wearing one of these mas!s has yet been attac!ed by a tiger. /hich of the statements below, if true, would best support the argument of those who advocate the use of the mas!$ ,#- 3any wor!ers in the nature reserve who do not wear the mas!s have been attac!ed recently by tigers. ,'- /or!ers in other nature reserves who wear similar mas!s have not been attac!ed recently by tigers. ,(- No tigers have been spotted on the nature reserve in recent years. ,)- 3any of the wor!ers who wear the mas!s also sing while they wor! in order to frighten away any tigers in the area. ,*- %he tigers have often been observed attac!ing small deer from in front rather than from behind. #nswer+ # E planation+ It9s e6tremely useful to be able to recognize common argument types on the <3#%. :ne of the most common ,perhaps %>* most common- is causation. /e can summarize this entire argument as+ 8%he reason why tigers aren9t attac!ing the wor!ers is due to the mas!s.8 or 8%he mas!s are preventing tiger attac!s.8 %o support a causation argument, we generally do one of two things+

,1- remove an alternative e6planation& or ,.- provide additional evidence lin!ing the purported cause and effect. (hoice ,#- removes an alternative e6planation. It9s possible that the reason for reduced attac!s is because the tigers aren9t attac!ing anyone at all. >owever, if the tigers are still attac!ing non0mas! wearing wor!ers, then we don9t have to worry about that possibility. (hoice ,'- doesn9t strengthen because it doesn9t help us determine if the mas!s themselves are ma!ing any difference. =or all we !now, tigers everywhere are on stri!e and are 2ust lying on the couch watching football and tal! shows. CF. # recent survey showed that many wor!ers in a certain company are dissatisfied with their 2obs. %he survey also showed that most of the dissatisfied wor!ers believe that they have little control over their 2ob assignments. %herefore, to increase wor!ers 2ob satisfaction the company s management need only concentrate on changing wor!ers beliefs regarding the degree of control they have over their 2ob assignments. /hich one of the following, if also shown by the survey, would most seriously call into question the conclusion made by the author of the passage$ ,#- %he dissatisfied wor!ers feel that their wages are too low and wor!ing conditions are unsatisfactory. ,'- %he number of wor!ers in the company who are satisfied with their 2obs is greater than the number who is dissatisfied. ,(- %he wor!ers in the company are more dissatisfied than wor!ers in other companies. ,)- 3ost people in company management believe that the wor!ers already have too much control over their wor!. ,*- %he wor!ers in the company who are satisfied with their 2obs believe that they have a lot of control over their 2ob assignments. #nswer+ # E planation+ %he conclusion 0 to increase wor!ers 2ob satisfaction the company s management need only concentrate on changing wor!ers beliefs regarding the degree of control they have over their 2ob assignments If # is true and company concentrates only on increasing 2ob satisfaction by changing wor!ers9 beliefs the company may not succeed in doing so. DG. ?octor+ Research shows that adolescents who play video games on a regular basis are three times as li!ely to develop carpal tunnel syndrome as are adolescents who do not play video games. =ederal legislation that prohibits the sale of video games to minors would help curb this painful wrist condition among adolescents. %he doctor s conclusion depends on which of the following assumptions$ #- %he ma2ority of federal legislators would vote for a bill that prohibits the sale of video games to minors. '- Not all adolescents who play video games on a regular basis suffer from carpal tunnel syndrome. (- 1laying video games is the only way an adolescent can develop carpal tunnel syndrome. )- 3ost parents would refuse to purchase video games for their adolescent children. *- %he regular playing of video games by adolescents does not produce such beneficial effects as better hand0eye coordination and improved reaction time. #nswer+ ) E planation+ %he aim of the legislation is to limit !ids9 e6posure to carpel0tunnel causing video0games. #s such, parents who can buy games for the !ids is very much within the scope of the argument. D1. #ccording to a recent study on financial roles, one0third of high school seniors say that they have Ksignificant financial responsibilities.L %hese responsibilities include, but are not limited to, contributing to food, shelter, or clothing for themselves or their families. #t the same time, a second study demonstrates that a crisis in money management e6ists for high school students. #ccording to this study, DGT of high school seniors have never ta!en a personal finance class even though the same percentage of seniors has opened ban! accounts and one0third of these account holders have bounced a chec!. /hich of the following conclusions can be properly drawn from the statements above$ #. >igh schools would be wise to incorporate personal finance classes into their core curricula. '. #t least one0third of high school seniors wor! part0time 2obs after school. (. %he number of high school seniors with significant financial responsibilities is greater than the number of

seniors who have bounced a chec!. ). #ny high school seniors who contribute to food, shelter, or clothing for themselves or their families have significant financial responsibilities. *. %he ma2ority of high school students have no financial responsibilities to their families. #nswer+ ( E planation+ DGT of the students have significant financial responsibilities. DGT have chec!ing accounts, of which, 1M5 of them have bounced a chec!. "o, 8%he number of high school seniors with significant financial responsibilities is greater than the number of seniors who have bounced a chec!8 is a concrete fact. %he other choices$ %hey might be good ideas and arguably correct as well, but ( is a fact, and therefore the best answer. D.. (alorie restriction, a diet high in nutrients but low in calories, is !nown to prolong the life of rats and mice by preventing heart disease, cancer, diabetes, and other diseases. # si60month study of 7D moderately overweight people, who each reduced their calorie inta!e by at least .A percent, demonstrated decreases in insulin levels and body temperature, with the greatest decrease observed in individuals with the greatest percentage change in their calorie inta!e. Low insulin level and body temperature are both considered signs of longevity, partly because an earlier study by other researchers found both traits in long0lived people. If the above statements are true, they support which of the following inferences$ #. (alorie restriction produces similar results in humans as it does in rats and mice. '. >umans who reduce their calorie inta!e by at least .A percent on a long0term basis will live longer than they would have had they not done so. (. (alorie inta!e is directly correlated to insulin level in moderately overweight individuals. ). Individuals with low insulin levels are healthier than individuals with high insulin levels. *. "ome individuals in the study reduced their calorie inta!e by more than .A percent. #nswer+ * E planation+ In an inference question, we need to find a choice that must be true based on one or more facts in the passage. %he second sentence states+ 8# si60month study of 7D moderately overweight people, who each reduced their calorie inta!e by at least 1@ percent, demonstrated decreases in insulin levels and body temperature, with the !reatest decrease observed in individuals with the greatest percentage change in their calorie inta!e.8 If you connect the two bold portions, you will see that it must be true that some e6perienced decreases of greater than .A percent ,otherwise, it wouldn9t ma!e sense to spea! of the 8greatest decrease8.Notice that valid inferences are not necessarily mind0boggling and don9t necessarily have to integrate the entire stimulus.

D5. %he violent crime rate ,number of violent crimes per 1,GGG residents- in 3eadowbroo! is BG percent higher now than it was four years ago. %he corresponding increase for 1ar!dale is only 1G percent. %hese figures support the conclusion that residents of 3eadowbroo! are more li!ely to become victims of violent crime than are residents of 1ar!dale. %he argument above is flawed because it fails to ta!e into account #. changes in the population density of both 1ar!dale and 3eadowbroo! over the past four years '. how the rate of population growth in 3eadowbroo! over the past four years compares to the corresponding rate for 1ar!dale (. the ratio of violent to nonviolent crimes committed during the past four years in 3eadowbroo! and 1ar!dale ). the violent crime rates in 3eadowbroo! and 1ar!dale four years ago *. how 3eadowbroo! s e6penditures for crime prevention over the past four years compare to 1ar!dale s e6penditures

#nswer+ )

E planation+ %he conclusion is+ K%hese figures support the conclusion that residents of 3eadowbroo! are more li!ely to become victims of violent crime than are residents of 1ar!dale. K *ven thought 3eadowbroo! has increased at a rate B times that of 1ar!dale over the past four years, what we don9t !now is their current rates. =or e6ample, let9s say four years ago that 3eadowbroo! had a rate of 1GG, and 1ar!dale had a rate of 1GGG. 3eadowbroo! is now at 1BG, while 1ar!dale is now at 11GG. (learly the conclusion is now invalid.

D7. # greater number of newspapers are sold in %own " than in %own %. %herefore, the citizens of %own " are better informed about ma2or world events than are the citizens of %own %.

*ach of the following, if true, wea!ens the conclusion above *E(*1%+ ,#- %own " has a larger population than %own %. ,'- 3ost citizens of %own % wor! in %own " and buy their newspapers there. ,(- %he average citizen of %own " spends less time reading newspapers than does the average citizen of %own %. ,)- # wee!ly newspaper restricted to the coverage of local events is published in %own ". ,*- %he average newsstand price of newspapers sold in %own " is lower than the average price of newspapers sold in %own %.

#nswer+ * E planation+ %he argument is that because there are more newspapers sold in one town, the residents of that town are better informed about world events. %he actual price of the newspaper is irrelevant to this claim+ even if the reason fewer newspapers are sold in %own % is that they cost more, it does not wea!en the conclusion that %own %9s residents are not as well informed about the world as those of %own ". #, on the other hand, does wea!en because it points to an alternative e6planation for why so many newspapers are sold in %own " ,it9s not that they are better informed, it is 2ust that there are more of them-. #nd, if this alternative e6planation is correct, it casts doubt on the e6planation ,conclusion- offered in the passage. >ence, after reading choice #, you should find the conclusion less convincing than you did prior to your !nowledge of choice #.

DA. &arket #nalyst+ Recent research confirms that the main cause of bad breath is bacteria build0up on the tongue. %he research also concludes that tongue scrapers, when used properly, can eliminate up to 7GT of the bacteria from the tongue. #s the effectiveness of tongue scrapers becomes more widely !nown, the mar!et for less effective breath freshening products, such as mints, gums, and sprays, will decline significantly. /hich of the following provides the best evidence that the analyst s argument is flawed$ #. "ome breath freshening products are advertised to eliminate up to 5GT of the bacteria from the tongue. '. %ongue scrapers have already been on the mar!et for a number of years. (. 3any dentists recommend regular flossing, and not the use of the tongue scraper, to combat bad breath. ). # recent survey shows that F7T of those who regularly purchase breath freshening products are aware of the effectiveness of the tongue scraper. *. "ome people buy breath freshening products for reasons other than to fight bad breath.

#nswer+ )

E planation+ %his is actually not a 8flaw in the reasoning question8. In a flaw question, all of the answer choices are putative descriptions of the author9s reasoning process. Instead of being descriptions of the author9s reasoning process, all of the answer choices here are new facts. %he question does not as! you to describe a fault or flaw in the author9s reasoning process& instead, it as!s you to find a choice that would provide the best evidence that the argument is flawed. #s such, it is a cleverly worded wea!en question. #s a wea!en question, we need to find a fact ,again all the answer choices are facts- that would ma!e the conclusion less li!ely to be true prior to our !nowledge of that fact. %he conclusion is that after learning of the effectiveness of tongue scrapers, the mar!et for alternative bacteria0!illing products ,breath0fresheners- will dwindle. (hoice ) tells us that those who buy the breath0fresheners are already aware of the effectiveness of the tongue scraper ,and yet they still buy the breath0fresheners-. ;ou should now find the conclusion less li!ely to be true.

DB. +ven+ %rade unions are traditionally regarded by governments and economists as restraints of trade, wor!ing against the complete freedom of the economy, but I believe that unions are indispensable since they are often the wor!er s only protection against e6ploitation. :avi+ I don t agree. %he e6ploitation of the wor!ers and their wor! is a normal part of ordinary trade 2ust li!e the e6ploitation of natural or other material resources.

"ven and Ravi will not be able to resolve their disagreement logically unless they ,#- define a !ey term ,'- rely on the opinions of established authorities ,(- question an unproved premise ,)- present supporting data ,*- distinguish fact from opinion

#nswer+ # E planation+ %he answer is # because the first spea!er is using the term 8e6ploitation8 in a pe2orative sense. /e can tell from the second spea!er9s response, that he did not pic! up on this pe2orative connotation. Instead, he interprets 8e6ploitation8 as 2ust 8utilizing8.

DC. %here is a great deal of geographical variation in the frequency of many surgical procedures 0 up to tenfold variation per hundred thousand people among different areas in the numbers of hysterectomies, prostatectomies and tonsillectomies. %o support a conclusion that much of the variation is due to unnecessary surgical procedures, it would be most important to establish which of the following$ a. # local board of review at each hospital e6amines the records of each operation to determine whether the surgical procedure was necessary b. %he variation is unrelated to factors ,other than the surgical procedures themselves- that influence the incidence of diseases for which surgery might be considered c. %here are several categories of surgical procedures that are often performed unnecessarily d. =or certain surgical procedures, it is difficult to determine after the operation whether the procedures were necessary or weather alternative treatment would have succeeded e. /ith respect to how often they are performed unnecessarily, hysterectomies, prostatectomies, and tonsillectomies are respective of surgical procedures in general

#nswer+ ' E planation: %he passage tells us that certain surgeries occur at differential rates across different regions. /e need to find a choice that supports the conclusion that this difference in rates is not indicative of certain surgeries being performed unnecessarily. "o before approaching the answer choices, you should as! yourself+ what other factor could account for the difference in rates$ %hen, you should answer yourself+ maybe certain diseases occur more frequently in certain places. (hoice ' matches this insight.

DD. In the years since the city of London imposed strict air0pollution regulations on local industry, the number of bird species seen in and around London has increased dramatically. "imilar air0pollution rules should be imposed in other ma2or cities.

*ach of the following is an assumption made in the argument above *E(*1%+ ,#- In most ma2or cities, air0pollution problems are caused almost entirely by local industry. ,'- #ir0pollution regulations on industry have a significant impact on the quality of the air. ,(- %he air0pollution problems of other ma2or cities are basically similar to those once suffered by London. ,)- #n increase in the number of bird species in and around a city is desirable. ,*- %he increased sightings of bird species in and around London reflect an actual increase in the number of species in the area.

#nswer+ # E planation+ 1remises of the argument are 1+ London imposed strict air0pollution regulations on local industry, .+ the number of bird species seen in and around London has increased. %he conclusion is+ "imilar air0pollution rules should be imposed in other ma2or cities. a. this is not necessarily the case. Local industry may contribute to only a small portion of air pollution. # decrease in this small amount may still be sufficient to increase the bird population. N: b. air pollution regulation has created some desirable results in increased bird sightings. ;*" c. if we want to apply the same regulations then cities must be similar. ;*" d. if they want to impose similar rules, they want to obtain similar effects, which is more birds. ;*" e. implied in the premise, birds in and around London are representative of the surrounding area. ;*"

DF. #ll of the best comedians have had unhappy childhoods. ;et, many people who have had happy childhoods are good comedians, and some good comedians who have had miserably unhappy childhoods are happy adults. If the statements in the passage are true, which one of the following (#NN:% be true$ ,#- %he proportion of good comedians who had unhappy childhoods is greater than the proportion of the best comedians who did. ,'- "ome good comedians have had unhappy childhoods and are unhappy adults. ,(- 3ost of the best comedians are happy adults. ,)- 3ore good comedians have had unhappy childhoods than have had happy childhoods. ,*- %he proportion of comedians who are happy adults is higher than the proportions who are unhappy adults.

#nswer+ # E planation+ %he answer must be #. %he first sentence says 8#ll of the best comedians have had unhappy childhoods.8 "ome of the !ood comedians have had unhappy childhoods. "ince all of the best comedians have had unhappy childhoods, it must be impossible that there is a higher fraction of good comedians with unhappy childhoods. (hoice ( while tempting could be true+ 2ust because all of the best comedians have had unhappy childhoods does not mean that any of them are unhappy as adults.

FG. Historian+ Newton developed mathematical concepts and techniques that are fundamental to modern calculus. Leibniz developed closely analogous concepts and techniques. It has traditionally been thought that these discoveries were independent. Researchers have, however, recently discovered notes of Leibniz that discuss one of Newton s boo!s on mathematics. "everal scholars have argued that since the boo! includes a presentation of Newton s calculus concepts and techniques, and since the notes were written before Leibniz own development of calculus concepts and techniques, it is virtually certain that the traditional view is false. # more cautious conclusion than this is called for, however. BLeibniz notes are limited to early sections of Newtons book, sections that precede the ones in which Newtons calculus concepts and techniques are presented"

In the historian s reasoning, the two boldfaced portions play which of the following roles$ #. %he first provides evidence in support of the overall position that the historian defends& the second is evidence that has been used to support an opposing position. '. %he first provides evidence in support of the overall position that the historian defends& the second is that position. (. %he first provides evidence in support of an intermediate conclusion that is drawn to provide support for the overall position that the historian defends& the second provides evidence against that intermediate conclusion. ). %he first is evidence that has been used to support a conclusion that the historian criticizes& the second is evidence offered in support of the historian s own position. *. %he first is evidence that has been used to support a conclusion that the historian criticizes& the second is further information that substantiates that evidence.

#nswer+ ) E planation+ 'asically this argument divides into two parts. "cholarsMhistorians. >istorians say that scholars conclude that it is virtually certain that the traditional view is false based on =IR"% ':L) sentence. >owever, historians conclude that a more cautious conclusion is needed based on "*(:N) ':L) sentence. %herefore, in order to find the answer, you need to !eep this two divergent view in mind. %hat9s what ) says. X%he first is evidence that has been used to support a conclusion that the historian criticizes& the second is evidence offered in support of the historians own position.J

F1. 3embership in the %heta )elta 1si fraternity is easily obtained by those who have a previously had strong social connections with e6isting fraternity members before college. >owever, one must have attended high school with one or more of the members in order to forge such strong social connections. 1eople who lac! these social connections because they have not attended high school with one or more current fraternity members will therefore find it difficult to 2oin the fraternity. %his argument displays flawed reasoning because it neglects to consider the possibility that #- many of those who went to high school with %): fraternity members did not themselves become members of the fraternity '- it is more important in the long run to socialize with non0fraternity members than to develop strong connections with fraternity members (- it is more difficult to forge social connections with fraternity members than with non0fraternity members )- one may easily obtain membership in the fraternity through means other than having strong social connections with e6isting members *- some current members of the fraternity did not go to high school with other members #nswer+ ) E planation+ #rgument says that people who have not been to high school and have not been socially connected with the current fraternity members will find it difficult to 2oin the fraternity. Y) says there are other easy ways of obtaining the membership other than having strong connections with the current members F.. /hich of the following, if true, provides evidence that most logically completes the argument below$ #ccording to a widely held economic hypothesis, imposing strict environmental regulations reduces economic growth. %his hypothesis is undermined by the fact that the states with the strictest environmental regulations also have the highest economic growth. %his fact does not show that environmental regulations promote growth, however, since 444444.

#. those states with the strictest environmental regulations invest the most in education and 2ob training '. *ven those states that have only moderately strict environmental regulations have higher growth than those with the least0strict regulations (. many states that are e6periencing reduced economic growth are considering wea!ening their environmental regulations ). after introducing stricter environmental regulations, many states e6perienced increased economic growth *. even those states with very wea! environmental regulations have e6perienced at least some growth

#nswer+ # E planation+ the author tal!s about economic growth in the premises and then as!s reasoning about 2ust growth in general. %his fact does not show that environmental regulations promote growth, however, since so as environmental policies does both increase economic growth and also decrease economic growth. /e have to loo! for some other reason for growth ,in general-. =or this # suggests a different reason for growth in countries with strict environmental policies

F5. #n e6perimental microwave clothes dryer heats neither air nor cloth. Rather, it heats water on clothes, thereby saving electricity and protecting delicate fibers by operating at a lower temperature. 3icrowaves are waves that usually heat metal ob2ects, but developers of a microwave dryer are perfecting a process that will prevent thin metal ob2ects such as hairpins from heating up and burning clothes.

/hich of the following, if true, most strongly indicates that the process, when perfected, will be insufficient to ma!e the dryer readily mar!etable$ ,#- 3etal snap fasteners on clothes that are commonly put into drying machines are about the same

thic!ness as most hairpins. ,'- 3any clothes that are currently placed into mechanical dryers are not placed there along with hairpins or other thin metal ob2ects. ,(- %he e6perimental microwave dryer uses more electricity than future, improved models would be e6pected to use. ,)- )rying clothes with the process would not cause more shrin!age than the currently used mechanical drying process causes. ,*- 3any clothes that are frequently machine0dried by prospective customers incorporate thic! metal parts such as decorative brass studs or buttons.

#nswer+ * E planation+ *vidence 0 e6perimental microwave not heat air or cloth 0J heats water 0 save electricity and protect fibers 0J lower operating temp 0 microwaves heat metal ob2ects 0 new process prevent thin metal ob2ects from burning clothes (onclusion %he new process will be insufficient. #- *liminate because it s outside the scope. %he passage isn9t as!ing for the thic!ness of hairpins or metal fasteners (- *liminate, using less energy is actually a bonus, so this would strengthen the argument. )- *liminate because it s outside the scope of the argument. %he passage does not mention clothing shrin!age. Loo!ing at answer ', if clothes are not dried with thin metal ob2ects then developers don9t have to worry about preventing metal ob2ects from heating up and burning clothes. %his actually strengthens the argument and ma!es the new technology more desirable. *liminate In answer *, the new technology prevents thin metal ob2ects from burning clothes. 'ut if the ma2ority of clothing incorporates thic! metal ob2ects, then clothing will still be damaged when dried with the new equipment. %his e6plains why the new process will not be readily mar!etable. #nswer should be *

F7. Researchers have found that when very overweight people, who tend to have relatively low metabolic rates, lose weight primarily through dieting, their metabolisms generally remain unchanged. %hey will thus burn significantly fewer calories at the new weight than do people whose weight is normally at that level. "uch newly thin persons will, therefore, ultimately regain weight until their body size again matches their metabolic rate.

%he conclusion of the argument above depends on which of the following assumptions$ #. Relatively few very overweight people who have dieted down to a new weight tend to continue to consume substantially fewer calories than do people whose normal weight is at that level. '. %he metabolisms of people who are usually not overweight are much more able to vary than the metabolisms of people who have been very overweight. (. %he amount of calories that a person usually burns in a day is determined more by the amount that is consumed that day than by the current weight of the individual. ). Researchers have not yet determined whether the metabolic rates of formerly very overweight individuals can be accelerated by means of chemical agents.

*. 'ecause of the constancy of their metabolic rates, people who are at their usual weight normally have as much difficulty gaining weight as they do losing it.

#nswer+ # E planation+ ',) and * are out ... as they are too broad... ( 00J If food consumption affects the calorie burning rate 00J food consumption changes metabolic rate000J contradicts the stimuli # 00J if very few people consume fewer calories then this e6plains why not many are able to remain at their new weight.

FA. # study followed a group of teenagers who had never smo!ed and trac!ed whether they too! up smo!ing and how their mental health changed. #fter one year, the incidence of depression among those who had ta!en up smo!ing was four times as high as it was among those who had not. "ince nicotine in cigarettes changes brain chemistry, perhaps thereby affecting mood, it is li!ely that smo!ing contributes to depression in teenagers. /hich of the following, if true, most strengthens the argument$ #. 1articipants who were depressed at the start of the study were no more li!ely to be smo!ers after one year than those who were not depressed. '. %he study did not distinguish between participants who smo!ed only occasionally and those who were heavy smo!ers. (. =ew, if any, of the participants in the study were friends or relatives of other participants. ). "ome participants entered and emerged from a period of depression within the year of the study. *. %he researchers did not trac! use of alcohol by the teenagers.

#nswer+ # E planation+ #. /ea!ens '. smo!ing is smo!ing, be it occasional or heavy and smo!ing would have caused depression. (. it might imply passive smo!ing, but there is no mention of it and we are not sure whether they are smo!ers or not. *. *ffect of alcohol is not in the argument.

FB. :ne state adds a C percent sales ta6 to the price of most products purchased within its 2urisdiction. %his ta6, therefore, if viewed as ta6 on income, has the reverse effect of the federal income ta6+ the lower the income, the higher the annual percentage rate at which the income is ta6ed.

%he conclusion above would be properly drawn if which of the following were assumed as a premise$ ,#- %he amount of money citizens spend on products sub2ect to the state ta6 tends to be equal across income levels. ,'- %he federal income ta6 favors citizens with high incomes, whereas the state sales ta6 favors citizens with low incomes. ,(- (itizens with low annual incomes can afford to pay a relatively higher percentage of their incomes in state sales ta6, since their federal income ta6 is relatively low. ,)- %he lower a state9s sales ta6, the more it will tend to redistribute income from the more affluent citizens

to the rest of society. ,*- (itizens who fail to earn federally ta6able income are also e6empt from the state sales ta6.

#nswer+ # E planation+ Now what is the conclusion$ %his ta6, therefore, if viewed as ta6 on income, has the reverse effect of the federal income ta6+ the lower the income, the higher the annual percentage rate at which the income is ta6ed. Now if the amount that you pay is the same then what happens to the percentage rate as it was discussed. %#EM,low income- and %#EM,high incomethe first one is definitely greater than the second and it is the way in which I reasoned it out. If we assume that people with different income levels spend different amounts then the relation as mentioned in the stimulus ,the lower the income, the higher the annual percentage rate at which the income is ta6ed- could not have been achieved. "o even if we ta!e the different approach for an assumption question ,negating the choices to get a wea!ening option-, # seems to be the best one.

FC. %he general availability of high0quality electronic scanners and color printers for computers has made the counterfeiting of chec!s much easier. In order to deter such counterfeiting, several ban!s plan to issue to their corporate customers chec!s that contain dots too small to be accurately duplicated by any electronic scanner currently available& when such chec!s are scanned and printed, the dots seem to blend together in such a way that the word 8H:I)8 appears on the chec! # questionable assumption of the plan is that #. in the territory served by the ban!s the proportion of counterfeit chec!s that are made using electronic scanners has remained appro6imately constant over the past few years '. most counterfeiters who use electronic scanners counterfeit chec!s only for relatively large amounts of money (. the smallest dots on the proposed chec!s cannot be distinguished visually e6cept under strong magnification ). most corporations served by these ban!s will not have to pay more for the new chec!s than for traditional chec!s *. the size of the smallest dots that generally available electronic scanners are able to reproduce accurately will not decrease significantly in the near future.

#nswer+ * E planation+ * says that the argument relies on the assumption that the printers will not be able to print smaller dots accurately in the near future. if the size that the scanners are accurately able to produce decreases then they are able to reproduce smaller sized dots thereby circumventing the efforts of the ban!s. I thin! 8questionable assumption8 questions should be treated 2ust li!e assumption questions. It9s questionability is irrelevant so long as it is the assumption on which the argument relies on. I suppose this assumption is questionable because it is not li!ely that the generally available printers will suddenly improve. It is more li!ely that there will be improved scanners that are brought into the mar!et.

FD. /hich of the following most logically completes the passage$ *ach species of moth has an optimal body temperature for effective flight, and when air temperatures fall much below that temperature, the moths typically have to remain inactive on vegetation for e6tended periods, leaving them highly vulnerable to predators. In general, larger moths can fly faster than smaller ones and hence have a better chance of evading flying predators, but they also have higher optimal body temperatures, which e6plains why 444444. #. large moths are generally able to maneuver better in flight than smaller moths '. large moths are proportionally much more common in warm climates than in cool climates (. small moths are more li!ely than large moths to be effectively camouflaged while on vegetation ). large moths typically have wings that are larger in proportion to their body size than smaller moths do *. most predators of moths prey not only on several different species of moth but also on various species of other insects #nswer+ # E planation+ =rom the argument 83ore body temperature 0J 3ore effective flight8. =rom this point choice # should be right

FF. #s a practical matter, the copper available for industrial use should not be thought of as limited by the quantity of copper deposits, !nown or un!nown. %he transmutation of one chemical element into another is a modern reality, through the methods of nuclear physics. %herefore, the quantity of a natural resource such as copper cannot be calculated even in principle, because copper can be made from other metals. /hich of the following, if true, is the strongest argument against the argument above$ #- #lthough it is possible that additional deposits of copper will be found, geological considerations strongly indicate that they will not amount to more than fifty0year supply. '- %he production of copper from other metals in industrial quantities would be prohibitively e6pensive in energy and materials. (- "ynthetic materials have been discovered that can serve as practical substitutes for copper in most of its uses. )- It will be impractical, in the foreseeable future, to mine any deposits of metal that may e6ist on the moon or on other planets. *- 3ethods for estimating the amount of copper available in currently !nown deposits have become very sophisticated and have proved some accurate #nswer+ ' E planation+ #lthough the passage establishes that the transmutation of one element to another is a theoretical reality, ' tells you that, in practice, it is not feasible ,bMc it is so e6pensive-. #ccordingly, we can9t ,as of yet- behave as though the supply of copper is unlimited. In order to wea!en the argument, we need to find a choice that will suggest that this process of transmutation does not establish an unlimited supply of copper. (hoice # fails to wea!en because it does not at all discuss transmutation 1GG.#t Legal "ervices, LL( last year, the average annual salary for attorneys was UCA,GGG, while the average salary for paralegals was UAG,GGG. %he average annual salary for all Legal "ervices, LL( employees was U7A,GGG. If the information above is correct, which one of the following conclusions can properly be drawn on the basis of it$ #. %here were twice as many attorneys at Legal "ervices, LL( as there were paralegals last year.

'. %here were more paralegals than attorneys at Legal "ervices, LL( last year. (. %here were two attorneys and three paralegals at Legal "ervices, LL( last year. ). %here was at least one Legal "ervices, LL( employee who earned less than the average paralegal earned last year. *. #t least one paralegal made less than UAG,GGG last year. #nswer+ ) E planation+ we !now that both lawyers and paralegals ma!e more than the average ,on average-. If that is the case, then there must be at least one employee at the firm who ma!es less than the average who will compensate for the 8pull0up8 coming from the average lawyers9 and average paralegals9 salaries.

It might be the mail boy who ma!es si6 dollars an hour or, perhaps, there are a whole bunch of first year attorneys and first year paralegals who ma!e very little, say .G!, while the established attorneys are ma!ing si6 figures ,and in the end it washes out to CA! avg for attorneys and AG! avg for paralegals-. 'ut if there wasn9t at least one employee at the firm who made less than the 7A O average, then each and every paralegal would be ma!ing more than the average and each and every attorney would be ma!ing more than the average of 7A !, which is obviously absurd. Instead, the average would be somewhere between the two numbers of CA! and AG! ,depending on the ration of lawyers to paralegals, and depending on the distribution within those two categories-, and that would falsify information in the passage ,which information the question told us to treat as correct-. 1G1.# ma2or city uses income from ta6 revenues to fund incentives for high0end retailers from out of town to open stores in its new downtown shopping district. #lthough city ta6es on such stores will generate ta6 revenues greater than the cost of the incentives, this practice is unwise. Locally based high0end retailers would open stores in the new shopping district without requiring the city to spend ta6 revenue on incentives. /hich of the following, if true, most strongly supports the city9s policy of offering cash incentives to out0of0 town retailers$ #. "ome retailers already headquartered in the city sell similar brands at similar prices to those of high0end retailers from out of town. '. %he city9s ta6 revenues have been steadily declining for the past decade. (. # longstanding city law e6empts locally0based businesses from having to pay city ta6es. ). %he practice of cities offering cash incentives to retailers began less than a decade ago. *. @nli!e retailers headquartered in the city, the high0end retailers offered cash incentives have hundreds of stores across the country.

#nswer+ ( E planation+ %he first is that of a city+ this city concludes that it should use ta6 money to fund incentives to persuade retailers from out of town to open locations in its shopping district, based on the evidence that ta6 revenue generated by these retailers will be greater than the cost of the incentives.

%he second is that of the author, who concludes that the city should not pursue this policy, based on the evidence that retailers from inside the city would move into its business district for free.

/e are as!ed to support the city9s argument, which will li!ely involve wea!ening the argument of the author against the city9s policy. :ur answer should give us additional evidence for why the city will be better off with out0of0town retailers in its shopping district, even though they must be paid to move there. (hoice (, does provide us with such information. If locally based businesses are e6empt from city ta6es, this means that Kthe city won9t gain any ta6 revenues at all from these businessesL. %herefore, despite the cost of the incentives, the city will gain greater net ta6 revenue from out0of0town retailers9 stores than from locally0 based retailers9 stores. %his weakens the author9s argument and thus stren!thens the city9s argument. 1G.. #dvertisement+ %he world9s best coffee beans come from (olombia. %he more (olombian beans in a blend of coffee, the better the blend, and no company purchases more (olombian beans than Oreemo (offee, Inc. "o it only stands to reason that if you buy a can of Oreemo9s coffee, you9re buying the best blended coffee available today. %he reasoning of the argument in the advertisement is flawed because it overloo!s the possibility that ,#- the equipment used by Oreemo to blend and pac!age its coffee is no different from that used by most other coffee producers ,'- not all of Oreemo9s competitors use (olombian coffee beans in the blends of coffee they sell ,(- Oreemo sells more coffee than does any other company ,)- Oreemo9s coffee is the most e6pensive blended coffee available today ,*- the best unblended coffee is better than the best blended coffee

#nswer+ ( E planation+ (onclusion of the argument+ if you buy a can of Oreemo9s coffee, you9re buying the best blended coffee available today 1remises ,1- %he world9s best coffee beans come from (olombia. ,.- %he more (olombian beans in a blend of coffee, the better the blend ,5- no company purchases more (olombian beans than Oreemo (offee, Inc. %herefore the argument has an inherent assumption that because !reemo purchases more coffee than any other company, the cans it ma!es have a higher blend of (olumbian beens. >owever, if Oreemo sells more coffee than any other company, then it manufacturers more coffee than any other company. >ence the e6tra (olumbian beans don9t increase the blend of individual cans of (offee, but they increase the Number of (ans, which is what is stated in (.

1G5.Lyme disease is caused by a bacterium transmitted to humans by deer tic!s. <enerally deer tic!s pic! up the bacterium while in the larval stage from feeding on infected whitefooted mice. >owever, certain other species on which the larvae feed do not harbor the bacterium. %herefore, if the population of these other species were increased, the number of tic!s acquiring the bacterium$ #nd hence the number of people contracting Lyme disease0would li!ely decline.

/hich of the following, if true, most strengthens the argument$ #. %ic!s do not suffer any adverse consequences from carrying the bacterium that causes Lyme disease in humans. '. %here are no !nown cases of a human9s contracting Lyme disease through contact with white0footed mice. (. # deer tic! feeds only once while in the larval stage. ). # single host animal can be the source of bacteria for many tic! larvae. *. None of the other species on which deer tic! larvae feed harbor other bacteria that tic!s transmit to humans.

#nswer+ ( E planation+ In the passage, part of the author9s evidence is that right now the deer tic! larvae are pic!ing up the bacteria that cause lyme disease by feeding on these infected white0footed mice. 'ecause this is part of the author9s evidence, this information must be true ,in arguments, the evidence is 8given8 to the author-.

%he question stem tells us to treat the answer choices as 8true8& in other words, as facts. =act ,(- tells us that they only feed once a year. If we put these two facts together we should be quite surprised. %hey only feed once, and yet they somehow manage to feed on these mice that transmit the bacteria.

%his suggests that the reason they are feeding on these mice is a limited availability of the infection0free food sources rather than some alternative e6planation ,such as preference-. =act ,(-, therefore, improves the li!elihood of the plan9s wor!ing& accordingly, it is a strengthener. %he other choices are either outside the scope or e6treme ,remember he was arguing that an increase in the supply of safe infection0free food sources would diminish the problem not negate it-.

1G7.:ne reason why *uropean music has had such a strong influence throughout the world, and why it is a sophisticated achievement, is that over time the original function of the music whether ritual, dance, or worship gradually became an aspect of its style, not its defining force. )ance music could stand independent of dance, for e6ample, and sacred music independent of religious worship, because each composition has so much internal coherence that the music ultimately depends on nothing but itself.

%he claims made above are compatible with each of the following *E(*1%+ ,#- #frican music has had a more powerful impact on the world than *uropean music has had. ,'- *uropean military and economic e6pansionism partially e6plains the global influence of *uropean music. ,(- %he original functions of many types of (hinese music are no longer their defining forces. ,)- 3usic that is unintelligible when it is presented independently of its original function tends to be the most sophisticated music. ,*- "ome wor!s of art lose their appeal when they are presented to serve a function other than their original one.

#nswer+ ) E planation+ %he author of the passage is e6plaining a phenomenon or, more technically, advancing an e6planation for a phenomenon. ,1henomenonIset of observed facts that demand an e6planation or a reconciliation.- %he phenomenon is 8...why *uropean music has had such a strong influence throughout the world, and why it is a sophisticated achievement...8 %he e6planation is that the various reasons for each musical form9s original e6istence ,*uropean dance music, *uropean religious music, etc- ceased to be factors informing the music9s 8defining force8& instead, they were relegated to being mere 8...aspect,s-...of style..8. In other words, the general reason *uropean music became such a sophisticated achievement is that it ,presumably, gradually- became intelligible independently from its original function. #s such, choice ) is incompatible with the passage, and must be the correct answer. 'ut what does 8compatible8 mean$ /ell, 8incompatible8 means 8contradict8, so 8compatible8 must mean 8anything that does not contradict8. "o, the four wrong answers could be true, and the right answer is something the passage will have proven false. %he more outside the scope of the passage an answer choice is, the less li!ely the passage will have proven it necessarily false, and the more li!ely it becomes something that is possibly true. (hoices ' and ( are very much outside the scope of the passage, so they very easily could be true ,Icompatible-, and can be eliminated immediately. (hoice * discusses 8wor!s of art8 but a- we don9t !now whether music qualifies as a wor! of art and b- the author has treated music in the passage as something much more than a mere 8wor! of art8. (hoice # is a bit tric!ier but it could still be true ,compatible-+ *uropean music having 8a strong influence8 clearly allows for #frican ,and other !inds of- music having a stronger influence.

1GA.3ullen has proposed to raise ta6es on the rich, who made so much money during the past decade. ;et 3ullen s ta6 records show heavy investment in business during that time and large profits& so 3ullen s proposal does not deserve our consideration.

%he flawed reasoning in the argument above is most similar to the flawed reasoning in which one of the following$ ,#- )o not vote for "mith s proposed legislation to subsidize child care for wor!ing parents& "mith is a wor!ing parent. ,'- )o not put any credence in )r. >an s recent proposal to ban smo!ing in all public places& )r. >an is a heavy smo!er. ,(- %he previous witness s testimony ought to be ignored& he has been convicted of both forgery and mail fraud. ,)- 'oard member %imm s proposal to raise the salaries of the company s middle managers does not deserve to be considered& %imm s daughter is a middle manager at the company s headquarters. ,*- )r. /asow s analysis of the design of this bridge should not be ta!en seriously& after all, )r. /asow has previously only designed factory buildings.

#nswer+ ' E planation+ %he author argues that 3ullen9s proposal should not be considered bMc 3ullen is a member of the group that would be detrimental by that proposal ,he would have to pay more ta6es-. "imilarly (hoice '9s author argues that )r. >an9s proposal should not be considered bMc )r. >an is a member of the group that would be detrimental by that proposal ,his freedom of choice would be limited-. (hoice # is tempting but rather than being harmed by the proposal, "mith would benefit from it.

1GB. )ear #pplicant+ %han! you for your application. @nfortunately, we are unable to offer you a position in our local government office for the summer. #s you !now, funding for summer 2obs is limited, and it is impossible for us to offer 2obs to all those who want them. (onsequently, we are forced to re2ect many highly qualified applicants. /hich of the following can be inferred from the letter$ ,#- %he number of applicants for summer 2obs in the government office e6ceeded the number of summer 2obs available. ,'- %he applicant who received the letter was considered highly qualified. ,(- Hery little funding was available for summer 2obs in the government office. ,)- %he application of the person who received the letter was considered carefully before being re2ected. ,*- 3ost of those who applied for summer 2obs were considered qualified for the available positions.

#nswer+ # E planation+ #n inference is something that must be true based on the passage. "o the right answer to an inference question is something that must be true while the four wrong answers are things that could be false. If a choice could be false, it is wrong. "o you can as! of each answer choice+ could it be false$ (ould choice ' be false$ ;es, it can. Nust because they re2ect highly qualified applicants, does not mean that this particular applicant was highly qualifed. In order to thin! that this choice must be true, you would have to assume that there was no other reason ,other than the applicant being highly qualifed- for why the letter0writer would tell the applicant that they had to re2ect many highly qualifed applicants. 'ut there is an alternative reason+ civility. %he moment you see that you would have to assume something else in order for the choice to be necessarily true is the moment you !now the choice could be false, and therefore, wrong. :n the other hand, could choice # be false$ (hoice # states 8%he number of applicants for summer 2obs in the government office e6ceeded the number of summer 2obs available.8 If this were false, it would mean that they did not have enough applicants to fill up all the 2obs. (an that be the case$ /ell, we learn in the passage that 8...it is impossible for us to offer 2obs to all those who want them.8 and that 8$onsequently, we are forced to re2ect many highly qualified applicants.8 "o, the reason they are unable to offer 2obs to many highly qualified applicants is because there are not enough 2obs. %hus, choice # cannot be false...if it were false, then one of the last two sentences in the passage ,or both- would be false. #nd in an inference question we have to treat everything in the passage as necessarily true.

1GC.%here is relatively little room for growth in the overall carpet mar!et, which is tied to the size of the population. 3ost who purchase carpet do so only once or twice, first in their twenties or thirties, and then perhaps again in their fifties or si6ties. %hus as the population ages, companies producing carpet will be able to gain mar!et share in the carpet mar!et only through purchasing competitors, and not through more aggressive mar!eting. /hich one of the following, if true, casts the most doubt on the conclusion aboveC ,#- 3ost of the ma2or carpet producers mar!et other floor coverings as well. ,'- 3ost established carpet producers mar!et several different brand names and varieties, and there is no remaining niche in the mar!et for new brands to fill. ,(- %wo of the three mergers in the industry s last ten years led to a decline in profits and revenues for the newly merged companies. ,)- 1rice reductions, achieved by cost0cutting in production, by some of the dominant firms in the carpet

mar!et are causing other producers to leave the mar!et altogether. ,*- %he carpet mar!et is unli!e most mar!ets in that consumers are becoming increasingly resistant to new patterns and styles #nswer+ ) E planation+ %he conclusion is+ 8as the population ages, companies producing carpet will be able to gain mar!et share in the carpet mar!et only through purchasing competitors, and not through more aggressive mar!eting8. %his conclusion is a prediction. /henever the author advances a prediction, one necessary assumption he ma!es is+ whatever must happen for the prediction to come to pass will happen. #nd notice how e6treme the conclusion is+ the :NL; way they can gain mar!et share is through purchase of competitors. "o, the author is assuming there are no other ways mar!et share can be gained ,this is what must happen in order for the prediction to happen& if there are other ways they can gain mar!et share, then the prediction will not necessarily happen-. In a wea!en question, we find a choice that attac!s the assumption...that is what choice ) is doing.

1GD.#mphibian populations are declining in numbers worldwide. Not coincidentally, the earth s ozone layer has been continuously depleted throughout the last AG years. #tmospheric ozone bloc!s @H0', a type of ultraviolet radiation that is continuously produced by the sun, and which can damage genes. 'ecause amphibians lac! hair, hide, or feathers to shield them, they are particularly vulnerable to @H0' radiation. In addition, their gelatinous eggs lac! the protection of leathery or hard shells. %hus, the primary cause of the declining amphibian population is the depletion of the ozone layer. Each of the followin!, if true, would stren!then the ar!ument ED$E%*: ,#- :f the various types of radiation bloc!ed by atmospheric ozone, @H0' is the only type that can damage genes. ,'- #mphibian populations are declining far more rapidly than are the populations of non0amphibian species whose tissues and eggs have more natural protection from @H0'. ,(- #tmospheric ozone has been significantly depleted above all the areas of the world in which amphibian populations are declining. ,)- %he natural habitat of amphibians has not become smaller over the past century. ,*- #mphibian populations have declined continuously for the last AG years.

#nswer+ #

1GF.

#nswer+ ' E planation+ %he bicyclists9 argument is that the regulation requiring them to wear helmets ought to be removed because it is aimed at a harm that only the bicyclists would suffer. In other words, the bicyclists are saying+ 8hey, only we would be hurt, and we should be allowed to do what we want with ourselves so long as it does not harm anyone else, so get rid of the helmet0rule.8 %he bicyclists are assuming that the only !ind of harm that could e6ist ,without the helmet regulation- is physical harm& that third parties won9t be harmed bMc they won9t be physically harmed. /e need to find a choice that attac!s this assumption. (hoice ' does a fine 2ob of that. Lawyers are warning that ,without the helmet0rule- everyone would have to pay more. *veryone else is a third party, and they would definitely be hurt if they had to pay more. /underlitch par! would now be able to use this information to respond effectively against the bicyclists9 claim+ 8;ou say that if we get rid of the helmet0rule only you would be hurt, but lawyers are telling us that without the helmet rule everyone would have to pay more.8 Now, had the bicyclists said 8we are the only ones who can be physically in2ured8 it would have been different from 8hurt8 to a 8third party8.

%he bicyclists9 assumption has to do with no harm befalling any third party. (hoice ) does not target this assumption because it fails to ma!e any e6plicit connection to a third party

11G. %olitician+ %hose economists who claim that consumer price increases have averaged less than 5 percent over the last year are mista!en. %hey clearly have not shopped anywhere recently. <asoline is up 1G percent over the last year& my auto insurance, 1. percent& newspapers, 1A percent& propane, 15T& bread, AG percent. %he reasoning in the politician s argument is most vulnerable to criticism on the grounds that the argument ,#- impugns the character of the economists rather than addressing their arguments ,'- fails to show that the economists mentioned are not e6perts in the area of consumer prices ,(- mista!enly infers that something is not true from the claim that it has not been shown to be so ,)- uses evidence drawn from a small sample that may well be unrepresentative ,*- attempts to persuade by ma!ing an emotional appeal

#nswer+ ) E planation: %he politician mentions a few things with price increases. >e has highlighted 2ust a few things from thousands of consumer products. %he argument tal!s about average consumer prices and the politician is tal!ing about individual price rise.

111.(Zzanne s art inspired the ne6t generation of artists, twentieth0century modernist creators of abstract art. /hile most e6perts ran! (Zzanne as an early modernist, a small few re2ect this idea. =ran[oise (achin, for e6ample, bluntly states that such an ascription is Koverplayed,L and says that (Zzanne s wor! is Ktoo often observed from a modern point of view.L

/hich one of the following statements is most strongly supported by the information above$ ,#- (Zzanne s wor! is highly controversial. ,'- (Zzanne was an early creator of abstract art. ,(- (Zzanne s wor! helped to develop modernism. ,)- 3odern art owes less to (Zzanne than many e6perts believe. ,*- (Zzanne s wor! tends to be misinterpreted as modernist.

#nswer+ ( E planation+ In an inference question, we need to find a choice that must be true based on one or more statements in the passage. %he four wrong answers are things that could be false. ;ou can as! of each choice+ 8did the stimulus prove that it has to be true or could it be false$8 Loo!ing at the choices, ,#- 3ost e6perts ran! him as a modernist but a small few re2ect this idea. "o the idea that he is 8highly controversial8 could easily be false. ,'- /e are told that the e6perts themselves are in disagreement about this idea. %herefore, it could easily be false. ,(- 'ecause (ezanne9s art 8inspired...the modernist creators of abstract art8 this choice must be true. #t this point, we would stop and choose (, but let9s loo! at the other choices. (hoices ) and * can be eliminated in one fell swoop+ although we learn what most e6perts believe and that a small few disagree with them, we never actually learn the author9s position. %herefore, these choices could easily be false.

11..%he mar!eting department recently announced that advertising in the quarterly 2ournal will cost 1A to .G percent more ne6t summer than it cost last summer. %he members of the mar!eting department argued that in spite of this increase, advertisers will continue to profit from advertising in the 2ournal, so advertising space will be no harder to sell ne6t summer than it was last summer.

/hich one of the following, if true, would most support the mar!eting department9s argument$ #. %he cost of production and distribution of products typically advertised in the 2ournal are e6pected to rise A to 1G percent in the ne6t year. '. %he system for trac!ing the number of people who subscribe to the quarterly 2ournal will change ne6t summer. (. Ne6t summer, advertising space in the 2ournal will no longer be available in bloc!s smaller than 1M7 page. ). %he amount of advertising space purchased by providers of services is increasing, while the amount of advertising space purchased by providers of products is decreasing. *. # recent survey has shown that the average amount of time people spend reading the quarterly 2ournal is increasing at a rate of 5 percent every month.

#nswer+ * E planation+ :ne assumption in the <3#% that you are e6pected to !now, I9ve found, is that any media charges advertisers for advertisements based on the circulation. %his ma!es sense+ if the advertisement is read by 1G,GGG people versus 1GG,GGG people, you9d pay more for the latter circulation. "o, the argument is that even if we charge .GT more for advertisers to place ads, they will still place ads. %his means anything that demonstrates more people will see the ads will strengthen this argument. ,#- %his wea!ens argument 0 if both mar!eting costs #N) production costs are going up, the profit potential

falls even faster. %his ma!es it >#R)*R to sell ads. ,'- %his doesn9t say anything about circulation increasing, nor if that means ads will be able to be targeted better. "o, we can9t draw anything from this. ,(- %his wea!ens argument 0 some advertisers probably li!e the smaller ,and necessarily cheaper- ads. %his ma!es it >#R)*R to sell ads. ,)- %his doesn9t say anything as to the %:%#L number of advertisers, so it9s un!nown if it will be *#"I*R or >#R)*R to sell ads. ,*- %his states that people will spend more time reading the magazine. It follows if they spend more time reading the magazine, they probably will see more ads. %his is more attractive to advertisers, thus it will be *#"I*R to sell ads.

115."amples from a ceramic vase found at a tomb in "icily prove that the vase was manufactured in <reece. "ince the occupant of the tomb died during the reign of a "icilian ruler who lived .,CGG years ago, the location of the vase indicates that there was trade between "icily and <reece .,CGG years ago.

/hich of the following is an assumption on which the argument depends$ ,#- "icilian potters who lived during the reign of the ruler did not produce wor! of the same level of quality as did <ree! potters. ,'- "icilian clay that was used in the manufacture of pottery during the ruler s reign bore little resemblance to <ree! clay used to manufacture pottery at that time. ,(- #t the time that the occupant of the tomb was alive, there were ships capable of transporting large quantities of manufactured goods between "icily and <reece. ,)- %he vase that was found at the "icilian tomb was not placed there many generations later by descendants of the occupant of the tomb. ,*- %he occupant of the tomb was not a member of the royal family to which the "icilian ruler belonged.

#nswer+ ) E planation+ %he scope of the argument is the 8location8. /here was the vase$ not the quality of the vase. If you read one more time, the author never mentions 8quality8 of vase in the premise or the conclusion. %herefore, # is the definite out of scope. ) says the vase wasn9t placed there later by descendents of the occupant of the tomb and it is the proper assumption of the argument. In other words, the descendents ,who live .1st century- can9t put the vase when their ancestor ,who lived 1.th century- passed away. %he descendents weren9t even born. ;ou !now what I mean... unless those people have a 8bac! to the future time machine8 to travel way bac! to 1.th century to attend the funeral.

117.# recent report determined that although only three percent of drivers on 3aryland highways equipped their vehicles with radar detectors, thirty0three percent of all vehicles tic!eted for e6ceeding the speed limit were equipped with them. (learly, drivers who equip their vehicles with radar detectors are more li!ely to e6ceed the speed limit regularly than are drivers who do not.

%he conclusion drawn above depends on which of the following assumptions$ ,#- )rivers who equip their vehicles with radar detectors are less li!ely to be tic!eted for e6ceeding the speed limit than are drivers who do not. ,'- )rivers who are tic!eted for e6ceeding the speed limit are more li!ely to e6ceed the speed limit regularly

than are drivers who are not tic!eted. ,(- %he number of vehicles that were tic!eted for e6ceeding the speed limit was greater than the number of vehicles that were equipped with radar detectors. ,)- 3any of the vehicles that were tic!eted for e6ceeding the speed limit were tic!eted more than once in the time period covered by the report. ,*- )rivers on 3aryland highways e6ceeded the speed limit more often than did drivers on other state highways not covered in the report.

#nswer+ ' E planation+ #J %his is opposite of the conclusion 'J %he correct answer (J %here is no evidence for this statement )J %his answer does not support the conclusion, even if we negate this assumption the conclusion will not get affected. *J :ut of scope.

11A.Rice is a staple crop in the country of 'hupet, and enough is produced each year to both meet the country9s demand and be a ma2or e6port. %his season, however, rice yields fell .GT due to infestation by rice blast fungus. 'ecause rice commands a somewhat higher price on the e6port mar!et than the domestic mar!et, economists warn that the 'hupet people will not be able to buy the rice they need. In order to help the local economy, therefore, the 1rime 3inister of 'hupet has proposed distributing state0subsidized rice coupons to each citizen.

/hich of the following, if true, most strongly calls into question the li!elihood that implementation of the 1rime 3inister9s proposal will have the desired consequence$ ,#- *ven if the rice coupons do not allow citizens of 'hupet to consume as much rice as they do in a typical year, the e6istence of the coupons will help maintain general confidence in the economy. ,'- Rice distributors in 'hupet are willing to sell rice to the government at a price below the domestic mar!et price. ,(- Rice is a fundamental component in the diet of most citizens of 'hupet, and it is their main source of vitamin '1. ,)- %he e6portation of rice ma!es up CAT of 'hupet9s foreign trade and is the direct or indirect source of employment for 1GT of the adult population. ,*- Not all citizens of 'hupet would require state0subsidized rice coupons to afford their usual rice inta!e.

#nswer+ )

E planation+ ,#- %his strengthens the argument+ %he coupons will ma!e people more confident in the economy. ,'- %his, if anything, strengthens the argument& the subsidized rice will not cost the government as much as it might otherwise. =or the most part, though, it is not relevant to the argument. ,(- #gain, this appears to strengthen the argument00it shows how important rice is to citizens. Li!e ,'though, it isn9t directly related to the economy. ,)- %his is correct. /hile the argument emphasizes the importance of the domestic mar!et, this choice shows us that the e6port mar!et has a direct effect on the health of the domestic economy. If the government acquires a large amount of rice, it is li!ely that rice e6ports will be curtailed by that amount, thus harming the economy. ,*-%his is irrelevant& the argument would still be valid if only a large percentage of citizens required the coupons.

11B./ithin (entral (ity, the high0end retailer RiverRoc! ma!es most of its sales at its flagship store in (entral 1laza, a ma2or commuter center in the business district. Nevertheless, mar!eting strategists at RiverRoc! propose increasing revenues by closing the lease on this high0rent location and focusing on its smaller satellite stores throughout the region.

/hich of the following, if true, casts most doubt on the viability of the plan by RiverRoc!9s mar!eting strategists to focus on smaller satellite stores$ ,#- 3ost of the merchandise available at RiverRoc!9s flagship store is also available at each of its satellite stores. ,'- %he frequency with which consumers who live near (entral 1laza shop at RiverRoc! is roughly equal to that of consumers who live in the suburbs, where most of the satellite stores are located. ,(- /hen RiverRoc! opened its flagship store fifteen years ago, it closed two smaller stores in the (entral (ity area. ,)- Retailers such as RiverRoc! find that smaller suburban stores e6perience more consistent sales from year to year than do flagship stores, which depend on huge sales in November and )ecember. ,*- %he sales of the flagship RiverRoc! store allow the company to devote large sums to television advertising in (entral (ity, which has a significant positive impact on satellite store sales.

#nswer+ * E planation+ %he goal of the plan is to increase revenues& the mar!eting strategists believe this will happen by closing the flagship store and focusing on smaller satellite stores. %he underlying assumption is that the high rent of the flagship store ,and, perhaps, other costs associated with maintaining the location- is not worth the resulting revenues. (onsider each choice in turn+ ,#- %his choice suggests that the satellite stores are capable of doing everything the flagship store can do, so it doesn9t cast doubt on the plan. ,'- %here9s no apparent distinction between the flagship stores and the satellite stores here& there9s nothing directly relevant to revenues. ,(- %his choice is also not directly relevant. It suggests there is some connection between opening flagship

stores and opening satellite stores, but it9s not clear what that has to do with revenue. ,)- %his would seem to strengthen the plan, if anything& the satellite stores are more consistent than the flagship. >owever, it still isn9t directly relevant+ (onsistency doesn9t necessarily translate into higher revenues. ,*- %his is correct. It shows that the revenues of the satellite stores are dependent on the results of the revenues of the flagship store. If the flagship store were closed, the revenues of the satellite stores would li!ely be affected. 11C.#s part of a plan to overhaul a car company9s lagging sales due to its image as environmentally irresponsible, consultants recently proposed the release of a new car line. %he new line of small, high0 mileage and hybrid vehicles would appeal to a set of consumers who would never have previously considered buying from the company. In that way, the company could easily retain the mar!et for its traditional vehicles while tapping into a new mar!et niche and e6panding its revenue base.

/hich of the following, if true, would most strongly support the consultants9 proposal$ ,#- %he ma2ority of cars currently manufactured by the company is large and is not !nown for their fuel efficiency. ,'- # reliable survey of the company9s previous customers showed that most of them would more strongly consider buying from the company again if it offered hybrid vehicles. ,(- (ar sales for all companies have lagged in the last two years, but are e6pected to increase in the ne6t si6 months. ,)- # new focus on small, high0mileage, and hybrid vehicles would require research and development investment greater than the company9s (*: has made in his tenure to date. ,*- #s most car companies develop small, high0mileage, and hybrid vehicles, the profit margins in this competitive arena are e6pected to shrin!.

#nswer+ ' E planation+ %he proposal involves releasing a new line of small, high0mileage, and hybrid vehicles, which don9t appear to have previously been the company9s strong suit. %he argument assumes that releasing the new line will have the effect of increasing revenue without losing its current customer base. (onsider each choice in turn+ ,#- %his choice suggests that the company has a lot of improvements to ma!e, but it doesn9t tell us whether those improvements, as suggested by the consultants, would have the desired impact. ,'- %his is correct. %he survey ,a 8reliable8 one, so we don9t have to worry about sampling issues- addresses the potential concern that the company would lose its previous customers, who were not customers of smaller or hybrid vehicles. ,(- %his wea!ens the argument somewhat, suggesting that perhaps this car company9s sales lagged only because of industry0wide declines. ,)- %his is irrelevant. %he argument is concerned with revenue, not with costs. ,*- #s with ,)-, this choice is not relevant to e6panding 8revenue8 base. 1rofit margins consider both revenue and cost. In order for this choice to be correct, we would need to !now that developing this type of vehicle would cause revenues to shrin!, not 2ust profits.

11D. Ener!y&ill $ompany #dvertisement + #n *nvironmental Impact (oalition report shows that wind power produces the fewest pollutants among all green energy alternatives. %his shows that *nergy3ill wind turbines are the best choice for powering new developments in rural areas.

/hich of the following, if true, most seriously wea!ens the argument in the advertisement$ ,#- #n earlier *nvironmental Impact (oalition report showed that solar power produces fewer pollutants than does wind power. ,'- In the last five years, the government has invested more in wind power than in any other form of green energy. ,(- %he dispersion of homes in rural areas ma!es wind power several times more e6pensive that conventional energy sources. ,)- %he difference between the number of pollutants generated by wind power and conventional power sources is quite pronounced. ,*- %he *nvironmental Impact (oalition issues reports only once a year.

#nswer+ ( E planation+ %he shift from the first sentence to the second ma!es the assumption clear. %he argument relies on the underlying claim that, because wind power produces the fewest pollutants, it is the best choice. 3ore probably, there are other considerations. %he correct choice will li!ely point out some other consideration that may outweigh the level of pollutants. ,#- # reference to an earlier ,or different- report is almost always wrong& we have no way of comparing the validity of the two reports. #nyway, it doesn9t address the assumption. ,'- %he level of government investment is similar to a separate report, as in ,#-. If anything, it strengthens the claims of wind power supporters, but to be a strengthener, we must assume that the government is ma!ing a valid choice. ,(- %his is correct. %his is a strong reason why wind power may not be the best choice for new rural areas. ,)- %his is an irrelevant comparison. %he evidence in the argument compares wind power to other green energy alternatives. If anything, this point strengthens the argument, but not very effectively. ,*- %he frequency of reports has no effect whatsoever on the argument. 11F.3ar!eting strategists at a ma2or video retailer are discussing ways to increase revenues by boosting mid0 wee! )H) rentals, which generally fall far below wee!end rentals. :ne plan to accomplish this is to e6tend the return date for )H)s rented between 3onday and /ednesday so that customers may !eep them until =riday. "ince more customers will return to the store on =ridays, they will be also be more li!ely to rent again for the wee!end. /hich of the following, if true, would indicate the most serious wea!ness in the plan above$ ,#- %he number of cler!s currently employed by the video retailer is not sufficient to handle a significant increase in business on 3onday, %uesday, and /ednesday. ,'- (ustomers who are lured by e6tended return dates for mid0wee! )H) rentals are much less li!ely than the average customer to rent a )H) on =riday.

,(- *ven if the video retailer increases its mid0wee! rentals, some customers will continue to rent only on =ridays. ,)- %he video retailer currently offers two0day rentals, so the plan would not provide an additional inducement to rent )H)s on /ednesday. ,*- %he video retailer would rent more )H)s by e6tending return dates on =riday and "aturday rentals than on 3onday through /ednesday rentals.

#nswer+ ' E planation+ %he plan9s goal is to increase revenues& the method of doing so is to increase mid0wee! )H) rentals. %he plan appears to rest of two premises+ *6tended return dates will bring in more customers during the wee!, and those customers will rent again on =riday when they return their mid0wee! rentals. %o attac! the argument, we only need to attac! one of those two claims. (onsider each choice in turn+ ,#- %his isn9t relevant. If the plan is implemented, more cler!s could be hired. ,'- %his is correct. %he argument rests on the claim that mid0wee! renters will rent again on =riday. %his choice suggests that that is unli!ely. ,(- %his doesn9t matter. %he argument is concerned with those who do rent during the wee!. ,)- %his doesn9t directly wea!en the argument, which relies on the inducement to renters on 3onday and %uesday, as well. ,*- %his is outside the scope of the argument, which is limited to whether the plan under consideration would be effective, not whether it is the most effective possible plan. 1.G. +a)itha+ "ales of Lodgewood9s specialty microbrew beers increased steadily from .GGG to .GG.. %he rise in sales was probably due in large part to the new label and ads developed by their advertising firm, which played up the microbrewery9s local history. Eyle+ %here must be another e6planation+ %he data you cite show the rise in sales started in early .GGG. ;et the company9s new advertising campaign did not go into effect until "eptember of that year. /hich of the following, if true, would most seriously wea!en the force of the ob2ection that Oyle presents to "a2itha9s e6planation$ ,#- 'eer buyers surveyed in .GG1 cited the advertising campaign as the primary reason they became aware of Lodgewood9s offerings. ,'- # nationwide beer distributor stopped placing two other specialty microbrew brands in stores in .GGG. ,(- Lodgewood9s new label was the main focus of its advertising campaign. ,)- 'uyers of specialty microbrew beers are more li!ely than the average beer consumer to e6periment with different brands and varieties of beers. ,*- Lodgewood9s new label appeared on their beer bottles in late 1FFF.

#nswer+ * E planation+ %his is a wea!en question. Oyle9s argument addresses only half of "a2itha9s claim. "he points out the effect of the new label and new ads. >e responds only to the claim about the new ads, showing why

the ads probably didn9t cause the first part of the increase in sales. #n obvious wea! point of his response, then, is that the new label may have had the effect "a2itha claimed it did. (onsider each choice in turn+ ,#- %his doesn9t limit the effectiveness of Oyle9s claim, which points out that the ads wouldn9t have caused a sales increase in early .GGG. ,'- %his choice suggests an e6ternal cause of the sales increase, but it isn9t strictly within the scope of Oyle and "a2itha9s dispute. # better choice would have to do with something "a2itha9s claim implied. ,(- 'y focusing on the advertising campaign, this choice doesn9t affect the validity of Oyle9s response. ,)- *6perimentation may benefit or not benefit Lodgewood. Li!e ,'-, it9s an e6ternal cause to the dispute between Oyle and "a2itha. ,*- %his is correct. /e9re loo!ing for something having to do with the new label, and this choice gives us a reason why, in line with "a2itha9s claim, the data shows a rise starting in early .GGG.

1.1. $ompany %resident+ Last year, 'road "treet Restaurant "uppliers switched from pen0and0paper order forms to an online process. @ne6pectedly, this resulted in a decrease in orders placed by restaurants. %hus, it is li!ely that many restaurateurs lac! the computer s!ills required by the online process and were more comfortable with the old forms. Fice<%resident of &arketin!+ Not so. "tatistics show a widespread decline in sales across the restaurant supply industry for last year.

/hich of the following, if true, most undermines the Hice 1resident of 3ar!eting9s response$ ,#- %he decline in sales across the restaurant supply industry affected the segment of the mar!et that represents the largest part of 'road "treet9s business. ,'- %he online process offered by 'road "treet allows restaurateurs to better customize their purchases and specify more precise delivery schedules. ,(- 'road "treet is !nown throughout the restaurant supply industry as a leader in customer service. ,)- )eclines in sales across the restaurant supply industry usually stem from smaller average orders placed by each restaurant, but one0quarter of 'road "treet9s previous customers placed no order at all last year. ,*- 3any restaurant supply companies have switched from pen0and0paper order forms to an online process.

#nswer+ ) E planation+ %his is a wea!en question. %he 1resident9s claim is that switching to an online process caused a decline in sales because customers were unable to wor! with the online process. %he H19s response counters that the decline is industry0wide, and thus not attributable to specifics of their own company. /e9re loo!ing for a reason why the 1resident9s claim may be correct. (onsider each choice in turn+

,#- %his choice directly strengthens the H19s response by ma!ing the industry0wide decline even more specific to 'road "treet. ,'- %he advantages o the online process are irrelevant, as the apparent problem with it is customers9 inability to wor! with it. ,(- 'road "treet9s reputation is irrelevant, as the dispute is whether a sales decrease was caused by a

switch to an online order process or a widespread decline in sales. ,)- %his is correct. It points out something typically associated with the H19s claim, and shows that something different occurred. =urther, what actually happened00one0quarter of customers not ordering00is consistent with the 1resident9s argument. ,*- /hether some of 'road "treet9s competition did the same thing that 'road "treet did does not tell us whether an industry0wide decline was responsible for 'road "treet9s lagging sales. 1... .rom an article in the 0all +treet $hronicle+ "ales statistics of ma2or electronics manufacturers with sales in the @nited "tates show that DGT of consumer electronics ,such as televisions, )H) players, and computers- sold in the @.". last year were manufactured in (hina. .rom an article in $onsumer :esults &a!azine+ %he results from last year9s survey on consumer electronics choices show that while products made in (hina are still very popular, more and more #mericans are buying products made in Napan, <ermany, and the @nited "tates. %hese three countries combined account for 5DT of products sold in the @.". last year.

=or both of the findings to be accurate, which of the following must be true$ ,#- 3ore #mericans who do not purchase consumer electronics prefer goods produced in (hina to those produced elsewhere. ,'- 3a2or electronics manufacturers do not limit their production plants to one country, often dividing different stages of manufacturing among plants around the world. ,(- 3ost consumer electronics purchased last year that were not manufactured in (hina were manufactured and sold in the @nited "tates. ,)- %he average price of a (hinese0manufactured consumer electronics device is lower than that of a device manufactured elsewhere. ,*- 3a2or electronics manufacturers sell a higher percentage of (hinese0produced consumer devices than do smaller manufacturers.

#nswer+ * E planation+ %his is an e6planationMparado6 question. =rom a quic! reading, the two reports seem to claim that DGT of electronics were manufactured in (hina, and 5DT of products were made in other places. (learly that9s incorrect00there must be more to the story. %he distinction is in the /all "treet (hronicle claim, which is limited to 8ma2or8 electronics manufacturers. %he second claim does not ma!e that distinction. %hus, it would appear that, while .GT of products sold in the @.". made by 8ma2or8 electronics manufacturers did not come from (hinese manufacturers , a greater percent of products sold in the @.". by non0ma2or manufacturers did not come from (hina. %hin! of it li!e a weighted average question. #ll manufacturers are either ma2or or non0ma2or. If .GT of the ma2or company sales were non0(hina made and 5DT of total sales were non0(hina made, then non0ma2or sales must have been greater than 5DT non0(hina made. (hoice ,*- is the only option consistent with that conclusion. If the products of non0ma2or manufacturers are more than 5DT non0(hina made, they must be much less than DGT made in (hina. %hus, ,*- is correct. 1.5./hich of the following most logically completes the argument$

Researchers have developed a blood test to screen for early signs of prostate cancer. %he test detects protein, prostate0specific antigen ,1"#-, that is produced by the prostate in greater amounts when cancer is present. %esting for 1"# can detect prostate cancer in the earliest stages in men who show no symptoms, but the diagnosis must be confirmed by additional tests due to a high rate of false positives. Recently, however, the National >ealth :rganization recommended against 1"# screening of men over CA. %his group would probably not benefit, since 4444444444.

,#- many men over CA have already been diagnosed with prostate cancer ,'- the additional tests required to confirm the diagnosis are especially ta6ing on men over CA ,(- even if early signs are present, men over CA who display no symptoms of prostate cancer are unli!ely ever to do so ,)- the fact that 1"# is present in the body does not itself strongly suggest that prostate cancer is present ,*- testing for 1"#, along with necessary follow0up tests, is very e6pensive, and many men over CA do not have private health insurance

#nswer+ ( E planation+ %esting for 1"# is beneficial because it diagnoses prostate cancer very early on, even before symptoms are present. 'ased on the structure of the last sentence, there must be some reason why 1"# testing is not beneficial for men over CA. (onsider each choice in turn+

,#- %his is irrelevant& presumably, men who already have prostate cancer would not be tested for early
signs of prostate cancer.

,'- It is not clear from this choice whether additional tests are so ta6ing that it isn9t worth identifying and
possibly treating prostate cancer& one suspects the opposite is the case.

,(- %his is correct. 'ased on the passage, it would appear that if prostate cancer is diagnosed only by 1"#
testing, it is in a very early stage. It would li!ely ta!e some time before it becomes symptomatic. If someone is unli!ely to develop symptoms, there is no reason to test for, and possibly treat, the cancer.

,)- %his choice is not specific to men over CA& in fact, the false0positive rate is already mentioned in the
passage.

,*- Li!e ,'-, it isn9t clear that this drawbac! is enough to ma!e testing not worthwhile. It also isn9t directly
relevant to the claim that men over CA 8would probably not benefit800they may well benefit, even if the test is very e6pensive. 1.7."ome large *uropean cities, such as 1aris and 'arcelona, have implemented bicycle sharing programs that allow people, for a small fee, to obtain a bi!e at any of hundreds of locations and drop it off near their destination. (urrently, most large @.". cities face congestion with cars and ta6is, have few bicycle lanes, and discourage the loc!ing of bicycles to poles and fences. %herefore, until the culture of cities becomes less hostile to bicyclists, a wide scale program will not be a viable form of alternative transportation. /hich of the following would it be most useful to determine in evaluating the argument$ ,#- /hether a sharp increase in the number of bicyclists in @.". cities would change attitudes toward bicyclists

,'- /hether @.". who drive cars !now how to operate bicycles ,(- /hether ma2or @.". cities have plans to e6pand the availability of bicycle lanes in downtown areas ,)- /hether the number of people interested in traveling by bicycle is greater in @.". than in *urope ,*- /hether small @.". cities are friendlier to bicyclists than large @.". cities

#nswer+ # E planation+ %his is an 8evaluate the argument8 question. In a sense, we9re loo!ing for an assumption, 2ust in a different format that in assumption questions. %his argument claims that, because @.". cities are not bicycle0friendly, @.". cities cannot implement *uropean0style bicycle sharing programs. *ach choice has two possible outcomes ,8whether8 it is the case, or it is not the case-, so we9re loo!ing for a choice in which one of the outcomes would have an impact on the argument. (onsider each in turn+ ,#- %his is correct. If an increase in the number of bicyclists could change attitudes toward bicyclists, a bicycle0sharing program may well solve the problem suggested in the argument. ,'- %his is not important& it doesn9t matter if people ,car0drivers or not- can operate bicycles if cities are too hostile to bicyclists. ,(- %his is outside the scope. %he problem described in the passage is not the lac! of bicycle lanes, it is the culture of @.". cities. ,)- #s with ,'-, this is not relevant, since it doesn9t address the issue of hostility to bicyclists. ,*- %his comparison is outside of the scope, as we9re concerned only with the viability of bicycle0sharing programs in large @.". cities. 1.A.)riving under the influence is a more severe problem among college students at rural and suburban universities than at urban universities. "ocial scientists wanted to determine how much of the problem is due to the distance students must drive to obtain alcohol. %hey loo!ed at police records for incidents involving students at a number of universities, and compared those with the mean distances between student housing and bars and liquor stores. %he longer the distances, the more li!ely students were to be arrested or involved in accidents. /hich of the following, if true, would it be most important to ta!e into account in evaluating the result$ ,#- 3any college students do not purchase alcohol at bars or liquor stores. ,'- "tudents at rural, suburban, and urban universities are arrested for driving under the influence more frequently than are non0student members of their communities. ,(- %he average enrollment at urban universities is greater than that at rural and suburban universities. ,)- Local sheriff9s departments near rural and suburban universities devote more resources to identifying and arresting those driving under the influence than do urban police forces. ,*- %he number of bars and liquor stores per square mile is nearly five times as high in urban areas as it is in rural and suburban areas.

#nswer+ ) E planation+ *ach one is simply a statement& it doesn9t begin with 8/hether.8 %he argument hinges on the difference between students at rural and suburban schools and those at urban schools. %he result

mentioned in the question is that the difference between those two groups of students, in terms of driving under the influence, is e6plained by the distances between housing and sources of liquor. /e9re loo!ing for a choice that would either strengthen or wea!en that claim. (onsider each choice in turn+ ,#- If we !new that 8all8 students didn9t purchase alcohol from these sources, this might cast some doubt on the claim, but 8many8 leaves an awful lot to be desired. It doesn9t directly affect the argument. ,'- %his is a different comparison00students vs. non0students. /e need a choice that focuses on the comparison drawn in the passage. ,(- *nrollment doesn9t matter, since the argument is phrased in terms of li!elihood that a certain student would drive under the influence, not the absolute number of students. ,)- %his is correct. %he conclusion was drawn based on police records. If this choice is true, students driving under the influence near rural and suburban universities are more li!ely to be caught doing so ,and thus appear in police records- than their counterparts at urban universities. ,*- %he density of bars and liquor stores doesn9t matter& it doesn9t address the distance traveled, or amount driven under the influence, of the students in these areas. 1.B. Economist+ %he law of demand predicts that as the price of a good goes down, demand for that good will increase, and vice0versa. In a recent e6periment, economists gave coupons for rice to families in a province of (hina, where it is a staple food. %he coupons effectively lowered the cost of rice, and should have led the families to buy more of it. Instead, households given the coupons purchased less rice than a control group who did not receive coupons. /hich of the following, if true, most helps to e6plain the amount of rice purchased by families who received coupons$ ,#- (hinese families spend an unusually high proportion of their income on rice. ,'- %he prices of staple goods, including foodstuffs such as rice, do not fluctuate as much as those of non0 necessary goods, such as consumer electronics. ,(- In the months before and after the e6periment, the average per0family consumption of rice in the (hinese province steadily decreased. ,)- 3any of the families given coupons discovered that they could sell the coupons on the blac! mar!et. ,*- %he availability of the coupons meant that families had more money to spend on other things, including more e6pensive substitutes for rice.

#nswer+ * E planation+ %he apparent parado6 is that, while the price of rice effectively declined, rice consumption also declined. (onsider each choice in turn, loo!ing for a reason why this might occur+ ,#- %his doesn9t tell us anything about the effect of the coupons. ,'- %he amount of fluctuation is outside the scope& we9re concerned with the effect of lowering the price of rice. ,(- %hat the average consumption of rice declined is not important& the study described in the passage included a control group, to which the households given coupons were compared. ,)- %his choice suggests that some families did not purchase rice with their coupons& however, this doesn9t e6plain why they would also purchase less rice. ,*- %his is correct. If these families had money to purchase preferable rice substitutes, they may have increased or !ept constant their consumption of rice and rice substitutes, but shifted some of their consumption from rice to substitutes because of the coupons.

1.C.)emographic e6perts predict that the global human population will reach its pea! sometime in the middle of this century, after which it will begin to decline. 1opulation growth is driven primarily by high birth rates in developing countries. It is !nown that when women have access to education and economic opportunities, they choose to have fewer children. %herefore, these e6perts propose e6panding educational and economic opportunities to women in developing countries to bring about an earlier and smaller population pea!.

/hich of the following, if true, provides the strongest grounds to doubt that the e6perts9 proposal, if adopted, will achieve its aim$ ,#- %he demographic e6perts proposing e6panding opportunities for women in developing countries made the same recommendations over twenty years ago. ,'- %he e6perts9 prediction is based on realistic assessments of the educational and economic opportunities that can be made available to women in developing countries before that time. ,(- 3any women in industrialized countries will continue to have four or more children despite access to a variety of educational and economic opportunities. ,)- %he demographic e6perts fail to e6plain why an earlier and smaller population pea! is preferable to a later and larger pea!. ,*- *6panding opportunities to women in developing countries is generally considered a positive outcome regardless of its impact on population levels.

#nswer+ ' E planation+ %his is a wea!en question. %he e6perts9 proposal involves e6panding opportunities for women so that affected women will have fewer children, thus limiting the size of the population pea! that the same e6perts have predicted. /e9re loo!ing for a choice that suggests that the proposal will not limit the size of the pea! or cause it to occur earlier. (onsider each one+ ,#- %his is irrelevant. If the proposal .G years ago was not followed ,and we don9t !now whether it was or not- the fact that it was made doesn9t tell us whether it would be effective or not today. ,'- %his is correct. %he proposal is based on the assumption that greater opportunities could be arranged for women in developing countries. If the ma6imum possible opportunities are already present, and have been built into the prediction, the proposal will not bring about any changes in the population pea!. ,(- %he argument is concerned primarily with developing countries, not industrialized countries. ,)- It doesn9t matter whether the goal is desirable& the question is concerned with whether the goal will be attained. ,*- %his is outside of the scope. It may be true, but it doesn9t bear on whether the proposal will have the desired effect. 1.D. $ity $ouncil+ (ities may issue municipal bonds to fund public pro2ects. 'ecause the interest paid to the bond holder is ta60e6empt, municipal bonds are an attractive investment. "o, to pay for the five state0of0the0 art school buildings needed to accommodate our growing student population, Northopolis should issue a ten0year, U.GG million bond, thereby paying for the buildings with revenues from an e6panding ta6 base. /hich of the following, if true, casts the most serious doubt on the li!elihood that the bond issue recommended above will have the result that is claimed$ ,#- 3ost Northopolis citizens would be reluctant to support a ta6 increase to pay for new school buildings.

,'- 'ecause municipal bond interest is ta60e6empt, bond issues can severely affect a city9s ta6 revenues for the life of the bond, despite the short0term benefits. ,(- 3any popular investments are created by pooling state and municipal bonds to create ta60e6empt inde6 funds. ,)- *stimates of the cost of five new school buildings vary from well below U.GG million to well above U.GG million. ,*- # significant percentage of municipal bonds issued by cities such as Northopolis are purchased by investors from other cities who aim to diversify their bond portfolios.

#nswer+ ' E planation+ %his is a wea!en question. %he argument ma!es a rather odd claim. 3uch of the passage concerns the benefits of municipal bonds, including a suggesting that Northopolis issue such a bond. %hen the conclusion is that the city will pay for new school buildings with ta6 revenues. 3oney from bond issues is not the same thing, so it9s unclear what one has to do with the other. %o wea!en the argument, that distinction00the assumption that issuing a bond will somehow increase ta6 revenues00must be made clear. (onsider each choice+ ,#- %his is irrelevant, as it doesn9t address either the bond issue or the effect of 8reluctance8 on ta6 revenues. ,'- %his is correct. It relates the e6istence of the proposed bond issue to ta6 revenues, which would decrease as a result of the bond issue. ,(- %his is irrelevant, as it doesn9t address ta6 revenues. ,)- #gain, this choice is irrelevant& the cost of the buildings is not what the conclusion of the argument is about. ,*- %his is outside of the scope. If anything, it wea!ens the argument by suggesting that the bond issue will not have a direct impact on local finances. 1.F.=or years, anthropologists have viewed as the inevitable result of contact between cultures, the domination and replacement of one culture by another. %his scenario was thought to have occurred when *gypt conquered its southern neighbor, Nubia, in 1AG '.(. >owever, a recent study of a burial site of high0ran!ing *gyptians in Nubia reveals that some were buried according to *gyptian tradition, and other were buried according to Nubian custom. %his finding indicates that members of both cultures may share in the ruling of a conquered region. /hich of the following, if true, most strongly supports the argument$ ,#- %he burial site also housed the remains of several visitors from nations other than *gypt and Nubia, and most of those visitors were buried according to *gyptian tradition. ,'- Nubians who were buried in *gypt during the same period were buried according to *gyptian tradition. ,(- %he burial site was not used as the resting place of *gyptians until .A years after *gypt conquered Nubia. ,)- 3ore than three0quarters of the people interred at the burial site are thought to have been Nubians. ,*- #fter *gypt conquered Nubia, few *gyptians other than colonial personnel relocated to Nubia.

#nswer+ ( E planation+ %his is a strengthen question. %he argument claims that, because of the evidence in a burial site, both the conquering *gyptians and the conquered Nubians shared in the ruling of Nubia. %he argument assumes that all of the evidence from the burial site stems from an era after *gypt conquered Nubia. If some of the evidence predated that time, the fact that some people were buried according to Nubian tradition would not say anything about who ruled the region after *gypt too! over. (onsider each choice, loo!ing for one that strengthens that assumption+ ,#- %his choice doesn9t have to do with the burial of *gyptians or Nubians, so it is outside of the scope. ,'- %his choice is also outside of scope, as it has to do with burials in *gypt, where the local culture and the ruling culture were ,as far as we !now from the passage- the same. ,(- %his is correct. %his confirms the assumption that the evidence from the burial site pertains to the period after *gypt too! over, solidifying the claim that Nubians had some control over burial practices even after the area was conquered. ,)- %he number of people buried at the site is unimportant& the focus is on the burial practices used. ,*- <iven the reference in the passage to 8high0ran!ing *gyptians,8 it seems li!ely that the argument refers mainly to colonial officials. %hus, the claim that few others moved to Nubia is not relevant to the argument. 15G. ,eolo!ist+ %he element tantalum, which originates as a mineral ore !nown as coltan, is used as a capacitor in computers and cell phones. 3ost tantalum is e6ported by (anada, (hina, "outh #merica, and #ustralia. >owever, a small percentage of the world supply comes from poorly governed or conflict0torn regions of #frica and is mined using ob2ectionable practices such as child labor. # !roup of scientists has be!un workin! on a method that ' believe will be able to detect chemical si!natures that can distin!uish tantalum samples accordin! to their location of ori!in. %heir technique needs to be further developed into a standardized and widely reproducible method of testing. If such testing becomes standard use among manufacturers and suppliers, it would allow companies and consumers a way to sanction human ri!hts abuses throu!h the marketplace"

In the geologist9s argument, the two portions in boldface play which of the following roles$ ,#- %he first presents a circumstance for which the geologist offers an e6planation& the second is part of that e6planation. ,'- %he first ac!nowledges a consideration that weighs against the conclusion of the argument& the second is that conclusion. ,(- %he first ac!nowledges a consideration that weighs against the conclusion of the argument& the second provides evidence in support of that conclusion. ,)- %he first provides evidence in support of the conclusion of the argument& the second ac!nowledges a consideration that weighs against that conclusion. ,*- %he first is a 2udgment advanced in support of the conclusion of the argument& the second is that conclusion.

#nswer+ * E planation+ %his is a boldface question. Read carefully for the structure of the argument. %he first bolded statement is a claim made by the geologist. %he second is the conclusion of the argument, which relies on that claim in the first bolded statement. ;ou9ll rarely be able to predict the e6act words used in the answer choices, but once you9ve figured out how the bold statements function, you should be able to identify a choice that matches reasonably well. In this case, choice ,*- is correct. :nly ,'- and ,*- refer to the second

statement as a conclusion& ,'- mista!enly refers to the first statement as something weighing against the conclusion.

151."ince the 1FAGs, 2ets have steadily replaced smaller propeller planes as the preferred method of transportation within the island nation of Oawaii. Nets are quieter and more comfortable for passengers, and can fly faster and higher. %he fuel efficiency of 2ets has steadily improved as much as CGT since the 1FAGs, and travel at cruising altitude is especially efficient, as motion relies mainly on aerodynamics. >owever, turboprop planes are now en2oying resurgence in Oawaii, replacing 2ets for short trips between the islands.

/hich of the following, if true, most helps to e6plain why turboprop planes are en2oying resurgence in Oawaii$ ,#- 'ecause trips within Oawaii are relatively short, planes spend little time at cruising altitude. ,'- #s 2ets have become more popular, their increased price has erased the financial advantages of their fuel efficiency. ,(- %urboprop planes have also en2oyed resurgence in other island nations, especially those with e6treme climates. ,)- 3any of the independent airlines that once served Oawaii have been purchased by larger multinational airlines. ,*- )ue to Oawaii9s remote location, 2et fuel is more e6pensive there than in most other parts of the world.

#nswer+ ' E planation+ %his is an e6planation question. %he passage describes the various benefits of 2ets00their fuel efficiency, comfort, and ability to fly higher and faster than smaller planes00but then notes that turboprop planes are en2oying resurgence. /e9re loo!ing for a choice that resolves the apparent parado6. ,#- %his choice addresses one of the benefits of 2et travel, but it doesn9t suggest why turboprop planes have overta!en 2ets despite the other clear advantages. ,'- %his is correct. /hile it ac!nowledges the benefits of 2et travel, it relates their very popularity with an increasing price that ma!es other alternatives more attractive. ,(- %his is a common form of incorrect choice. Nust because the same phenomenon is observed elsewhere does not e6plain why it is occurring as described in the passage. ,)- %his is off0topic. It doesn9t matter who owns the planes. /e9re loo!ing for a choice that e6plains why 2ets are losing popularity despite several benefits. ,*- %his choice suggests one cost of travel by 2et, but it doesn9t suggest that turboprop planes escape the same fate, nor does it confirm that the increase in cost is enough to offset the benefits of 2et operation and travel. 15..# product s design can be more valuable than the product itself. /ith the advent of the personal computer in the 1FCGs, the computer0manufacturing sector, an industry then dominated by those giants able to afford incredibly specialized !nowledge and to produce equally e6pensive products, suddenly found its brightest lights shining in the garages of clever engineers. 'ased on the information given above, which of the following was an advantage possessed by large computer manufacturers prior to the emergence of small computer0ma!ers$

#. %heir ability to produce e6pensive products '. %he substantial resources at their disposal (. %heir ability to adapt quic!ly to changes in the computer industry ). %he high quality computers they were able to produce *. %heir development of the earliest personal computer prototypes #nswer+ ' E planation+ an industry then dominated by those giants able to afford incredibly specialized !nowledge and to produce equally e6pensive products

155.@nited *nergy recently invested in a series of large windmills which are able to produce renewable energy with minimal negative effect to the environment. %he company has not drilled oil wells in the same area, even though !reater revenues and profits could be !enerated from oil wells. 'ecause any drilling would disrupt the native habitat of certain marine species in the area, some environmentalists assert that, by foregoing this drilling, Gnited Ener!y has established that it places environmental impact over financial returns. >owever, @nited *nergy may be acting in a manner consistent with its financial goals. Recent patterns of increasing annual hurricane activity have some e6perts questioning the long0term viability and profitability of oil wells in the area. %he two boldfaced portions play which of the following roles$ #. %he first supports the conclusion of the argument& the second calls that conclusion into question. '. %he first states the conclusion of the argument& the second supports that conclusion. (. %he first supports the environmentalists conclusion& the second states that conclusion. ). %he first states the environmentalists conclusion& the second provides a consideration in support of that conclusion. *. %he first supports the conclusion of the argument& the second also supports the conclusion of the argument.

#nswer+ (

157.In 1atton (ity, days are categorized as having heavy rainfall ,more than two inches-, moderate rainfall ,more than one inch, but no more than two inches-, light rainfall ,at least a trace, but no more than one inch-, or no rainfall. In 1FFG, there were fewer days with light rainfall than in 1F1G and fewer with moderate rainfall, yet total rainfall for the year was .G percent higher in 1FFG than in 1F1G.

If the statements above are true, then it is also possible that in 1atton (ity #. the number of days with heavy rainfall was lower in 1FFG than in 1F1G '. the number of days with some rainfall, but no more than two inches, was the same in 1FFG as in 1F1G (. the number of days with some rainfall, but no more than two inches, was higher in 1FFG than in 1F1G ). the total number of inches of rain that fell on days with moderate rainfall in 1FFG was more than twice what it had been in 1F1G *. the average amount of rainfall per month was lower in 1FFG than in 1F1G

#nswer+ # E planation+ %here are four !inds of days+ 1- days with no rainfall&

.- days with light rainfall, which is more than zero up to ,and including- one inch& 5- days with moderate rainfall, which is more than one and up to ,and including- two inches& and 7- days with heavy rainfall, which is anything more than two inches. In 1FFG, there were fewer days with light and moderate rainfall than in 1F1G. #lso, in 1FFG, there was more total rainfall. %his does not necessarily mean that there were more heavy rainfall days in 1FFG. Remember, we9re tal!ing ranges here. #ll of the light days in 1F1G could have been close to zero while all of the light days in 1FFG could have been closer to one. "o, the e6cess rain in 1FFG could be due either to more heavily rainy days or else the light and moderate days being more 8rainy8 than they were in 1F1G.

%herefore, choice # could be true or is possible ,matches the above deduction-. 'ecause we9ve found a match to our prediction, and because there can be only one credited response, we are done. "eriously, you are better off ta!ing an e6tra moment confirming that this could be true, then trying to prove that the other four choices must be false ,or impossible-. ;ou don9t get rewarded for figuring four reasons why four wrong answers are wrong. ,"teps 5 and 7 are predict and match-. 'ut let9s loo! at the other choices+ '. these are the days with light and moderate rainfall. %here were more of these days in 1F1G than in 1FFG. %herefore, this must be false ,impossible-. (. %his choice must be false for the same reason that choice ' must be false. ). %his is mathematically impossible. /e !now there are fewer moderately rainy days in 1FFG than in 1F1G, and the range is less than one inch. *ven if we were to ma6imize moderate rainy days in 1FFG and minimize in 1F1G, and minimize rain per moderately rainy day in 1F1G while ma6imizing rain per moderately rainy day in 1FFG, this is still impossible. =or e6ample, let9s say there are 11 moderately rainy days in 1F1G with 1.1 inch per day, and 1G moderately rainy days in 1FFG with 1.F inches per day. %hen we would have 11\1.1 I 1..1 inches of rainfall on moderately rainy days in 1F1G while we would have 1G\1.F I 1F inches of rainfall on moderately rainy days in 1FFG. It is impossible that the rainfall on moderately rainy days in 1FFG is more than twice as great as the amount of rain that fell on moderately rainy days in 1F1G. ,'ut see how long that too! to figure out$ >onestly, process of elimination is not efficient-. =inally, choice *+ *. because the total amount of rain in 1FFG was greater than it was in 1F1G, then the average rainfall per month will also be greater. =or e6ample, if total rainfall in 1F1G was 1GG and in 1FFG was 1.G ,.GT greater-, then obviously 1.GM1. is greater than 1GGM1.. %herefore, this choice also must be false ,impossible-. (hoose #.

15A.)avid is a .B years old, very successful entrepreneur and he argues that money and time invested in acquiring a degree in management administration are totally wasted. #s evidence supporting his view, he recounts the story of his brother who, at considerable e6pense of money and time, completed his 3'# at a top tier business school and then married an heiress and abandoned a promising career to stay at home and care for their children. )avid ma!es the unsupported assumption that+ #- what was unacceptable to the previous generation has no hope of being embraced today. '- 3arriage with children cannot be as fulfilling as lifelong bachelorhood. (- 1rofessional success is a function of the quality of one9s formal education. )- #n education in management administration is useful only in pursuing career related activities. *- # 3'# lost its prestige and weight it once carried in the world of business.

#nswer+ ) E planation+ )avid is not ma!ing an argument about the 8prestige and weight of an 3'#.8 Instead, he is ma!ing an argument about how useful it is+ 8)avid...argues that...that money and time invested in acquiring

a degree in management administration are totally wasted.8 /hile prestige and weight may be related ideas to usefulness, they are different from usefulness. #s such, choice * is ,slightly- outside the scope of the argument. 'ecause it is outside the scope, it can9t be the case that his argument depends on assuming it. /hy is ) right$ /ell )avid supports his conclusion ,that 3'# is a waste or not useful-, by offering up the e6ample of his brother, who got an 3'# but then did not use it in a professional or wor!0related capacity. "o, he is assuming that the :NL; /#; an 3'# can be useful ,not a waste- is if it were used to pursue a career. /hile this is a reasonable assumption, it is still an assumption ,the fact that it is a reasonable assumption ma!es it harder to identify-.

15B.1anda corp is a large is a large #merican manufacturer of children clothing that has recently underta!en sharp measures to remain competitive in today9s global mar!et. In response to offshore pricing pressure, 1anda corp laid off over AGG employees, reducing operational e6penses by 1DT. "ince clothing manufacturers realize a one percentage point of increase in sales margins for every percentage point decrease in operational costs, the board of directors is satisfied that these measures will ensure panda corp long term sustainability. /hich, if true, most wea!ens the board stance$ #- a panda corp main competitor is also an #merican manufacturer. '- 1anda corp main competitor is based in (hina. (- %he largest manufacturer of children clothes in (hina has 2ust reduced its operational costs by 1AT )- @" consumers do not want to buy clothes manufactured offshore. *- # large @" manufacturer of children clothes plans to relocate its business to (hina.

#nswer+ ( E planation+ %he correct answer is definitely (. (hoice ) is definitely incorrect. #s usual on the <3#%, there is one and only one correct answer ,it is not an issue of which answer is 8better8 than another-. %his is because if there wasn9t a categorical distinction between the correct answer and the four incorrect ones, then the test would lose its9 meaning, and would no longer be an ob2ective way of testing test0ta!ers ability. /hen wor!ing on arguments, it is important to understand the conclusion and evidence precisely. #s! yourself what the evidence establishes ,the evidence is always 8given8 to the author-. %hen as! yourself what the author is trying to prove ,i.e., conclusion-. %he author must prove every idea in the conclusion, so once you9ve summed the argument up, you should then as! yourself+ Khow are the conclusion and evidence different$L or Kwhat ideas are in the conclusion that are not in the evidence, and what ideas are in the evidence that are not in the conclusion$L %he assumption will bridge the gap between these differentially present ideas.

Let9s loo! at the 8board9s stance8 ,i.e., argument- more closely+ %he argument can basically be summed up as+ because they are getting more profits ,from the laying0off of employees-, long0term sustainability is ensured. "o, how are the conclusion and evidence different$ /ell, the evidence is about profits but the conclusion is about long0term sustainability. #re these the same$ Nope. ;ou could be very profitable but if every other company in your industry is even more profitable, then your long0term sustainability is threatened.

%his is why ( wea!ens the argument, and must be correct. =ine, 1anda (ompany will realize more profits because they 2ust lay off a bunch of employees ,AGG-. 'ut ( tells us that the largest player in the game has 2ust responded by matching this move. ,It is true that it is 1AT rather than 1DT but it would be 1AT of a bigger number since it is the largest company-. #ccordingly, (hoice ( casts doubt on the board s contention that their move ,of laying off employees- will ensure the long0term sustainability of the company. /hy is ) incorrect$ If @" consumers do not want to buy clothes manufactured offshore, then this applies to all the companies in 1anda s industry, and there is no reason to thin! that 1anda will be hurt more than any other company in its industry ,they will all be hurt-.

15C.=irms adopting Kprofit0related0payL ,1R1- contracts pay wages at levels that vary with the firm s profits. In the metalwor!ing industry last year, firms with 1R1 contracts in place showed productivity per wor!er on average 15 percent higher than that of their competitors who used more traditional contracts.

If, on the basis of the evidence above, it is argued that 1R1 contracts increase wor!er productivity, which of the following, if true, would most seriously wea!en that argument$ ,#- Results similar to those cited for the metalwor!ing industry have been found in other industries where 1R1 contracts are used. ,'- @nder 1R1 contracts costs other than labor costs, such as plant, machinery, and energy, ma!e up an increased proportion of the total cost of each unit of output. ,(- 'ecause introducing 1R1 contracts greatly changes individual wor!ers relationships to the firm, negotiating the introduction of 1R1 contracts is comple6 and time consuming. ,)- 3any firms in the metalwor!ing industry have modernized production equipment in the last five years, and most of these introduced 1R1 contracts at the same time. ,*- In firms in the metalwor!ing industry where 1R1 contracts are in place, the average ta!e0home pay is 1A percent higher than it is in those firms where wor!ers have more traditional contracts.

#nswer+ ) E planation+ Evidence 0 =irms with 1R1 pay wages that vary with profit 0 Last year, metal industry with 1R1 had 15T increase in productivity $onclusion 0 1R1 contracts increase wor!er productivity

%his is a Kwea!en the argumentL question type, so loo! for an incorrect assumption or a piece of contradictory evidence.

#nswers ' and ( are outside the scope of the conclusion, so they can be immediately eliminated. #nswer ' tal!s about the increased cost of each unit, which is not relevant to wor!er productivity. #nswer (, the fact that 1R1 contracts are time consuming and comple6 is not relevant to wor!er productivity as well. In solution #, similar results have been found. %his does not wea!en the argument, so eliminate "olution *, firms with 1R1 contracts have higher ta!e home pay than firms with traditional contracts. %his is not relevant to wor!er productivity, so eliminate. "olution ) introduces a new piece of evidence. It states that new equipment was introduced around the same time as 1R1 contracts. %his is contradictory evidence because it wea!ens the conclusion that 1R1 contracts are responsible for increased wor!er productivity. %he answer has to be ).

15D. $afeteria patron+ %he apples sold in this cafeteria are greasy. %he cashier told me that the apples are in that condition when they are delivered to the cafeteria and that the cafeteria does not wash the apples it sells. 3ost fruit is sprayed with dangerous pesticides before it is harvested, and is dangerous until it is washed. (learly, the cafeteria is selling pesticide0covered fruit thereby endangering its patrons.

/hich one of the following is an assumption on which the argument depends$ ,#- %he apples that the cafeteria sells are not thoroughly washed after harvest but before reaching the cafeteria ,'- 3ost pesticides that are sprayed on fruit before harvest leave a greasy residue on the fruit ,(- 3any of the cafeteria9s patrons are unaware that the cafeteria does not wash the apples it sells. ,)- :nly pesticides that leave a greasy residue on fruit can be washed off ,*- =ruits other than apples also arrive at the cafeteria in a greasy condition

#nswer+ # E planation+ $onclusion+ #pples are dangerous. Evidence+ #pples are not washed by the cafeteria.

#pples are dangerous until someone washes them. #ssumption+ No one *L"* washed the apples.

# restates this assumption clearly. %he word 8most8 is the reason ' is incorrect. /e don9t need to !now anything about 8most8 pesticides, we only need to !now about the one used on the apples. In (, we don9t care about what patrons are aware of& ) doesn9t matter, because we still don9t !now the greasy residue is pesticide& * tal!s about other fruit, so is wrong. %his problem is a rarity in <3#% terms& most <3#% arguments don9t include such a blatant red herring as the 8greasiness8 of the residue. >owever, the fact that there is grease is secondary and doesn9t play a direct role in the cafeteria patron9s chain of logic& we need to recognize that the evidence, and conclusion, is both about whether the apples have been washed, not about whether the apples are greasy.

15F. Environmental or!anizations want to preserve the land surroundin! the 0il!rinn 0ilderness #rea from residential development" %hey plan to do this by purchasing that land from the farmers who own it. %hat plan is ill0conceived+ if the farmers did sell their land, they would sell it to the highest bidder, and developers would outbid any other bidders. :n the other hand, these farmers will never actually sell any of the land, provided that farmin! it remains viable" 'ut farming will not remain viable if the farms are left unmodernized, and most of the farmers lac! the financial resources modernization requires. #nd that is e6actly why a more sensible preservation strategy would be to assist the farmers to modernize their farms to the e6tent needed to maintain viability.

In the argument as a whole, the two boldface proportions play which of the following roles$ #. %he first presents a goal that the argument re2ects as ill0conceived& the second is evidence that is presented as grounds for that re2ection. '. %he first presents a goal that the argument concludes cannot be attained& the second is a reason offered in support of that conclusion. (. %he first presents a goal that the argument concludes can be attained& the second is a 2udgment disputing that conclusion. ). %he first presents a goal, strategies for achieving which are being evaluated in the argument& the second is a 2udgment providing a basis for the argument9s advocacy of a particular strategy. *. %he first presents a goal that the argument endorses& the second presents a situation that the argument contends must be changed if that goal is to be met in the foreseeable future.

#nswer+ )

E planation+ *nvironmental organizations /#N% %: preserve the land. %herefore, the first sentence introduces the organizations goal. %he second sentence introduces their plan. %he third sentence tells us that the author thin!s their plan to achieve this goal is ill0conceived, and why he thin!s it is ill0conceived ,the farmers will 2ust sell the land to developers, and there goes preservation-. %he fourth sentence ,8on the other hand8-, is the beginning of the author s arguing towards a different plan. >e tells us that the farmers won t sell the land if it remains viable. %hat means if the lands are viable farmland, they won t fall into the hands of developers and they will be preserved. "o, he argues, a better preservation strategy would be to assist the farmers in !eeping the farmland viable. %hat way, the land will certainly not fall into the hands of developers. In bold face questions, ma!e sure you analyze the role of all the sentences, not 2ust the emboldened ones. ;ou need to get the gist of the argument, and the gist derives from consideration of the argument in its totality, and as a unified whole.

Let s now loo! at the choices+ #. %he author is arguing that the plan ,not the goal- is ill0conceived. Ni6, and don t even read the second clause. '. /e should e6it this choice at Kcannot be attainedL. If the author thought the goal could not be conceived, he would not bother advancing an alternate plan to meet the goal. (hoice ( can be eliminated for the same reason, and 2ust as quic!ly. ). %he second bold statement+ 8these farmers will never actually sell any of the land, provided that farming it remains viable.8 (an this bold statement be regarded as a K2udgmentL$ ;ep, because it is not a fact, it is a value 2udgment. 'ut because it is part of the author s evidence, we don t argue with it. /e shouldn t be evaluating the quality of the argument anyways in bold face questions. #nd, is the 2udgment a 8basis for the argument s advocacy of a particular strategy8$ ;ep. %he author uses this value 2udgment as evidence to support the plan he is arguing for+ assisting the farmers in !eeping ,andMor ma!ing- their farmlands viable ,so that the lands remain beyond the KclawsL of the developers, and so that the land is preserved.- #t this point, we would select choice ). 'ut let s loo! at *+ *. >e is not arguing that the farmlands should be made unviable. >e is arguing the opposite.

17G. *heater $ritic+ %he play La =inestrina, now at (entral %heater, was written in Italy in the eighteenth century. %he director claims that this production is as similar to the original production as is possible in a modern theater. #lthough the actor who plays >arlequin the clown gives a performance very reminiscent of the twentieth0century #merican comedian <roucho 3ar6, 3ar69s comic style was very much within the comic acting tradition that had begun in si6teenth0century Italy.

%he considerations given best serve as part of an argument that

,#- modern audiences would find it hard to tolerate certain characteristics of a historically accurate performance of an eighteenth0century play ,'- <roucho 3ar6 once performed the part of the character >arlequin in La =inestrina ,(- in the @nited "tates the training of actors in the twentieth century is based on principles that do not differ radically from those that underlay the training of actors in eighteenth0century Italy ,)- the performance of the actor who plays >arlequin in La =inestrina does not serve as evidence against the director9s claim ,*- the director of La =inestrina must have advised the actor who plays >arlequin to model his performance on comic performances of <roucho 3ar6

#nswer+ ) E planation+ K%he considerations given best serve as part of an argument thatL :!ay, so there are considerations given ,by the author- leading up to an argument. %he Kconsiderations givenL is evidence, and the question wants us to determine the author s main point, or conclusion. /e call this type of question a Kmain pointL question. +tep two is to analyze the stimulus+ Normally, we identify a conclusion by loo!ing for words li!e Kthus, therefore, hence, etc.L '@%, in a main point question, we actually don t want to be attracted by those !inds of words$8%hey are put there by the test0ma!er because they !now you will naturally be attracted to them in a main point question. Instead, in a main point question, the author s main point will often be signaled by a contrast !eyword such as Kbut,L Khowever,L Kwhile,L or, Kalthough.L

In the passage of this question, we learn of the director s claim in the second sentence. %he director s claim is that their reproduction of the play is a lot li!e the original play. %he ne6t sentence starts with the word Kalthough.L >ere, the function of KalthoughL is to dismiss potential counter0evidence against the director s claim+ the fact that the actor is li!e .Gth century <roucho 3ar6 could be used against the director s claim ,that the reproduction is a faithful representation of the original 1Bth century version."o from the word Kalthough,L and using some critical reasoning, we can determine that the author s intent in arguing is to defend the director s claim against evidence that could go against it ,against the director s claim-. +tep three of the method is to ma!e a prediction of the right answer. "ay to you+ K%he author is defending the director. %he author s main point is that the director s claim is correct.L +tep four is to aggressively scan for a match to the prediction. 'ecause we spent so much time generating the prediction, and because we don t care about wrong answers and why they are wrong, we scan for a choice that matches our insight. %hen, choice ) is correct.

171.%he fishing industry cannot currently be relied upon to help the government count the seabirds !illed by net fishing, since an accurate count might result in restriction of net fishing. %he government should therefore institute a program under which tissue samples from the dead birds are e6amined to determine the amount of to6ins in the fish eaten by the birds. %he industry would then have a reason to turn in the bird carcasses, since the industry needs to !now whether the fish it catches are contaminated with to6ins.

/hich one of the following, if true, most strongly indicates that the government program would not by itself provide an accurate count of the seabirds !illed by net fishing$ ,#- %he seabirds !illed by net fishing might be contaminated with several different to6ins even if the birds eat only one !ind of fish ,'- %he fishing industry could learn whether the fish it catches are contaminated with to6ins if only a few of the seabirds !illed by the nets were e6amined ,(- %he government could gain valuable information about the source of to6ins by e6amining tissue samples of the seabirds caught in the nets. ,)- %he fish caught in a particular net might be contaminated with the same to6ins as those in the seabirds caught in that net. ,*- %he government would be willing to certify that the fish caught by the industry are not contaminated with to6ins if tests done on the seabirds showed no contamination

#nswer+ ' E planation+ >ere, we have a proposal00which, per the Oaplan method, we paraphrase+ 8/e will get an accurate count of seabirds by testing them for to6ins.8 :n the <3#%, predictions and proposals assume that they are viable on their own terms& we aren9t loo!ing for unrelated evidence that might or might not affect the conclusion, but rather for something that e6plains why testing birds for to6ins will not lead to an accurate count. ' says that testing birds for to6icity can be accomplished with only a few seabirds& in other words, there is no reason for testing to give an accurate count. It is our answer

17..@nless tiger hunting decreases, tigers will soon be e6tinct in the wild. %he countries in which the tiger9s habitats are located are currently debating 2oint legislation that would ban tiger hunting. %hus, if these countries can successfully enforce this legislation, the survival of tigers in the wild will be ensured.

%he reasoning in the argument is most vulnerable to criticism on the grouns that the argument #. assumes without sufficient warrant that a ban on tiger hunting could be successfully enforced. '. considers the effects of hunting on tigers without also considering the effects of hunting on other endangered animal species.

(. fails to ta!e into account how often tiger hunters are unsuccessful in their attempts to !ill tigers. ). neglects to consider the results of governmental attempts in the past to limit tiger hunting. *. ta!es the removal of an impediment to the tiger9s survival as a guarantee of their survival.

#nswer+ * E planation+ %he author]s first sentence is evidence that establishes that if hunting does not decrease, the tiger will become e6tinct in the wild. %hat means if steps are not ta!en to prevent the hunting, there will no longer be #N; tigers in the wild. %his is evidence, and we 8give8 the author his evidence& it is true information. In the second sentence, we learn that there is legislation on the table, which, if passed, would ban tiger0 hunting. 'ecause tiger0hunting will definitely !ill off all of the wild tigers, legislation ,or some other thing- is necessary for the tiger]s survival in the wild. %his does not mean that the legislation will guarantee ,ie, is sufficient for- the tiger]s survival.

Let]s thin! of an analogy. <as is needed for driving a car. 'ut having gas in the car]s tan! does not guarantee that you will be able to drive the car because there are other things you need to drive a car+ transmission, steering wheel, etc. In other words, gas is necessary but not sufficient for driving a car. %he author]s evidence establishes that legislation ,or some other thing- is necessary ,needed- for the tiger]s survival in the wild. 'ut the legislation does not necessarily guarantee the tiger]s survival because there may be other needed things+ a stiff penalty for breaching the tiger0hunting ban, resources to implement the ban, etc. /ithout the legislation aimed at stopping tiger0hunting ,or something else that stops tiger0hunting-, the wild tiger are doomed. %he passing of the legislation gives the tiger a chance of surviving ,i.e., there e6tinction is no longer guaranteed- but does not guarantee that they will survive. In other words, the argument is vulnerable to criticism on the grounds that it 8ta!es the removal of an impediment to a tiger9s survival as a guarantee of their survival.8 ,(hoice *-

175.*missions from automobiles that burn gasoline and automobiles that burn diesel fuel are threatening the quality of life on our planet, contaminating both urban air and global atmosphere. %herefore, the only effective way to reduce such emissions is to replace the conventional diesel fuel and gasoline used in automobiles with cleaner0burning fuels, such as methanol, that create fewer emissions.

/hich one of the following is an assumption on which the argument depends$

,#- Reducing the use of automobiles would not be a more effective means to reduce automobile emissions than the use of methanol. ,'- %here is no fuel other than methanol that is cleaner0burning than both diesel fuel and gasoline. ,(- If given a choice of automobile fuels, automobile owners would not select gasoline over methanol. ,)- #utomobile emissions constitute the most serious threat to the global environment. ,*- #t any given time there is a direct correlation between the level of urban air pollution and the level of contamination present in the global atmosphere.

#nswer+ # E planation+ "ince author believes that only way to reduce pollution is change of fuel, author must have ruled out other alternate mechanism, which may reduce pollution, such as less automobile.

177.)enoma, a ma2or consumer0electronics ma!er, had a sizeable decline in sales revenue for its most recent fiscal year. %his result appears surprising, because electronics retailers report that although their overall sales were considerably lower than in the previous year, their sales revenue from )enoma models actually grew, largely than!s to some innovative and popular models that )enoma introduced.

/hich of the following, if true, does most to e6plain the apparently surprising result$ #. 'ecause of the need to educate the public about its new models9 capabilities, )enoma9s advertising spending was higher than normal over the period. '. =or the period at issue, )enoma9s ma2or competitors reported declines in revenue that were, in percentage terms, greater than )enoma9s. (. # significant proportion of )enoma9s revenue comes from ma!ing components for other consumer0 electronics manufacturers. ). @nli!e some of its ma2or competitors, )enoma has no lines of business outside consumer electronics to provide revenue when retail sales of consumer electronics are wea!. *. )uring the period, consumer0electronics retailers sold remaining units of )enoma9s superseded models at prices that were deeply discounted from those models9 original prices.

#nswer+ ( E planation+ In a parado6 question, you should have the answers to two questions very clear in your head before you go to the answer choices+

1- /hat is the parado6 ,i.e., surprise-$ .- /hy is it parado6ical ,i.e., surprising-$


Remember, a parado6 is 2ust+ something une6pected happened ,surpriseS- #lways search for a !eyword that will help you find the surprise, usually+ but, yet, although, or in this case surprising

%he surprise is+ )enoma s sales fell '@% sales from their )enoma cars increased It is surprising because+ if their sales fell, you would e6pect the opposite.

Now, when we go to the answer choices, we only as! one question to ourselves+ /ould the surprise ma!e sense$ Onow that many wrong answers will be outside the scope or else be 1DGs ,they will actually ma!e the surprise more surprising not less-.

"canning for a choice that will be logically capable of resolving the surprise+ #. 'ecause of the need to educate the public about its new models9 capabilities, )enoma9s advertising spending was higher than normal over the period. >igh ad spending would not e6plain why sales went down. (hoice ' is clearly outside the scope. (. # significant proportion of )enoma9s revenue comes from ma!ing components for other consumer0 electronics manufacturers. "o a huge chun! of their sales comes from components to other manufacturers rather than selling cars directly to people. "o even though they are selling a lot of cars to people, this fact introduces a salient e6planation for why their overall sales went down at the same time+ the company s bread and butter is selling parts not cars. /hat if parts sales went way down$ %hat would definitely be capable of resolving our surprise. (hoose ( is correct.

17A.Nohn wor!s five days each wee! e6cept when on vacation or during wee!s in which national holidays occur. =our days a wee! he wor!s in an insurance company& on =ridays he wor!s as a blac!smith. Last wee! there were no holidays, and Nohn was not on vacation. %herefore, he must have wor!ed in the insurance company on 3onday, %uesday, /ednesday, and %hursday last wee!. /hich one of the following is an assumption on which the argument depends$ #. Nohn never ta!es a vacation of more than one wee! in length. '. *very day last wee! that Nohn wor!ed, he wor!ed for an entire wor!day. (. Nohn does not ta!e vacations in wee!s in which national holidays occur. ). Last wee! Nohn wor!ed neither on "aturday nor on "unday. *. %here were no days last wee! on which Nohn both wor!ed in the insurance company and also wor!ed as a blac!smith.

#nswer+ ) E planation+ %he author is concluding that Nohn 3@"% have wor!ed at the insurance company on 3on, %ues, /ed, and %hurs of last wee!...but the evidence never did establish that Nohn only wor!s on wee!days. =ine he wor!s four days at the insurance company, but those days could have included "aturday and "unday. (hoice ) matches this. /e can also use the denial test on choice ) to chec! whether it is correct+ what if Nohn )I) wor! on either of "aturday or "unday$ %hen the author9s argument00that Nohn 3@"% have wor!ed on 3onM%uesM/edM%hurs falls apart. 'ecause, in the absence of choice ), the argument falls apart, choice ) is a necessary assumption for the argument to hold.

17B.#lthough fullerenes P spherical molecules made entirely of carbon P were first found in the laboratory, they have since been found in nature, formed in fissures of the rare mineral shungite. "ince laboratory synthesis of fullerenes requires distinctive conditions of temperature and pressure, this discovery should give geologists a test case for evaluating hypotheses about the state of the *arth s crust at the time these naturally occurring fullerenes were formed.

/hich of the following, if true, most seriously undermines the argument$

A. (onfirming that the shungite genuinely contained fullerenes too! careful e6perimentation. B. "ome fullerenes have also been found on the remains of a small meteorite that collided with a
spacecraft.

C. %he mineral shungite itself contains large amounts of carbon, from which the fullerenes apparently
formed.

D. %he naturally occurring fullerenes are arranged in a previously un!nown crystalline structure. E. "hungite itself is formed only under distinctive conditions.

#nswer+ ) E planation+ %he author9s evidence has established that the labaratory conditions leading to fullerenes are unique. >e also tells us that we have now found fullerenes in nature ,shungite stuff is red herring-. >e is arguing that because lab0fullerene0formation requires unique conditions, we can e6amine these new natural fullerenes to figure out what the conditions in the *arth9s crust were li!e bac! when these fullerenes were formed. >e is assuming that fullerenes everywhere ,i.e., the naturally occurring ones, outside of the laboratory- also come about through unique conditions. ,It might 2ust be the case that it is only in the laboratory that unique conditions are required or that they have yet to hit upon other ways of ma!ing fullerenes in the lab-.

/e can use the Oaplan denial test to verify whether we have figured out the assumption correctly+ what if, outside of the lab, there are many ways that fullerenes can be formed$ %hen, what happens to the argument that we can e6amine these naturally occurring fullerenes to figure out what the conditions were li!e when they were formed$ /ell because there are many ways they could have formed, and who !nows what the conditions were li!e bac! then+ the argument now falls apart. %herefore, we have properly identified a necessary assumption.

'ecause this is a wea!en question, we loo! for a choice that attac!s this assumption. (hoice ) opens up the possibility of fullerenes forming in ways un!nown in the lab, and is correct. In a harder argument, it is very important to resist the temptation to go to the answer choices prematurely. <ot to analyze the stimulus and figure out how, precisely, the evidence is not good enough to establish the conclusion. <ot to figure out that assumptionS 3a!e sure you always as! what idea is in the conclusion that is not in the evidence and what idea is in the evidence that is not in the conclusion, and then bridge the gap. %his is the classic Oaplan method for arguments. #pplied here+ /hat idea is in the evidence that is not in the conclusion$ 8Lab fullerenes8 #nd the idea in the conclusion that is not in the evidence$ 8Naturally occurring fullerenes8. %hen bridge the gap+ 8%he author is assuming some sort of similarity between the two. %he argument is about unique conditions, so he is assuming that conditions are unique for both lab and natural fullerenes.8

17C.It might seem that an airline could increase profits by reducing airfares on all its flights in order to encourage discretionary travel and thus fill planes. :ffers of across0the0board discount fares have, indeed, resulted in the sale of large numbers of reduced0price tic!ets. Nevertheless such offers have, in the past, actually cut the airline s profits.

/hich one of the following, if true, most helps to resolve the apparent discrepancy described above$ ,#- =ewer than 1G percent of all air travelers ma!e no attempt to see! out discount fares. ,'- =ares for trips between a large city and a small city are higher than those for trips between two large cities even when the distances involved are the same. ,(- #cross0the0board discounts in fares tend to decrease revenues on flights that are normally filled, but they fail to attract passengers to unpopular flights. ,)- :nly a small number of people who have never before traveled by air are persuaded to do so on the basis of across0the0board discount fares. ,*- It is difficult to devise an advertising campaign that ma!es the public aware of across0the0board discount fares while fully e6plaining the restrictions applied to those discount fares.

#nswer+ ( E planation+ In parado6 questions we don t predicting something specific. Instead, we predict that the right answer will resolve the surprise. In a parado6 question, you should have the answers to two questions very clear in your head before you go to the answer choices+ 1- /hat is the parado6 ,ie, surprise-$ .- /hy is it parado6ical ,ie, surprising-$ >ere, the surprise is that some airlines profits actually get hurt when they cut tic!et prices. #nd, it is surprising because, ordinarily, airlines profits are boosted when they cut tic!et prices.

Now, we go through the choices searching for something that will be logically capable of resolving this surprise ,"tep 7 is match-. /e are loo!ing for a fact that will ma!e everything in the passage ma!e sense. "ay to you+ KI want an e6planationSL Let s do this+ ,#- )oesn t matter. Instead of resolving the parado6, this choice casts doubt on the idea that cutting prices will boost profits in general. 'ut we were told in the passage that, ordinarily, cutting prices will boost profits. In a parado6 question ,li!e in an inference- question, everything in the passage must be true. ,'- %his is clearly outside of the scope. In order to even thin! this choice was relevant we would have to ma!e a bunch of assumptions. ,(- #cross0the0board discounts in fares tend to decrease revenues on flights that are normally filled, but they fail to attract passengers to unpopular flights. "o, a certain !ind of price0cutting off the board price cutting can actually hurt profits. #cross the board price cutting will fail to attract customers. %his is capable of e6plaining our parado6 ,that sometimes price0cutting hurts profits-, and we are done. ,/e don t now stop and say+ oh but how do I !now that the airlines whose profits were hurt in the passage are doing across0the0board cutting.- %he right answer 2ust has to be something that is logically capable of resolving the parado6. 17D.%he growing popularity of computer0based activities was widely e6pected to result in a decline in television viewing, since it had been assumed that people lac! sufficient free time to maintain current television0 viewing levels while spending increasing amounts of free time on the computer. %hat assumption, however, is evidently false+ in a recent mail survey concerning media use, a very large ma2ority of respondents who report increasing time spent per wee! using computers report no charge in time spent watching television. /hich of the following would it be most useful to determine in order to evaluate the argument$ #. /hether a large ma2ority of the survey respondents reported watching television regularly '. /hether the amount of time spent watching television is declining among people who report that they rarely or never use computers (. /hether the type of television programs a person watches tends to change as the amount of time spent per wee! using computers increases ). /hether a large ma2ority of the computer owners in the survey reported spending increasing amounts of time per wee! using computers *. /hether the survey respondents9 reports of time spent using computers included time spent using computers at wor!

#nswer+ * E planation+ 'asically you have two types of computer usage+ 0 at wor! and does not affect free time 0 at home and does affect free time, competing with other pass time activities ,i.e. %H watching-.

%he reason why this distinction is important is because we mustn9t compare apples to oranges. %he only lin! between computer usage and %H watching refers to their 8competition8 for the user9s free time. No matter how much you9d li!e to watch %H at wor!, that won9t be possible, since you are supposed to be doing your 2ob and you have to use the computer. ;ou ):N9% have a choice. /hile you9re at home, you can either watch %H or play (". ;ou ): have a choice. %here is a crucial difference here. 81opularity8 is strictly related to choice+ something is popular because many people choose it.

17F.In their native habit, amaryllis plants go dormant when the soil in which they are growing dries out during the dry season. %herefore, if amaryllis plants !ept as houseplants are to thrive, water should be withheld from them during part of the year so that the plants go dormant.

/hich one of the following is an assumption on which the argument depends$ ,#- 3ost !inds of plants go dormant at some time or other during the year. ,'- #maryllis are more difficult !eep as houseplants than other !inds of plants are. ,(- /ater should be withheld from amaryllis plants !ept as houseplants during the e6act time of year that corresponds to the dry season in their native habitat ,)- #ny amaryllis plant that fails to thrive is li!ely to have been dormant for too short a time. ,*- <oing dormant benefits amaryllis plants in their native habitat in some way other than simply preventing death during overly dry periods

#nswer+ * E planation+ Nust say to yourself+ 8the evidence established that they go dormant in dry season.8 'ut the conclusion is+ 8!eep them dry to induce dormancy so that they thrive in the home.8 %hen analyze+ 8well in nature they will get dry for sure and that9s why they go dormant ,to protect against dryness-. 'ut he thin!s ma!ing them go dormant is a good idea for the home too....he must be assuming that inducing dormancy will help in some way other than protecting against dryness.8 Now we scan for a match, which is choice * If you narrowed it down to ) and *, you could have eliminated ) on the basis of it being too e6treme 8any8. In assumption questions, the argument is unli!ely to be so bold as to rely on an e6treme assumption. Is this argument so bold as to )*1*N) on assuming that #ll plants that fail to thrive do so because of not being dormant long enough$ #nd, of course, you can also use denial test on *. 1AG.3ost people believe that yawning is most powerfully triggered by seeing someone else yawn. %his belief about yawning is widespread not only today, but also has been commonplace in many parts of the world in the past, if we are to believe historians of popular culture. %hus, seeing someone else yawn must be the most irresistible cause of yawning.

%he argument is most vulnerable to which one of the following criticisms$ ,#- It attempts to support its conclusion solely by restating that conclusion in other words. ,'- It cites the evidence of historians of popular culture in direct support of a claim that lies outside their area of e6pertise. ,(- It ma!es a sweeping generalization about yawning based on evidence drawn from a limited number of atypical cases. ,)- It supports its conclusion by appealing solely to opinion in a matter that is largely factual. ,*- It ta!es for granted that yawns have no cause other than the one it cites.

#nswer+ ) E planation+ %he truth value of E doesn9t depend upon the truth value of 8I believe that E8. %hat s why 8that8 is called intentional predicate.

1A1.# recent research study of undergraduate students analyzed the effects of music on human emotions. *ach of the .GG participants attended at least 1 two0hour concert of classical music per wee! over the course of 1. wee!s of their spring semester. #t the end of the e6periment, all of the students filled out a questionnaire assessing their emotional state. 'ased on the results of the questionnaires, all of the 1G students who attended the greatest number of concerts reported lower stress levels and higher satisfaction with their lives. #lso, most of the .G students who attended the fewest number of concerts reported below0average levels of emotional comfort. /hich of the following must be true based on the evidence presented above$ a. 3ost of the .GG participants improved their emotional state and lowered their stress levels. b. )uring each wee! of the e6periment, the participants spent at least . hours less on their academic wor! as a result of concert attendance. c. Listening to classical music for at least . hours per wee! improves the emotional well0being of the ma2ority of young adults. d. 3ore than B participants attended at least 17 concerts during the course of the e6periment. e. #t least some of the students participated in the study in order to gain free access to classical concerts.

#nswer+ ) E planation+ In an inference question, we need to find a choice that must be true based on one or more of the statements in the passage. %he four wrong answers are things that either could or must be false. /hen reading the passage of an inference question, determine everything that must be true but also what is not necessarily true. >ere, the students are reporting how they feel....'@% this is not necessarily the same as what they actually feel ,students can under0 or over0report how they feel, or they can lie, etc-. %herefore, based on this study we actually don t !now anything about the lin! between listening to music and emotion or stress...therefore, choices # and ( could be false. (hoice * obviously could be false ,we have no idea what the students motivation was for participating in the study 0 maybe the study was mandatory for them-. (hoice ' could be false. =ine, because of going to the concert they have two hours removed from their schedule. 'ut they can ma!e up the two hours for studying elsewhere. (hoice ) must be true on the numbers. /e !now that all .GG attended 8at least one8 concert per wee!. %hen, the stimulus tells us that there are .G participants who attended the 8fewest number of concerts8. "o, maybe they attended 2ust once per wee! for a total of 1.. #nd, there are 1G who attended the 8greatest number of concerts8. 'ut that leaves .GG0,.GW1G- I 1CG participants who attended some number of concerts that was intermediate between the KfewestL and the KgreatestL. If we minimize the number of concerts attended by this intermediate group, then they had have to attended at least 15 total concerts ,to be more than the KfewestL-. %hat means the group of ten who attended the greatest number had to have attended at least 17, and therefore choice ) must be true.

1A.. 'f interest rates remain at their current hi!h levels, many people who currently rent their residences will hesitate to purchase homes" #s the price of real estate continues to climb, the costs of a mortgage will be too burdensome. "ellers will be forced to lower their as!ing prices. "o, hi!h interest rates will eventually cause the real estate market to stabilize" In the argument above, the portions in boldface play which of the following roles$ #. %he first is the author9s main point& the second is a prediction that follows from that point. '. %he first is a consideration that the author believes will result in a certain situation& the second is that situation. (. %he first is a consideration that weighs against the author9s main point& the second is the author9s main

point. ). %he first is a prediction& the second is evidence in support of that prediction. *. %he first is the author9s main point& the second is evidence used to argue against that point.

#nswer+ ' E planation+ 'oldface questions are pretty straightforward. the answer choices are 2ust descriptions of what is going on in the stimulus. %he e6plicit parts of a stimulus are either premise or conclusion. %he answer choices will use different words to throw you off, li!e 8consideration8 I premise or 8main point8 I conclusion. 'ut stay focused and you can get the answer quic!ly. =or this one, 8if then8 statements are usually conditional statements that are premises. and 8so8 is a conclusion indicator. "o I m loo!ing for answers that say that. # and * 0 eliminate...first bold is not author9s main point. ( 0 *liminate...stay away from 8weighs against8 ) 0 *liminate...first bold is not prediction. It s a conditional statement used as evidence for the author s main point. ' 0 # consideration the author believes will result in a certain situation sounds li!e 8a conditional statement8. "econd bold gives it away with word 8eventually8 i.e. based on condition author sets up this situation will follow.

1A5.=armers in developing countries claim that the @nited "tates government, through farm subsidies, is responsible for the artificially low global price of wheat. 'ecause the @.". government buys whatever wheat #merican farmers are unable to sell on the open mar!et, #merican farmers have no incentive to modulate the size of their crops according to the needs of the global mar!et. #s a result, #merican farmers routinely produce more wheat than the global mar!et can absorb and the global price of wheat is !ept low. /ithout these subsidies, the farmers in developing economies claim, #merican farmers would produce only the amount of wheat that they could sell on the open mar!et and the global price of wheat would rise. /hich of the following, if true, most wea!ens the claims of the farmers in developing countries regarding the price of wheat$ #. /heat that is not processed for consumption is often used for certain industrial applications. '. Non0governmental buyers of wheat and wheat products are able to predict how much wheat they will need several years in advance. (. the @nited "tates government offers similar subsidies to soybean farmers, though the global price of soybeans is significantly higher than that of wheat. ). :ther countries, such as (anada and Russia, are li!ely to produce more wheat if the @nited "tates were to reduce its output. *. %he price of sorghum, a crop for which the @nited "tates government offers no subsidies, is lower than that of wheat.

#nswer+ ) E planation+ @" subsidies 00J increased supply 00J price drops. @" subsidies are causing more wheat prod which in turn low prices of wheat i.e. @" sub 0J Low prices.

%he author believes that only @" sub are responsible for low prices and nothing else. In ,d- author is giving alternate reason for low prices i.e. e6tra production by (anada and Russia. "ince there is an alternate way for reducing prices, this answer ,d- wea!ens the author s assumption and hence is the correct answer.

1A7.(ommensalism is any relationship between two living things in which one benefits and the other is neither helped nor harmed. :6pec!er birds are commensal species that floc! with the large mammals of the #frican "avannah. %hey feed on tic!s, fleas, and flies that are attracted to the mammals9 fur.

/hich of the following, if true, can most reasonably be inferred from the statements above$ ,#- :6pec!er birds are neither helped nor harmed by the large mammals of the #frican "avannah. ,'- %ic!s, fleas, and flies are commensal species in their relationship with both o6pec!er birds and the large mammals of the #frican "avannah. ,(- No species e6ist in a commensal relationship with o6pec!er birds e6cept for large mammals of the #frican "avannah. ,)- In commensal relationships, the smaller of the species in the relationship usually benefits while the larger is neither helped nor harmed. ,*- 1reying on small creatures drawn to the fur of the large mammals of the #frican "avannah does not significantly affect those mammals.

#nswer+ * E planation+ %his is an inference question. %he passage suggests that, since o6pec!er birds are commensal species with large mammals, they benefit from the creatures that are attracted to the mammals9 fur, but the mammals themselves are neither helped nor harmed by the relationship. (onsider each choice, loo!ing for a reasonable inference+ ,#- %his choice gets the commensal relationship e6actly bac!wards. ,'- %his is clearly wrong. If o6pec!er birds feed on tic!s, fleas, and flies, clearly the tic!s, fleas, and flies are neither benefiting nor neutral in their relationship with the o6pec!er birds. ,(- %his choice is too e6treme. %he passage only describes this relationship& it doesn9t tell us that it is e6clusive. ,)- %his is also too e6treme. It may be true in some instances, but the three sentences of the passage don9t provide enough evidence to reasonably deduce this. ,*- %his is correct. It merely restates the definition of commensalism in terms of the role of the mammals in their relationship with o6pec!er birds. 1AA. 'f interest rates remain at their current hi!h levels, many people who currently rent their residences will hesitate to purchase homes" #s the price of real estate continues to climb, the costs of a mortgage will be too burdensome. "ellers will be forced to lower their as!ing prices. "o, hi!h interest rates will eventually cause the real estate market to stabilize" In the argument above, the portions in boldface play which of the following roles$ #. %he first is the author9s main point& the second is a prediction that follows from that point. '. %he first is a consideration that the author believes will result in a certain situation& the second is that

situation. (. %he first is a consideration that weighs against the author9s main point& the second is the author9s main point. ). %he first is a prediction& the second is evidence in support of that prediction. *. %he first is the author9s main point& the second is evidence used to argue against that point. #nswer+ ' E planation+ 'oldface questions are pretty straightforward. %he answer choices are 2ust descriptions of what is going on in the stimulus. %he e6plicit parts of a stimulus are either premise or conclusion. %he answer choices will use different words to throw you off, li!e 8consideration8 I premise or 8main point8 I conclusion. 'ut stay focused and you can get the answer quic!ly. =or this one, 8if then8 statements are usually conditional statements that are premises. #nd 8so8 is a conclusion indicator. "o I am loo!ing for answers that say that. # and * 0 eliminate...first bold is not author9s main point. ( 0 eliminate...stay away from 8weighs against8 ) 0 eliminate...first bold is not prediction. Its a conditional statement used as evidence for the authors main point. ' 0 a consideration the author believes will result in a certain situation sounds li!e 8a conditional statement8. "econd bold gives it away with word 8eventually8 i.e. based on condition author sets up this situation will follow.

1AB.

#nswer+ # E planation+ %he second sentence reads+ "ome commentators have argued, correctly, that since %>*R* I" 1R*"*N%L; N: :'N*(%IH* %*"% =:R />I1L#">, spurious reports of whiplash cannot be readily identified. %he 8since8 tells us that there being no test for whiplash is the commentators9 evidence of some !ind. %he 8correctly8 tells us the author does not dispute this evidence. In the ne6t sentence, however, we get the important contrast !eyword 8however8 ,no pun, promiseS-. (ontrast !eywords often usher in the author9s point of view in boldface questions. %he author is chiming in saying+ yeah fine, they are right about no test for whiplash but they are wrong to @"* that fact to support this idea of theirs ,i.e., their conclusion that people are fa!ing whiplash in the countries whose insurance schemes compensate whiplash victims.%he author uses the phrase 8further conclusion8 in the third sentence. %his tells us that the commentators are using the argument in the first bold statement00the one the author does agree with00to support an

argument that the author does not agree with ,they 8are, however, wrong8-. %he first clause of choice # correctly identifies the first bold statement s function as the 8commentators98 evidence. %he second clause of choice # properly pic!s up on the huge 8however8 !eyword, telling us that the author is out to dispute these commentators9 main ,further- conclusion.

1AC.(ertain instruments used in veterinary surgery can be made either of stainless steel or of nylon. In a study of such instruments, 1G complete sterilizations of a set of nylon instruments required 5.7 times the amount of energy used to manufacture that set of instruments, whereas AG complete sterilizations of a set of stainless steel instruments required ..1 times the amount of energy required to manufacture that set of instruments.

If the statements above are true, each of the following could be true *E(*1%+ ,#- %he AG complete sterilizations of nylon instruments used more energy than did the AG complete sterilizations of the stainless steel instruments. ,'- 3ore energy was required for each complete sterilization of the nylon instruments than was required to manufacture the nylon instruments. ,(- 3ore nylon instruments than stainless steel instruments were sterilized in the study. ,)- 3ore energy was used to produce the stainless steel instruments than was used to produce the nylon instruments. ,*- %he total cost of AG complete sterilizations of the stainless steel instruments was greater than the cost of manufacturing the stainless steel instruments. #nswer+ ' E planation+ %he passage tells us that the ratio of energy required for sterilization to energy required for manufacture is greater for nylon instruments than it is for stainless steel instruments. 'ut we have no idea how much energy it ta!es to manufacture either of these. =or e6ample, it might ta!e 1 unit of energy to manufacture a set of nylon instruments and 1GGG units of energy to manufacture a set of stainless steel instruments. In that case, we would clearly need more energy to sterilize stainless steel than we would to sterilize nylon. #nd it could also be the other way around...."o, both choices # and ) could be true. (hoice ( could be true because a 8set8 of nylon instruments could be AGG nylon instruments while a 8set8 of stainless steel could be A instruments. (hoice * could be true because we have no info about cost. (hoice ' cannot be true on the numbers. # sterilization of a set of nylon instruments only requires 57T of the energy required to manufacture that set. "o, when sterilizing a set of nylon instruments, we will definitely need L*"" energy than we did to manufacture that set.

1AD.Radio stations with radio data system ,R)"- technology broadcast special program information that only radios with an R)" feature can receive. 'etween 1FF7 and 1FFB, the number of R)" radio stations in Herdland increased from .AG to BGG. >owever, since the number of R)"0equipped radios in Herdland was about the same in 1FFB as in 1FF7, the number of Herdlanders received the special program information probably did not increase significantly. /hich of the following is an assumption on which the argument depends$ ,#- =ew if any of the R)" radio stations that began broadcasting in Herdland after 1FF7 broadcast to people with R)"0equipped radios living in areas not previously reached by R)" stations.

,'- In 1FFB most Herdlanders who lived within the listening area of a R)" station already had a radio equipped to receive R)". ,(- *quipping a radio station with R)" technology does not decrease the station9s listening area. ,)- In 1FFB Herdlanders who did not own radios equipped to receive R)" could not receive any programming from the R)" radio stations that began broadcasting in Herdland after 1FF7. ,*- %he R)" radio stations in Herdland in 1FFB did not all offer the same type of programming.

#nswer+ # E planation+ %he number of R)"0equipped radios did not increase. >e thin!s this means that the number of Herdlanders receiving the special program also has not increased. 'ut he is assuming that #LL of the radios were always receiving radio signals. 'ut 2ust because they are the 8only8 radios capable of receiving signals does not mean that all of them were receiving signals ,bac! in 1FF7-. )*NI#L %*"%+ if some of the radios were N:% receiving signals in 1FF7, but after the increase of radio stations sending these signals, #R* receiving signals in 1FFB, then the argument falls apart. .or e ample+ It could be that in 1FF7 all .AG of the radio stations were congregated in 2ust a couple of cities on the west coast of Herdland. 'ut some of the people who own the receivers could have been on the *ast (oast, and so, were too far away to receive signals. %hen, in 1FFB there are BGG of these radio stations. Now, there are also radio stations on the *ast (oast receiving signals. Now, a greater number of Herdlanders are receiving signals, and the argument falls apart. +trate!ically+ because the conclusion is tal!ing about 8not increase8 the scope of the argument is whether or not there was an increase. #ccordingly, we should be partial to answer choices that bring up the idea of difference and number. 1AF. %olitical #nalyst + 'ecause our city is a border city, illegal immigration is an important issue in the current race for mayor. :f the two candidates for mayor, one supports a plan that would attempt to deport the city s F,GGG illegal immigrants and the other does not. "urveys consistently show that about BGT of the city s residents are opposed to the plan, while about 5AT are in support of the plan. %herefore, the candidate who does not support the plan will win the election for mayor. #ll of the following statements wea!en the analyst s argument, *E(*1%+ #0 In the city at issue, most voters ma!e their voting decisions based on the candidates positions on abortion. '0 :f the 5AT of residents who support the plan, some are willing to consider alternate plans for addressing illegal immigration. (0 3any of the residents who oppose the plan are not registered voters. )0 %he candidate who supports the plan is the incumbent mayor, and has been elected to four consecutive terms despite ta!ing controversial positions on many important issues. *0 Nust under 5GT of the city s residents are illegal immigrants who cannot vote.

#nswer+ # E planation+ %he analyst argues that the mayoral candidate who opposes the deportation plan will win the governor s race because BGT of city residents also oppose the plan. %he analyst assumes that a ma2ority of residents will vote for this candidate based on his position on illegal immigration. #ny statement that calls this assumption into question will wea!en the argument. /e are loo!ing for the one statement that does N:% call this assumption into question.

,#- %his statement calls into question the assumption that voters will cast their ballots based on the illegal immigration issue. %herefore, this statement wea!ens the analyst9s argument. ,'- (:RR*(%. %his does not wea!en the argument. In fact, if some of those who support the plan are willing to reconsider, they may ultimately oppose the original plan and decide to vote for the candidate who is also in opposition. If anything, this would help 2ustify the analyst9s claim that the candidate who opposes the plan will win the election. ,(- %his statement calls into question the assumption that a ma2ority of residents will vote for the candidate who opposes the plan. If many of these residents are not registered voters, they will not be able to vote, regardless of their position on the immigration issue. %his wea!ens the argument. ,)- %his calls into question the assumption that the residents will vote based on the illegal immigration issue. %his statement shows that voters have a history of voting for the incumbent despite his controversial position on important issues. It is possible that the voters will again vote for the incumbent, even if he has ta!en an unpopular position on the illegal immigration issue. %his wea!ens the argument. ,*- If 2ust under 5GT of the residents are illegal immigrants, it is li!ely that many of the BGT in opposition to the plan are actually illegal immigrants themselves. If these people can t vote, it is less li!ely that the candidate who opposes the plan will win.

1BG.Impact craters caused by meteorites smashing into *arth have been found all around the globe, but they have been found in the greatest density in geologically stable regions. %his relatively greater abundance of securely identified crater in geologically stable regions must be e6plained by the lower rates of destructive geophysical processes in those regions.

%he conclusion is properly drawn if which one of the following is assumed$ ,#- # meteorite that stri!es e6actly the same spot as an earlier meteorite will obliterate all traces of the earlier impact. ,'- Rates of destructive geophysical processes within any given region vary mar!edly throughout geological time. ,(- %he rate at which the *arth is struc! by meteorites has greatly increased in geologically recent times. ,)- #ctual meteorite impacts have been scattered fairly evenly over the *arth s surface in the course of *arth s geological history. ,*- %he *arth s geologically stable regions have been studied more intensively by geologists than have its less stable regions. #nswer+ )

1B1.# certain airport security scanner designed to detect e6plosives in luggage will alert the scanner s operator whenever the piece of luggage passing under the scanner contains an e6plosive. %he scanner will erroneously alert the operator for only one percent of the pieces of luggage that contain no e6plosives. %hus in ninety0nine out of a hundred alerts e6plosives will actually be present. %he reasoning in the argument is flawed because the argument ,#- ignores the possibility of the scanner s failing to signal an alert when the luggage does contain an e6plosive ,'- draws a general conclusion about reliability on the basis of a sample that is li!ely to be biased ,(- ignores the possibility of human error on the part of the scanner s operator once the scanner has alerted him or her ,)- fails to ac!nowledge the possibility that the scanner will not be equally sensitive to all !inds of e6plosives ,*- substitutes one group for a different group in the statement of a percentage

#nswer+ * E planation+ Let s assume for argument9s sa!e that the machine erroneously alerted the scanner for e6plosives when there were none present in 1G out of a thousand cases. or 1T of all bags. If you were to randomly choose 1GG cases, then it is quite possible that three of those 1G cases ,where the machine tagged the baggage by mista!e- ended up in the set of 1GG. %hus the conclusion drawn is wrong, and only * points the flaw in the conclusion.

1B..Hitcorp, a manufacturer, wishes to ma!e its information booth at an industry convention more productive in terms of boosting sales. %he both offers information introducing the company9s new products and services. %o achieve the desired result, Hitacorp9s mar!eting department will attempt to attract more people to the booth. %he mar!eting director9s first measure was to instruct each salesperson to call his or her five best customers and personally invite them to visit the booth. /hich of the following, if true, most strongly supports the prediction that the mar!eting director9s first measure will contribute to meeting the goal of boosting sales$ ,#- Hitacorp9s salespeople routinely inform each important customer about new products and services as soon as the decision to launch them has been made. ,'- 3any of Hitacorp9s competitors have made plans for ma!ing their own information booths more productive in increasing sales. ,(- #n infomation booth that is well attended tends to attract visitors who would not otherwise have attended the booth. ,)- 3ost of Hitacorp9s best customers also have business dealings with Hitcorp9s competitors. ,*- Hitacorp has fewer new products and services available this year than it had in previous years.

#nswer+ ( E planation+ If the best customers come out to the booth, ma!ing it well0attended, and good attendance leads to more sales, then the logic holds true.

1B5. %sycholo!ist+ "ome theories posit completely different causal mechanisms from those posited by the "mith psychological theory and that are more successful at predicting human behavior. %herefore, the "mith theory of behavior, no matter how elegant or comple6, ought to be abandoned in favor of these other theories. /hich of the following is an assumption made in drawing the conclusion above$ #- %he "mith theory has led to intriguing predictions, which have been shown to be false, about the causes of human behavior. '- # psychological theory with greater predictive success than another is scientifically preferable. (- %he "mith theory has had remar!able success in predicting how people will behave in certain situations. )- 3easuring the predictive success of a psychological theory always involves considering other theories that attempt to e6plain the same phenomena. *- "cientific theories become impractical if they posit causal mechanisms beyond a certain level of comple6ity.

#nswer+ ' E planation+ #nd, whatever the quality of the question, choice ' is definitely the correct answer. Loo! at his conclusion+ he uses the word 8ought8. /ords li!e 8ought8 and 8should8 signal that the author is ma!ing a recommendation. /hat is this author9s recommendation$ %hat the "mith theory should ,ought- be abandoned in favor of theories that have greater predictive success. "o the big supporting idea in the evidence is 8predictive success8. ,Notice that this idea is not in his conclusion.- "o he is assuming that when it comes to what theories are ,scientifically- preferable ,this was the language in choice '-, that predictive success is what matters #nd if you deny choice ', his argument falls apart+ if he didn9t thin! theories with better predictive success were preferable then how can he ma!e his argument$ ,Remember his argument was that other theories should be adopted ,in replacement of "mith9s- because they have better have predictive success.(hoice ) has nothing to do with the argument. Is the author ma!ing an argument about how best to measure the predictive success of a theory$ No. It is given in the author9s evidence that some theories ): IN =#(% en2oy better predictive success than "mith9s. %he author never argued anything about how that predictive success was or should have been measured.

1B7.#t present the >ollywood restaurant has only standard0height tables. >owever, many customers come to watch the celebrities who frequent the >ollywood, and they would prefer tall tables with stools because such seating would afford a better view of the celebrities. 3oreover, diners seated on stools typically do not stay as long as diners seated at standard0height tables. %herefore, if the >ollywood replaced some of its seating with high tables and stools, its profits would increase. %he argument is vulnerable to criticism on the grounds that it gives reason to believe that it is li!ely that #- "ome celebrities come to the >ollywood to be seen, and so might choose to sit at the tall tables if they were available '- %he price of meals ordered by celebrities dining at the >ollywood compensates for the longer time, if any, they spend lingering over their meals (- # customer of the >ollywood who would choose to sit at a tall table would be an e6ception to the generalization about lingering )- # restaurant9s customers who spend less time at their meals typically order less e6pensive meals than those who remain at their meals longer *- /ith enough tall tables to accommodate all of the >ollywood9s customers interested in such seating, there would be no view e6cept of other tall tables

#nswer+ ( E planation+ #- /rong 0 %he question is about seating the customers not celebrities '- /rong 0 Nobody tal!s about meals ordered by celebrities and it doesn9t wea!en the stated argument in any way (- Right 0 %his argument is vulnerable to the criticism that, there could be e6ceptional customers who might prefer sitting on taller stools as opposed to lingering. )- /rong 0 %his might wea!en the argument to some e6tent but we are not sure what the customers would order. #lso the argument doesn t mention anything about the meals ordered by customers, so it s out of scope. *- /rong0 It s completely irrelevant. %he argument s conclusion is about the profits that the restaurant would ma!e based on customers seating. Not about being able to accommodate all the customers.

1BA.%he cities with the densest population have the highest ratio of police officers to citizens. "uch cities also have the lowest rates of property crime without contact between perpetrator and victim. %hus maintaining a high ratio of police officers to citizens can serve as an effective deterrent to at least certain !inds of property crime.

/hich of the following, if true, most seriously wea!ens the argument above$ ,#- %he quality of training that police receive varies from city to city. ,'- >igh population density itself ma!es it difficult to commit a property crime that involves no contact between perpetrator and victim. ,(- 3any nonviolent crimes in large cities are drug0related. ,)- # ma2ority of the perpetrators of property crimes in densely populated cities are not apprehended by the police. ,*- 1roperty crimes without contact between perpetrator and victim represent only a small proportion of overall crime.

#nswer+ ' E planation+ %he conclusion tal!s about crime being 8deterred8. /hether or not criminals are being caught does not have anything to do with whether they are committing crimes, and so does not have anything to do with whether crime is being deterred. ) shares an attribute with a variety of popular wrong choices+ it ma!es a comment about subset of a logical variable. "ometimes, such a choice wea!ens the argument, unless the predicate00in this case, not apprehended by police00is the cru6 of the argument. %he argument doesn9t tal! about how many got apprehended. "o, it is useless to tal! about that.

1BB.(omplaints that mil! bottlers ta!e enormous mar!ups on the bottled mil! sold to consumers are most li!ely to arise when least warranted by the actual spread between the prices that bottlers pay for raw mil! and the price at which they sell bottled mil!. %he complaints occur when the bottled0mil! price rises, yet these price increases most often merely reflect the rising price of the raw mil! that bottlers buy from dairy farmers. /hen the raw0mil! price is rising, the bottlers mar!ups are actually smallest proportionate to the retail price. /hen the raw0mil! price is falling, however, the mar!ups are greatest.

If all of the statements above are true, which one of the following must also be true on the basis of them$ ,#- (onsumers pay more for bottled mil! when raw0mil! prices are falling than when these prices are rising.

,'- Increases in dairy farmers cost of producing mil! are generally not passed on to consumers. ,(- 3il! bottlers ta!e substantially greater mar!ups on bottled mil! when its price is low for an e6tended period than when it is high for an e6tended period. ,)- 3il! bottlers generally do not respond to a decrease in raw0mil! prices by straightaway proportionately lowering the price of the bottled mil! they sell. ,*- (onsumers tend to complain more about the price they pay for bottled mil! when dairy farmers are earning their smallest profits.

#nswer+ ) E planation+ 3il! bottlers generally do not respond to a decrease in raw0mil! prices by straightaway proportionately lowering the price of the bottled mil! they sell. (ould this be false$ %hat is, what would happen if mil! bottlers )I) respond to falling prices by straightaway proportionately lowering the price of the bottled mil! they sell$

/hat would happen to the last sentences of the passage$ In the last sentences we learn that the bottlers en2oy proportionately greater mar!ups when prices are falling. /ould they be able to en2oy these lavish mar!ups if they straightaway formed their own prices$ :f course not.

If we deny choice )00if the bottlers did respond to the falling mil! prices by lowering their prices, then one or more statements in the passage would no longer be true. 'ut in inference we must treat the passage necessarily true00therefore, choice ) must be true. ,Its denial must be false or else the last sentences of the passage must be false-.

1BC.# primate 2awbone found in Namibia in southern #frica has been identified by anthropologists as that of an ape that lived between 1G million and 1A million years ago. Researchers generally agree that such ancient primates lived only in dense forests. (onsequently, the dry, treeless e6panses now dominating the landscape in and around Namibia must have replaced an earlier heavily forested terrain.

%he argument assumes which one of the following$ ,#- 3odern apes also tend to live only in heavily forested terrain. ,'- %he ape whose 2awbone was found lived in or near the area that is now Namibia. ,(- %here were no apes living in the area that is now Namibia prior to 1A million years ago. ,)- %he ape whose 2awbone was found was adapted to a diet that was significantly different from that of any modem ape.

,*- %he ancient primates were numerous enough to have caused severe damage to the ecology of the forests in which they lived.

#nswer+ ' E planation+ #+ 3odern #pes00J its e6tra information. not part of the original argument )0 "ame as # *0J the conclusion is about current vegetation is dry. which may have replaced the earlier veg of ..)estruction of ecology by apes does not help tie the argument together (+ :ut of scope '+ is the last one remaining..hence the answer. #lso+ 1rimate 2awbone 00J found in Namibia ,research1rimates00J lived in dense forest (onclusion+ )ense 00J replaced by )R;

1BD.Recently in (ity E, developers have stopped buying land, contractors have found themselves going without wor! for longer periods, and ban!s have issued fewer mortgages. %here must be fewer new residents moving to (ity E than there were previously. /hich of the following indicates a flaw in the reasoning above$ # %his year several housing bloc!s have gone on the mar!et after being held up for months by legal red tape. ' %he average size of a new home has increased significantly over the past several years. ( Re0sales of condominiums have increased over the past si6 months. ) %he cost of materials such as lumber and cement has decreased over the past year. * "ales of other big0tic!et items, such as automobiles and boats, has remained steady over the past year.

#nswer+ # E planation+ %he conclusion of the argument is that fewer new residents move to city E based on 5 premises. 1. contractors stop buying land .. no wor! to do 5. fewer mortage %he conclusion is based on these 5 premises. In other words, 5 premises causes the conclusion and the author assumes that there are no other possibilities to e6plain this situation. 'ut, we need to wea!en the argument so there must be other alternate possibility that we can find. Namely, while these,5 premises- may be true, it is also possible that there are other causes. ,Remember, wea!en0the0argument question is really as!ing you to find a hole,missing lin!- in the argument.-

Let9s find an alternative. %hat9s what ,#- says. Not because of 5 premises, but because of 8unsold houses8 or 8avaiable houses8 on the mar!et. ,'-8size8 of the house$ it9s out of the scope. ,(-,)- and ,*- are irrelevant. %herefore, the answer is ,#- for sure.

1BF.=or a contest at the upcoming county fair, 3arie advocates for a ring toss because the rin! toss is consistently the favorite activity of the children who attend the fair. #nne, however, points out that adults win most of the prizes when playing ring toss. %o ma!e the chances of winning fair for everyone, #nne proposes a raffle. &arie rebuts that those who can afford to spend more on raffle tickets will have better chances of winnin!. %he two agree to hold a raffle in which each person can buy only one tic!et. %he two boldface portions in the argument above are best described by which of the following statements$ #. %he first is an e6ample of one activity& the second is an argument in favor of choosing that activity. '. %he first is an argument promoting one activity& the second invalidates the reason for promoting the other activity. (. %he first is the conclusion of one of the participants& the second is an argument that supports that conclusion. ). %he first is an argument promoting one activity& the second is an argument against promoting that activity. *. %he first is the conclusion of one of the participants& the second invalidates the other participant9s reasoning.

#nswer+ *

1CG.#ccording to a recent magazine article, of those office employees who typically wor! D hours at the office each day but sometimes say that they will wor! at home on a particular day, .A percent actually wor! less than one hour. #t the same time, over FG percent of those same office employees believe they are more productive wor!ing at home than wor!ing in their office. %he statements above, if true, best support which of the following conclusions about the office employees discussed in the article$ #. :n average, the office employees wor!ing at home for a day wor! fewer hours than office employees wor!ing at the office. '. 1G percent of the office employees are less productive wor!ing from home than wor!ing in their office. (. #t least 1A percent of the office employees do not define productivity e6clusively in terms of the number of hours wor!ed. ). #t least .A percent of the office employees can complete the same amount of wor! in one hour at home as in D hours at the office. *. "ome of the office employees ma!e statements regarding their productivity that are not in fact true.

#nswer+ (

E planation+ %he question in which we have to 8identified the conclusion8 can be solved with the help of conditional reasoning. Let us ma!e the conditional diagram from the given premises. I am ma!ing rough diagram through te6t only. =or more clarity, please refer to the boo!. 1. #ccording to a recent magazine article, of those office employees who typically wor! D hours at the office each day 722;0 A H hours 4at office6 .. 'ut sometimes say that they will wor! at home on a particular day, .A percent actually wor! less than one hour. 1@;0 A 7 hour 4at home6 5. #t the same time, over FG percent of those same office employees believe they are more productive wor!ing at home than wor!ing in their office. 82;0 A more %roductive 4at home6 Now let9s see each given answer choice one by one+ 0 %he tric! is to identified the correct option which retrace the path of the premises. #- /e !now only that .AT people have spent less than one hour at home on wor!ing. 'ut we don9t the wor!ing hours of other employees at home. "o, we can9t say this ,option #- with surety because may be people the remaining CAT of people have spend more than D hours or may be less. Nothing is sure at all. >ence IN(:RR*(%. R*3*3'*R+ "tic! to premise only. '- 1GT/ I N:% ,productive ,at home-the above condition is opposite of the .nd premises9 condition. "o, it is wrong. R*3*3'*R+ %o choose only choice that retraces the premises9 path. #lso, to understand further, we !now only about the FGT more productive employee ,at home- and we don9t !now whether rest 1GT are whether productive at home same as at office or less productive at home as compare to office. "%I(O %: %>* 1R*3I"* IN=:R3#%I:N. (- 1AT/ I N:% ,define wor! in terms of hours=rom the premises conditional statements. ,5- states that FGT productive at home and ,.- states that .AT wor! for one hour. ,It means 1GG 0 FG W .A-T/ are doing same amount of wor! in less hour ,i.e. 1 hour or less-. >ence, 1AT are not tal!ing productivity in terms of wor!ing hours. >ence (:RR*(%. (hoose this choice. %o simplify further, we !now that FGT productive at home and we don9t !now anything about rest 1GT. #lso, we !now that .AT wor!ed for less than 1 hour at home& those supposed as to be less productive. "o if we subtract 1GT ,@n!nown production value- from these .AT ,!nown wor!ing timing- then we can conclude that 1AT from these .AT ,wor!ing for less than 1 hour- completed their wor! within that time at home. %hat means, they are not considering time spend I production value. >ence, choose this choice" )- .AT/ ,D hours ^ office- I .A/T ,1 hour ^home)o we have any such statement in any of the conditional statements created from premises$ N:. >ence IN(:RR*(%. *- "ome of the office employees ma!e statements regarding their productivity that are not in fact true. "ome ,/- I "pea!ing LI* %his is totally against the premises. #s premises conditions are considered to be true only, hence ,*- is IN(:RR*(%

1C1."pectroscopic analysis has revealed the e6istence of frozen nitrogen, methane, and carbon mono6ide on the surface of 1luto. "uch ices have a tendency to vaporize, producing an atmosphere. "ince the proportion of any gas in such an atmosphere depends directly on how readily the corresponding ice vaporizes, astronomers have concluded that the components of 1luto s atmosphere are nitrogen, carbon mono6ide, and methane, in order of decreasing abundance.

%he astronomers argument relies on which one of the following assumptions$ ,#- %here is no more frozen nitrogen on the surface of 1luto than there is either frozen carbon mono6ide or methane. ,'- @ntil space probes reach 1luto, direct analysis of the atmosphere is impossible. ,(- %here is no frozen substance on the surface of 1luto that vaporizes more readily than methane but less readily than carbon mono6ide. ,)- Nitrogen is found in the atmosphere of a planet only if nitrogen ice is found on the surface of that planet. ,*- # mi6ture of nitrogen, carbon mono6ide, and methane is characteristic of the substances from which the "olar "ystem formed.

#nswer+ (

1C..3ost of the world9s supply of uranium currently comes from the mines. It is possible to e6tract uranium from seawater, but the cost of doing so is greater than the price that @ranium fetches on the world mar!et. %herefore, until the cost of e6tracting uranium from seawater can somehow be reduced, this method of obtaining uranium is unli!ely to be commercially viable. /hich of the following would it be most useful to determine in evaluating the argument$ a. /hether the uranium in deposits on land is rapidly being depleted b. /hether most uranium is used near where it is mined c. /hether there are any technological advances that show promise of reducing the costs of e6tracting uranium from seawater d. /hether the total amount of @ranium in seawater is significantly greater than the total amount of uranium on land e. /hether uranium can be e6tracted from freshwater at a cost similar to the cost of e6tracting it from seawater.

#nswer+ # E planation+ In order to be 8useful8 in evaluating the argument an answer choice has to fall directly within the scope of the argument. #ccordingly, in evaluate the argument or relevant information questions it is important to determine the author s central assumption. %he correct answer will always be closely related to the author s assumption. It is also useful to view these questions as being hybrid strengthenMwea!en. %he answer choices will often be questions. >ere, all of the answer choices begin with Kwhether.L #s! of each answer choice whether it is the case or whether it is not. %he correct answer will be something where if it goes one way, it will strengthen the argument, and if it goes another way, it will wea!en the argument. >ere, the author has two main pieces of evidence+ 1- most of the world s supply of uranium currently comes from mines and .- it is costly to e6tract uranium from the sea. >e uses these two pieces of evidence to conclude that this method will remain commercially unviable ,unless the cost can be reduced-. Let s loo! at choices # and (+ a. /hether the uranium in deposits on land is rapidly being depleted /ell, what if uranium from land was not being depleted$ %hen the argument is strengthened, as the supply of coal won t be diminished, and so there will be no ,economic- reason to turn to the sea.

#nd what if uranium from land /#" being rapidly depleted$ %hen the supply of coal would dry up, coal prices would most li!ely go up, and all of a sudden it might be commercially viable ,profitable- to mine coal from the sea00the argument is wea!ened. In fact, this answer choice is essentially the author s assumption. It is a necessary assumption of the argument that the coal from mines continue to provide a ready supply. )enial test+ if coal from mines dried up, it may well be economically ,commercially- viable to turn to the sea, and the argument falls apart. =iguring out the assumption before going to the answer choices is of huge benefit in these questions. Remember you can always verify whether you ve figured out the necessary assumption correctly by applying the denial test. c. /hether there are any technological advances that show promise of reducing the costs of e6tracting uranium from seawater /ell, what if there were$ %he author would simply remind you that he said 8.until the cost of e6tracting uranium from seawater can somehow be reduced...8 %he author ac!nowledged that mining coal from the sea might be worth it if they could figure out a cheaper way of doing it. %herefore, determining whether there are any technological advances that could ma!e it cheaper would not be very useful in evaluating the merits of his argument.

1C5. #rcheolo!ist+ %he discovery of chic!en bones in a recently e6cavated Incan settlement, dating between 15.G and 171G (.*., suggests that chic!ens were brought to the #mericas by 1olynesians before introduction by the "paniards. It is also possible that contamination caused the specimens to appear older than they actually are. %o settle this, scientists sequenced )N# from the ancient chic!en bones and from 1olynesian chic!ens and found that they share a genetic signature. # later genetic study revealed that the same signature is found in chic!ens from throughout *urope and #sia.

/hich of the following, if true, most helps to e6plain the findings of the later study$ ,#- *uropean e6plorers in the 1Bth century collected live animals from throughout the #mericas and returned them to *urope for study and breeding. ,'- %he Incas did not raise chic!en for food. ,(- )N# sequencing is a reliable method of identifying the migration of species from one continent to another, even over a period of centuries. ,)- In diaries of #sian e6peditions, *uropean e6plorers refer to feasting on a game bird that may be related to the 1olynesian chic!en. ,*- %he "paniards did not introduce chic!ens to the #mericas until at least 1AGG (.*.

#nswer+ # E planation+ %his is an e6planation question. %he information in the passage suggests that genetic evidence lin!s ancient chic!ens from the #mericas to 1olynesian chic!ens, and also to chic!ens in *urope and #sia. /e9re loo!ing for a choice that e6plains why the genetic signature for the ancient and 1olynesian chic!ens is the same as that of the *uropean and #sian chic!ens.

,#- %his is correct. If e6plorers too! chic!ens that were related to the ancient chic!ens from the #mericas, then bred them in *urope, it would e6plain the presence of the same genetic signature. ,'- %he use of the chic!ens is completely off0topic. ,(- %his is a general statement that doesn9t e6plain or contradict the studies in the passage. @sually, such general statements are incorrect. ,)- %his choice suggests that chic!ens similar to 1olynesian chic!ens may been in #sia for some time, but the words 8may be8 ma!e the choice much wea!er. ,*- %he study suggests that chic!ens first appeared in the #mericas because they were introduced by someone other than the "panish. If thatYs the case, it doesn9t matter when the "paniards introduced chic!ens to the #mericas.

1C7.;orco and _ortech are two corporations that employ large numbers of full0time wor!ers who are paid by the hour. 1ublicly available records indicate that ;orco employs roughly the same number of such hourly wage wor!ers as _ortech does but spends a far higher total sum per year on wages for such wor!ers. %herefore, hourly wages must be higher, on average, at ;orco than at _ortech, since 44444.

#. _ortech spends a higher total sum per year than ;orco does to provide its hourly wage wor!ers with benefits other than wages '. the wor! performed by hourly wage wor!ers at _ortech does not require a significantly higher level of s!ill than the wor! performed by hourly wage wor!ers at ;orco does (. the proportion of all company employees who are hourly wage wor!ers is significantly greater at ;orco than it is at _ortech ). overtime wor!, which is paid at a substantially higher rate than wor! done during the regular wor! wee!, is rare at both ;orco and _ortech *. the highest hourly wages paid at ;orco are higher than the highest hourly wages paid at _ortech

#nswer+ ) E planation+ %his can be treated as another e6plain the phenomenon argument. %he phenomenon+ *qual number of hourly paid employees but ;orco spent more money paying theirs. %he author9s e6planation ,conclusion-+ ;orco pays their employees more. /e need to fill in the blan! after the word 8since.8 8"ince8 is an evidence !eyword. "o, to support the author9s conclusion that his e6planation is the correct one, we should remove an alternative e6planation for the phenomenon. (hoice ) does that. ,>igher rates of overtime wor! at ;orco could have been an alternative e6planation for the phenomenon-. 1CA.(urrent farm policy is institutionalized penalization of consumers. It increases food prices for middle0 and low0income families and costs the ta6payer billions of dollars a year. /hich of the following statements, if true, would provide support for the author s claims above$ I. =arm subsidies amount to roughly U.G billion a year in federal payouts and U1. billion more in higher food prices. II. #ccording to a study by the )epartment of #griculture, each U1 of benefits provided to farmers for ethanol production costs consumers and ta6payers U7. III. %he average full0time farmers have an average net worth of over U5GG,GGG. ,#- I only

,'- II only ,(- III only ,)- I and II only ,*- I, II, and III

#nswer+ ) E planation+ %his is because both the first and the second statement refer to food costs ,1st one- and ta6ation ,1st and .nd-. /ell, subsidies come from ta6es 0 it9s 2ust another way of saying that the ta6payer is paying for the e6tra money that the farmers get from the government. >igher food prices usually affect people with low and medium income, since food is a basic good and needs to be covered all the time.

1CB. /ennifer+ Hideo rental outlets in (enterville together handled 1G,GGG fewer video rentals in 1FF7 than in 1FF5. %he decline in rentals was probably due almost entirely to the =ebruary 1FF7 opening of Hideorama, the first and only video rental outlet in the area that, in addition to renting videos, also sold them cheaply. 5rad+ %here must be another e6planation+ as you yourself said, the decline was on the order of 1G,GGG rentals. ;et Hideorama sold only 7,GGG videos in 1FF7. /hich of the following, if true, would most seriously wea!en the force of the ob2ection that 'rad presents to Nennifer9s e6planation$ ,#- In 1FF7 Hideorama rented out more videos than it sold. ,'- In 1FF7 two new outlets that rent but that do not sell videos opened in (enterville. ,(- 3ost of the video rental outlets in (enterville rent videos at a discount on certain nights of the wee!. ,)- 1eople often buy videos of movies that they have previously seen in a theater. ,*- 1eople who own videos frequently loan them to their friends

#nswer+ * E planation+ 3any strengthenMwea!en questions can be viewed as cause and effect or e6plain the phenomenon arguments+ *he phenomenon 4effect6: (enterville9s video rentals declined by 1G,GGG. /ennifer(s e planation 4cause6: %he opening of Hideorama. %he heart of Nennifer9s e6planation is that Hideorama sells videos cheaply. %his is the substance or main evidence for her conclusion that it was Hideorama9s opening that caused the phenomenon. 5rad(s ob)ection: %here must be some other e6planation because Hideorama only sold 7GGG. "o 'rad9s ob2ection centers on how Hideorama9s selling of videos can9t have been a sufficient e6planation ,for (enterville9s decline in number of videos rented out.%he question stem instructs us to wea!en the force of 'rad9s ob2ection. (hoice * succeeds because it suggests that Hideorama9s selling of videos may well be a sufficient e6planation for the phenomenon ,the decline in the number of videos rented out by (enterville-. (hoice # does strengthen the general idea that (enterville9s decline was due to Hideorama enticing away customers. '@% it suggests that Hideorama did this more by renting out rather than by selling videos. %herefore, 'rad9s ob2ection still stands.

1CC.%o hold criminals responsible for their crimes involves a failure to recognize that criminal actions, li!e all actions, are ultimately products of the environment that forged the agent s character. It is not criminals but people in the law0abiding ma2ority who by their actions do most to create and maintain this environment. %herefore, it is law0abiding people whose actions, and nothing else, ma!e them alone truly responsible for crime.

%he reasoning in the argument is most vulnerable to criticism on the grounds that ,#- it e6ploits an ambiguity in the term KenvironmentL by treating two different meanings of the word as though they were equivalent ,'- it fails to distinguish between actions that are socially acceptable and actions that are socially unacceptable ,(- the way it distinguishes criminals from crimes implicitly denies that someone becomes a criminal solely in virtue of having committed a crime ,)- its conclusion is a generalization of statistical evidence drawn from only a small minority of the population ,*- its conclusion contradicts an implicit principle on which an earlier part of the argument is based

#nswer+ * E planation+ %he author wants to absolve criminals of fault hood because 8criminal actions, li!e all actions are ultimately products of the environment...8 'ut then, later in the argument, the author wants to center fault hood on the law0abiding citizenry because they 8by their actions do the most to create and maintain that environment.8 I9m sure you see the contradiction in his argument+ %he reason he wants to absolve one group of fault hood is the same as the reason he wants to ground fault hood on another group. %herefore, he his contradicting himself00that is what ma!es his argument flawed, and that would be a good prediction of the right answer. Now, aggressively scan for a match to that prediction+ %hen, (hoice * matches this prediction. 'ecause we spent time solving the argument and generating a prediction, we don9t want to spend a lot of time drowning in the answer choices. "o, we aggressively scan for the match, and we don9t overuse 1:*. 1:* means figuring out four reasons why four wrong answers are wrong& an arduous process considering how good the test0ma!er is at designing seductive wrong answers. #lso, by doing this, we get suc!ed in by choices and our original analysis of the argument dims. /e don9t care about wrong answers and why they are wrong. #nd we get rewarded for one and one thing only+ selecting the correct answer. Remember, you don9t have to decide whether a choice is correct the first time you read it& instead, !eep the predictionMsolution clear as day in your head, and aggressively scan for a response that impresses you as matching. #nyways, because this is a clear flaw in the argument, and because it matches one of the answer choices, choice * must be correct. #nd (hoice ( is wrong because (hoice * is right. ,/e don9t care why wrong answers are wrong, and critical reasoning is 2ust as ob2ective as math00there is one and only one correct answer00no such thing as a 8good8 or 8better8 answer.#nyways, for reviewing ,rather than performingMpracticing- purposes, let9s ta!e a loo! at choice (+ ,(- >ow, e6actly, does the stimulus 8distinguish8 criminals from crimes$ >ow does anyone distinguish a criminal from a crime$ Let9s see, 8distinguish8 means 8to tell apart.8 "o, how does one tell apart a criminal from a crime$ /ell, obviously, a 8criminal8 is a person and a 8crime8 is an act. %hat9s how you tell those things apart. %he

author clearly does not have to draw such an obvious distinction. #nd so he didn9t. %his means that choice ( is wrong+ the right answer to a flaw question is always a reasoning error that the author actually committed00 you can always eliminate a flaw answer choice if the author didn9t even 8do8 the thing that the answer choice describes. 'ut even if the answer choice had instead said 8fails to distinguish between criminals and crimes,8 it would still be wrong because it is tremendously unli!ely that failure to draw such an obvious distinction would ever be the reason why a <3#% arguer9s reasoning would be flawed.

#lso, choice ( states that the author implicitly denies that someone becomes a criminal 2ust because they committed a criminal act. In fact, the author more or less e6plicitly denies this.

1CD.In the state of 3ichigan, from 1FDG to 1FDF, total spending on boo!s purchased from all the sources increased by 57T. 'ut during the same period, spending on fiction boo!s, most of which were purchased from boo!stores selling only new boo!s, grew by 1B T.

/hich of the following statements about the period mentioned is best supported by the statements above$ a- "pending on non0fiction boo!s increased by more than 57T percent. b- "hoppers were more li!ely to buy fiction boo!s when they went to a boo!store than they were to buy non0 fiction. c- %he prices of boo!s purchased at boo!0stores are higher than those of boo!s purchased elsewhere d- Individual spending on the boo!s increased, while institutional spending decreased. e- %he number of people who bought boo!s from secondhand boo!stores increased during this period.

#nswer+ #

1CF.1remiums for automobile accident insurance are often higher for red cars than for cars of other colors. %o 2ustify these higher charges, insurance companies claim that, overall, a greater percentage of red cars are involved in accidents than are cars of any other color. If this claim is true, then lives could undoubtedly be saved by banning red cars from the roads altogether.

%he reasoning in the argument is flawed because the argument ,#- #ccepts without question that insurance companies have the right to charge higher premiums for higher0 ris! clients ,'- =ails to consider whether red cars cost the same to repair as cars of other colors ,(- ignores the possibility that drivers who drive rec!lessly have a preference for red cars

,)- )oes not specify precisely what percentage of red cars are involved in accidents ,*- 3a!es an unsupported assumption that every automobile accident results in some loss of life

#nswer+ ( E planation+ 3ust be (, because ( shows that the author does not consider that the cause of these accidents by red cars is not the color of the cars, but the drivers who prefer to drive red cars.

1DG.# law requiring companies to offer employees unpaid time off to care for their children will harm the economic competitiveness of our nation s businesses. (ompanies must be free to set their own employment policies without mandated parental0leave regulations.

/hich of the following, if true, would most seriously wea!en the conclusion of the argument above$ ,#- # parental0leave law will serve to strengthen the family as a social institution in this country. ,'- 3any businesses in this country already offer employees some form of parental leave. ,(- "ome of the countries with the most economically competitive businesses have strong parental0leave regulations. ,)- :nly companies with one hundred or more employees would be sub2ect to the proposed parental0leave law. ,*- In most polls, a ma2ority of citizens say they favor passage of a parental0leave law.

#nswer+ ( E planation+ Loo!ing through the answer choices, # and * can be eliminated. %hese choices are outside the scope of the conclusion. %he main sub2ect of the passage is Keconomic competitiveness of our nation s business,L not the Kfamily as a social institution.L #lso the citizen s opinion is not relevant to the message in the passage. Loo! for answer choices that offer detracting evidence or reveal faulty assumptions. ' does neither. ( offers a corollary to the law but does not have information pertaining to the conclusion, which is the economic competitiveness of a nation s business. #nswer ( states that many other countries manage to stay competitive despite strong parental leave laws. ( is the correct answer.

1D1. Editorialist: News media rarely cover local politics thoroughly, and local political business is usually conducted secretively. %hese factors each tend to isolate local politicians from their electorates. %his has the effect of reducing the chance that any particular act of resident participation will elicit a positive official response, which in turn discourages resident participation in local politics.

/hich one of the following is most strongly supported by the editorialist s statements$ ,#- 1articular acts of resident participation would be li!ely to elicit a positive response from local politicians if those politicians were less isolated from their electorate. ,'- Local political business should be conducted less secretively because this would avoid discouraging resident participation in local politics. ,(- %he most important factor influencing a resident s decision as to whether to participate in local politics is the chance that the participation will elicit a positive official response. ,)- 3ore0frequent thorough coverage of local politics would reduce at least one source of discouragement from resident participation in local politics. ,*- If resident participation in local politics were not discouraged, this would cause local politicians to be less isolated from their electorate.

#nswer+ #

1D.. .rom an article in the 0all +treet $hronicle: "ales statistics of ma2or electronics manufacturers with sales in the @nited "tates show that DGT of consumer electronics ,such as televisions, )H) players, and computers- sold in the @.". last year were manufactured in (hina. .rom an article in $onsumer :esults &a!azine: %he results from last year9s survey on consumer electronics choices show that while products made in (hina are still very popular, more and more #mericans are buying products made in Napan, <ermany, and the @nited "tates. %hese three countries combined account for 5DT of products sold in the @.". last year.

=or both of the findings to be accurate, which of the following must be true$ ,#- 3ore #mericans who do not purchase consumer electronics prefer goods produced in (hina to those produced elsewhere. ,'- 3a2or electronics manufacturers do not limit their production plants to one country, often dividing different stages of manufacturing among plants around the world. ,(- 3ost consumer electronics purchased last year that were not manufactured in (hina were manufactured and sold in the @nited "tates. ,)- %he average price of a (hinese0manufactured consumer electronics device is lower than that of a device manufactured elsewhere. ,*- 3a2or electronics manufacturers sell a higher percentage of (hinese0produced consumer devices than do smaller manufacturers.

#nswer+ * E planation+ %his is an e6planationMparado6 question. =rom a quic! reading, the two reports seem to claim that DGT of electronics were manufactured in (hina, and 5DT of products were made in other places. (learly that9s incorrect00there must be more to the story. %he distinction is in the /all "treet (hronicle claim, which is limited to 8ma2or8 electronics manufacturers. %he second claim does not ma!e that distinction. %hus, it would appear that, while .GT of products sold in the @.". made by 8ma2or8 electronics manufacturers did not come from (hinese manufacturers, a greater percent of products sold in the @.". by non0ma2or manufacturers did not come from (hina.

%hin! of it li!e a weighted average question. #ll manufacturers are either ma2or or non0ma2or. If .GT of the ma2or company sales were non0(hina made and 5DT of total sales were non0(hina made, then non0ma2or sales must have been greater than 5DT non0(hina made. (hoice ,*- is the only option consistent with that conclusion. If the products of non0ma2or manufacturers are more than 5DT non0(hina made, they must be much less than DGT made in (hina. %hus, ,*- is correct. 1D5.1lant scientists have used genetic engineering on seeds to produce crop plants that are highly resistant to insect damage. Gnfortunately, the seeds themselves are quite e pensive, and the plants require more fertilizer and water to !row well than normal ones. %hus, for most farmers the savings on pesticides would not compensate for the higher seed costs and the cost of additional fertilizer. However, since consumer demand for !rains, fruits, and ve!etables !rown without the use of pesticides continues to rise, the use of !enetically en!ineered seeds of this kind is likely to become widespread"

In the argument given, the two portions in boldface play which of the following roles$ ,#- %he first supplies a conte6t for the argument& the second is the argument9s main conclusion. ,'- %he first introduces a development that the argument predicts will have a certain outcome& the second is a state of affairs that the argument denies will be part of that outcome. ,(- %he first presents a development that the argument predicts will have a certain outcome& the second ac!nowledges a consideration that weighs against that prediction. ,)- %he first provides evidence to support a prediction that the argument see!s to defend& the second is that prediction. ,*- %he first and the second each provide evidence to support the argument9s main conclusion.

#nswer+ ( E planation+ %he !ey thing to notice is, first, the two boldface statements are on opposite sides of the fence. "econd, the first boldface is a fact that is used to support sentence 5. %he second boldface goes against sentence 5 0 it can9t actually go against sentence ., since ". is a fact, but it does go against the main point of view given in "5. I want a choice that is consistent with the above relationships. #- %he first is not conte6t ,or bac!ground- 0 it is a premise used to draw a conclusion. %he second is not the main point of view but a contrasting point of view. ,%his one also implies the two are on the same side of the fence and they are notS'- %he first part is fine, but the second is not. %he argument does not deny that the second boldface will be part of the outcome 0 rather, it says that the contrasting viewpoint is li!ely to overcome the main point of view. (- (orrect )- %he author does not actually attempt to defend either conclusion 0 it 2ust presents the two. %his choice

also says the two are on the same side of the fence when they9re not. *- %his one says the two are on the same side of the fence. 0000 81rediction8 means that the argument is predicting that something will happen. If there is a prediction, it 3@"% be the conclusion of the argument, unless it is being used as the basis for future predictions ,which it probably won9t-. # claim is a statement that is not a fact, and requires an argument andMor supporting evidence. *vidence is =#(%@#L information that is used to support a claim. # consideration can be either a fact or a claim& it9s used to support some other claimMconclusion.

1D7.:f patients over BA years old who survived coronary bypass surgery P a procedure widely prescribed for people with heart disease P only CA percent benefited from the surgery. %hus it appears that for one in four such patients, the doctors who advised them to undergo this surgery, with its attendant ris!s and e6pense, were more interested in an opportunity to practice their s!ills and in their fee than in helping the patient. /hich of the following, if true, most seriously undermines the argument$ #. 3any of the patients who received coronary bypass surgery are less than AA years old '. 1ossible benefits of coronary bypass surgery include both relief from troubling symptoms and prolongation of life. (. 3ost of the patients in the survey decided to undergo coronary bypass surgery because they were advised that the surgery would reduce their ris! of future heart attac!s ). %he patients over BA years old who did not benefit from the coronary bypass surgery were as fully informed as those who did benefit from the surgery as to the ris!s of the surgery prior to undergoing it *. %he patients who underwent coronary bypass surgery but who did not benefit from it were medically indistinguishable, prior to their surgery, from the patients who did benefit. #nswer+ * E planation+ argument is 00 )octors were more interested in an opportunity to practice their s!ills and in their fee than in helping the patient. ) 00 "ays that all the patients were fully informed of the ris!s of the surgery. If the doctor !new that a certain patient won9t benefit from the surgery, then also they were informed of the ris!s. Q'ut is not that they will not benefit from the surgeryR * 00 3edically doctors could not distinguish between patients who benefited from the surgery from those who did not. %his sentence truly wea!ens the argument. 1DA.#lthough the discount stores in <oreville s central shopping district are e6pected to close within five years as a result of competition from a "pendLess discount department store that 2ust opened, those locations will not stay vacant for long. In the five years since the opening of (olson s, a nondiscount department store, a new store has opened at the location of every store in the shopping district that closed because it could not compete with (olson s. /hich of the following, if true, most seriously wea!ens the argument$ #. 3any customers of (olson s are e6pected to do less shopping there than they did before the "pendLess store opened. '. Increasingly, the stores that have opened in the central shopping district since (olson s opened have been discount stores. (. #t present, the central shopping district has as many stores operating in it as it ever had. ). :ver the course of the ne6t five years, it is e6pected that <oreville s population will grow at a faster rate than it has for the past several decades.

*. 3any stores in the central shopping district sell types of merchandise that are not available at either "pendLess or (olson s.

#nswer+ ' E planation+ %he !ey to the argument is the #""@31%I:N that the previous trend of stores replacing old ones will continue. "pecifically, when the previous store closures occurred, new stores too! their place. %he argument assumes that, should these new stores close within A years, still more new stores will ta!e their place. #nything that casts doubt on this #""@31%I:N 0 i.e., that ma!es it L*"" li!ely that even more new stores will spring up to ta!e the place of the old ones 0 will wea!en the argument. %his is what ,b- does. If the new stores were discount stores, that9s why they were able to compete with colson9s. >owever, since spendless is a big discount store, even these discount stores won9t be able to compete with it.

1DB.*very fall (roton9s 2ays migrate south. %he 2ays always 2oin floc!s of migrating croo!bea!s with which they share the same summer and winter territories. If a 2ay from the croo!bea!s it is accompanying, it wanders until it comes across another floc! of croo!bea!s. (learly, therefore, (roton9s 2ays lac! the navigational ability to find their way south on their own. "trengthen the argument+ #. (roton9s 2ays lay their eggs in the nests of croo!bea!s which breed upon completing their southern migration. '. %he three species most closely related to croo!bea!s do not migrate at all. (. In the spring, (roton9s 2ays migrate north in the company of %attersall warblers. ). "pecies other than (roton9s 2ays occasionally accompany floc!s of migrating croo!bea!s. *. In the spring, croo!bea!s migrate north before croton9s 2ays do. #nswer+ ( E planation+ takeaway: in +*:EN,*HEN 9 0E#EEN, you should #L0#I+ select answers that have as ?':E$* an impact on the ar!ument as possible" /e need a choice that )IR*(%L; evinces the 2ay9s lac! of navigational ability. \ /here the 2ays lay their eggs has nothing whatsoever to do with navigational ability. ,%his may 2ust mean that they9re clever enough to ta!e advantage of others9 nests, li!e cuc!oos, rather than going to the trouble of building their own.\ If the 2ays #L": need help finding their way bac! north, this is much stronger evidence that they can9t navigate by themselves.

1DC.=rom 1FCD to 1FDD, beverage containers accounted for a steadily decreasing percentage of the total weight of household garbage in the @nited "tates. %he increasingly widespread practice of recycling aluminum and glass was responsible for most of this decline. >owever, although aluminum recycling was more widely practiced in this period than glass recycling, it was found that the weight of glass bottles in household garbage declined by a greater percentage than the weight of aluminum cans. /hich of the following, if true of the @nited "tates in the period 1FCD to 1FDD, most helps to account for the finding$ ,#- <lass bottles are significantly heavier than aluminum cans of comparable size.

,'- Recycled aluminum cans were almost all beverage containers, but a significant fraction of the recycled glass bottles had contained products other than beverages. ,(- 3anufacturers replaced many glass bottles, but few aluminum cans, with plastic containers. ,)- %he total weight of glass bottles purchased by households increased at a slightly faster rate than the total weight of aluminum cans. ,*- In many areas, glass bottles had to be sorted by color of the glass before being recycled, whereas aluminum cans required no sorting. #nswer+ ( E planation+ this problem turns on the differences between 1*R(*N%#<*" and #'":L@%* N@3'*R". =acts+ \ the 1*R(*N%#<* of recycled aluminum was higher than the 1*R(*N%#<* of recycled glass. this is what it means when we say that aluminum recycling was more widely practiced. '@% \ the N@3*RI(#L (>#N<* in aluminum in the trash was lower than the N@3*RI(#L (>#N<* in glass in the trash. If all the stuff missing from the trash had been recycled, this would be impossible. %herefore, we need another e6planation, besides recycling, for />; %>* %:%#L @"* := <L#"" >#" <:N* ):/N. %hat s the only way that these findings are mathematically possible. ,c- Is such a statement. ,d- Is the *E#(% :11:"I%* of this sort of statement. If ,d- were true, we would e6pect to see aluminum, not glass, declining at a faster rate in the trash.

1DD.<enerally scientists enter their field with the goal of doing important new research and accept as their colleagues those with similar motivation. %herefore, when any scientist wins renown as an e6pounder of science to general audiences, most other scientists conclude that this popularizer should no longer be regarded as a true colleague. %he e6planation offered above for the low esteem in which scientific popularizers are held by research scientists assumes that ,#- serious scientific research is not a solitary activity, but relies on active cooperation among a group of colleagues ,'- research scientists tend not to regard as colleagues those scientists whose renown they envy ,(- a scientist can become a famous popularizer without having completed any important research ,)- research scientists believe that those who are well !nown as popularizers of science are not motivated to do important new research ,*- no important new research can be accessible to or accurately assessed by those who are not themselves scientists

#nswer+ ) E planation+ the passage already says that not doing important research X00J should not be regarded as colleague so, if you add in assumption ,d-, you get popularizer X00J not doing important research X00J should not be regarded as colleague ...which is e6actly what you are loo!ing for.

1DF.'ecause postage rates are rising, >ome )ecorator magazine plans to ma6imize its profits by reducing by one half the number of issues it publishes each year. %he quality of articles, the number of articles published per year, and the subscription price will not change. 3ar!et research shows that neither subscribers nor advertisers will be lost if the magazine9s plan is instituted. /hich of the following, if true, provides the strongest evidence that the magazine9s profits are li!ely to decline if the plan is instituted$ #. /ith the new postage rates, a typical issue under the proposed plan would cost about one0third more to mail than a typical current issue would. '. %he ma2ority of the magazine9s subscribers are less concerned about a possible reduction in the quantity of the magazine9s articles than about a possible loss of the current high quality of its articles. (. 3any of the magazine9s long0time subscribers would continue their subscriptions even if the subscription price were increased. ). 3ost of the advertisers that purchase advertising space in the magazine will continue to spend the same amount on advertising per issue as they have in the past. *. 1roduction costs for the magazine are e6pected to remain stable.

#nswer+ ) E planation+ )+ if u loo! at the stimulus+ quality W quantity W rate are all same. "o we can assume that production cost is relatively unchanged. ) says+ advertisers will pay same rate per issue as they were paying earlier. "o if the cost is constant, but there is a drop of AGT in ad revenue, profits should obviously decrease.

1FG. &a!azine %ublisher+ :ur magazine does not have a liberal bias. It is true that when a boo! review we had commissioned last year turned out to e6press distinctly conservative views, we did not publish it until we had also obtained a second review that too! a strongly liberal position. (learly, however, our actions demonstrate not a bias in favor of liberal views but rather a commitment to a balanced presentation of diverse opinions. )etermining which of the following would be most useful in evaluating the cogency of the magazine publisher s response$ #- /hether any other magazines in which the boo! was reviewed carried more than one review of the boo! '- /hether the magazine publishes unsolicited boo! reviews as well as those that it has commissioned (- /hether in the event that a first review commissioned by the magazine ta!es a clearly liberal position the magazine would ma!e any efforts to obtain further reviews )- /hether the boo! that was the sub2ect of the two reviews was itself written from a clearly conservative or a clearly liberal point of view the magazine publishes *- /hether most of the readers of the magazine regularly read the boo! reviews that the magazine publishes

#nswer+ ( E planation+ the biggest weapon you have in problems li!e this is the ability to simplify the argument. In this case, here9s a simplified version+ they got a conservative review, but then waited until they got a liberal review. %hey say they were 2ust being balanced. %hey say that they ):N9% have a bias in favor of liberal views.

/e need to pic! an answer choice that will )I"%IN<@I"> '*%/**N %>*"* %/: 1:""I'ILI%I*". i.e., 3#<#_IN* I" LI'*R#L vs. 3#<#_IN* I" '#L#N(*).

%he only one of the situations described that would affect this 2udgment is ,c-. If the magazine were liberal, then they would N:% obtain further reviews. If the magazine were balanced, then they /:@L) obtain further reviews. %he other four situations would be unaffected by whether the magazine is balanced or liberal.

1F1. #stronomer+ :bservations of the "hoema!er0Levi comet on its collision course with Nupiter showed that the comet bro!e into fragments before entering Nupiter9s atmosphere in 1FF7, but they did not show how big those fragments were. In hopes of gaining some indication of the fragments9 size, astronomers studied spectrographic analyses of Nupiter9s outer atmosphere. %hese analyses revealed unprecedented traces of sulfur after the fragments9 entry. *he fra!ments themselves almost certainly contained no sulfur, but many astronomers believe that the cloud layer below Nupiter9s outer atmosphere does contain sulfur. "ince sulfur would have seeped into the outer atmosphere if comet fragments had penetrated this cloud layer, it is likely that some of the fra!ments were at least lar!e enou!h to have passed throu!h /upiter(s outer atmosphere without bein! burned up" In the astronomer9s argument, the two portions in boldface play which of the following roles$ #. %he first presents a circumstance for which the astronomer offers an e6planation& the second is part of that e6planation. '. %he first ac!nowledges a consideration that weighs against the conclusion of the argument& the second is that conclusion. (. %he first ac!nowledges a consideration that weighs against the conclusion of the argument& the second provides evidence in support of that conclusion. ). %he first provides evidence in support of the conclusion of the argument& the second ac!nowledges a consideration that weighs against that conclusion. *. %he first is a 2udgment advanced in support of the conclusion of the argument& the second is that conclusion.

#nswer+ * E planation+ It is clear that the second bold part is the conclusion. 8"ome of the fragments were large and they penetrated8. %his is the reason behind posting this whole argument. "o on this basis& we can disregard choices # and '. Now, first part is serving as a premise for this conclusion. %hese fragments did not have sulfur therefore even though these fragments are present in outer cloud, they cannot provide sulfur. >owever, sulfur is present in outer part. "o this sentence is a premise that 8=ragments did not have sulfur8. #nd this sentence is supporting the conclusion that sulfur came from inner cloud. "o ' and ( can be disregarded. "o * should be the answer. * says that first part is supporting conclusion ,(orrect- and second part is the conclusion ,(orrect-.

1F..%wo computer companies, <arnet and Renco, each pay "alcor to provide health insurance for their employees. 'ecause early treatment of high cholesterol can prevent stro!es that would otherwise occur several years later, "alcor encourages <arnet employees to have their cholesterol levels tested and to obtain early treatment for high cholesterol. Renco employees generally remain with Renco only for a few years, however. %herefore, "alcor lac!s any financial incentive to provide similar encouragement to Renco employees.

/hich of the following, if true, most seriously wea!ens the argument$ #. *arly treatment of high cholesterol does not eliminate the possibility of a stro!e later in life. '. 1eople often obtain early treatment for high cholesterol on their own. (. <arnet hires a significant number of former employees of Renco. ). Renco and <arnet have appro6imately the same number of employees. *. Renco employees are not, on average, significantly younger than <arnet employees.

#nswer+ ( E planation+ %he conclusion is that +alcor lacks any financial incentive to provide similar encoura!ement to :enco employees. ;ou should !eep the scope of financial incentive in mind. :ption ( is the correct answer choice deals with this problem. If <arnet hires a significant number of former employees of Renco, then "alcor will have a financial incentive because <arnet employees from Renco will be less li!ely to suffer stro!es.

1F5.Oernland imposes a high tariff on the e6port of unprocessed cashew nuts in order to ensure that the nuts are sold to domestic processing plants. If the tariff was lifted and unprocessed cashews were sold at world mar!et prices, more farmers could profit by growing cashews. >owever, since all the processing plants are in urban areas, removing the tariff would seriously hamper the government9s effort to reduce urban unemployment over the ne6t five years. /hich of the following, if true, most seriously wea!ens the argument$ #- "ome of the byproducts of processing cashews are used for manufacturing paints and plastics. '- :ther countries in which cashews are processed subsidize their processing plants. (- 3ore people in Oernland are engaged in farming cashews than in processing them. )- 'uying unprocessed cashews at lower than world mar!et prices enables cashew processors in Oernland to sell processed nuts at competitive prices *- # lac! of profitable crops is driving an increasing number of small farmers in Oernland off their land and into the cities.

#nswer+ * E planation+ # reduced diagram+ O+ ,up- tariff cashew e6ports 000J sold to domestic plants If no tariff 0000J more farmers get U '@% plants in cities, so no tariff <<<<= hurt gov effort to ,down- unemployment ,#O# we need the plants to stay openNotice that we must wea!en the conclusion, which is the cause and effect relationship in bold above. #t this point, notice that the conclusion is immediately following the word 8'@%.8 "o, strengthen the preceding idea, and as a result you can wea!en the (. * is right because it shows us that without good crops to grow for profit, poor farmers will move to the city. /ell, the tariff, if removed, would allow those poor farmers to ma!e money growing cashews. #s a result, they wouldn9t need to move to the city to find wor!. >ence, the unemployment rates in the city would not go up because of these new wor!ers. *verything in this argument is tied together. It is essential that you see important words li!e 8however8 and the relationships that these words create among various parts of the argument.

1F7. $omcorp +hippin! $lerk+ Last wee!, no shipments of building supplies were sent out on friday. %he five specially ordered shipments sent out last wee! were sent out on %hursday, and each of those specially ordered shipments consisted entirely of building supplies. =our shipments were sent to %rua6 (onstruction last wee!, none of which consisted of building supplies. If the shipping cler!9s statements are true, which of the following must also be true$ a. #ll of (omcorp9s shipments of building supplies last wee! were specially ordered. b. None of (omcorp9s shipments sent on friday of last wee! was sent to %rua6 (onstruction. c. None of the shipments sent by (omcorp by %rua6 (onstruction last wee! was specially ordered. d. None of (omcorp9s shipments sent on thursday of last wee! was sent to %rua6 (onstruction. e. #ll of (omcorp9s shipments of building supplies last wee! were sent out on thursday.

#nswer+ ( E planation+ (. %he five specially ordered shipments sent out last wee! were sent out on %hursday, and each of those specially ordered shipments consisted entirely of building supplies. Q"pecial :rder 0J building suppliesR #N) 8=our shipments were sent to %rua6 (onstruction last wee!, none of which consisted of building supplies.8 Q"hipments to %rua6 0J N:% building "uppliesR

%he above two statements tells us that %rua6 (onstruction did not get any building supplies last wee!, and all the special order were of building supplies. >ence we can conclude that "hipments sent by (omcorp to %rua6 (orp were N:% specially ordered. Q"pecial :rder 0J building suppliesR II QN:% building supplies 0J N:% special orderR therefore Qshipments to %rua6 0J N:% building "uppliesR 0J N:% "pecial order

1FA.In a certain wildlife par!, par! rangers are able to trac! the movements of many rhinoceroses because those animals wear radio collars. /hen, as often happens, a collar slips off, it is put bac! on. 1utting a collar on a rhinoceros involves immobilizing the animal by shooting it with a tranquilizer dart. =emale rhinoceroses that have been frequently recollared have significant lower fertility rate than uncollared females. 1robably, therefore, some substances in the tranquilizer inhibit fertility. In evaluating the argument, it would be most useful to determine which of the following$ a. /hether there are more collared female rhinoceroses than uncollared female rhinoceroses in the par!. b. >ow the tranquilizer that is used for immobilizing rhinoceroses differs, if at all, from tranquilizers used in wor!ing with other large mammals. c. >ow often par! rangers need to use tranquilizer dart to immobilize rhinoceroses for reasons other than attaching radio collars. d. /hether male rhinoceroses in the wild par! lose their collar any more often than the par!9s female rhinoceroses do e. /hether radio collar is the only practical means that par! rangers have for trac!ing the movements of rhinoceroses in the par!.

#nswer+ ( E planation+ the best way to 82ustify8 the answer here is to eliminate the other answers. %his is more straightforward than on many other problems, because #LL of the wrong answers are H*R; much outside the argument9s scope. ,a- Irrelevant, as the numbers of collared vs. uncollared rhinos are irrelevant to fertility rates ,presumably measured in babies per rhino, or V of copulations required per pregnancy, or some other figure that doesn9t have anything to do with the total population size-. ,b- Irrelevant& the argument deals only with rhinos. ,d- Irrelevant& the argument deals only with =*3#L* rhinos. ,e- Irrelevant& the purpose of the collar doesn9t affect the fertility issue. 3oreover, other means of trac!ing the rhinos lie outside the scope of the argument. 00 %hat leaves ,c-. %he reason ,c- matters is because the study purports to cover the differences between rhinos that have been hit with tranquilizer darts ,let9s call them 8tran!s8- and those that haven9t. >owever, the study ):*"N9% directly split the rhinos into 8tran!8 and 8non0tran!8 groups& it splits them into 8frequently recollared8 and 8not frequently recollared8 groups. *he ar!ument therefore depends on the assumption that 3frequently recollared3 is an adequate pro y for 3been hit by tranks3 and that 3not frequently recollared3 is an adequate pro y for 3not been hit by tranks3" (hoice ,c- is very much relevant to this assumption, because that association falls apart if the rhinos are getting tran!ed for lots of other reasons in addition to the collar issue.

1FB."mithtown @niversity9s fund0raisers succeeded in getting donations from DG percent of the potential donors they contacted. %his success rate, e6ceptionally high for university fund0raisers, does not indicate that they were doing a good 2ob. :n the contrary, since the people most li!ely to donate are those who have donated in the past, good fundraisers constantly try less0li!ely prospects in an effort to e6pand the donor base. %he high success rate shows insufficient canvassing effort. /hich of the following, if true, provides more support for the argument$ #. "mithtown @niversity9s fund0raisers were successful in their contacts with potential donors who had never given before about as frequently as were fundraisers for other universities in their contacts with such people. '. %his year the average size of the donations to "mithtown @niversity from new donors when the university9s fund0raisers had contacted was larger than the average size of donations from donors who had given to the university before. (. %his year most of the donations that came to "mithtown @niversity from people who had previously donated to it were made without the university9s fund0raisers having made any contact with the donors. ). %he ma2ority of the donations that fund0raisers succeeded in getting for "mithtown @niversity this year were from donors who had never given to the university before. *. 3ore than half of the money raised by "mithtown @niversity9s fund0raisers came from donors who had never previously donated to the university.

#nswer+ # E planation+ (onclusion+ fund raiser contacted 1GG donor and DG gave donations still the effort was not good. =act+

1- ppl who have donated in past are more li!ely to donate currently. .-<ood fund raiser constantly tried to e6pand the donor base. >ow conclusion is lin!ed to =acts$ =und raiser tried to contact the previous donor instead of contacting new donors, so it needs less effort...something in this category. #- >mm, the language is quite tric!y here, but as per my understanding its saying+ =und raiser had contacts with . type of donators+ 1- donors who had N*H*R give =R*?@*N% ):N#%I:N" .- )onors who had given frequent )onations. %hey were successful with first type of donors. It tal!s about effort of fund raisers and also give us the possibility that they get donation from less frequent donors not from new donors. "o the effort was not sufficient. #lthough very vague but i thin! this is closed to answer. '- again wea!ening the argument (- "aying that ma6imum donation come out from donor without contacting, so it doesn9t say anything about fund raiser effort. )- /ea!ening *- /ea!ening

1FC.In general, 2obs are harder to get in times of economic recession because many businesses cut bac! operations. >owever, any future recessions in Hargonia will probably not reduce the availability of teaching 2obs at government0funded schools. %his is because Hargonia has 2ust introduced a legal requirement that education in government0funded schools be available, free of charge, to all Hargonian children regardless of the state of the economy, and that current student0teacher ratios not be e6ceeded.

/hich of the following, if true, most strengthens the argument$ #. %he current student0teacher ratio at Hargonia9s government0funded schools is higher than it was during the most recent period of economic recession. '. )uring recent periods when the Hargonian economy has been strong, almost .A percent of Hargonian children have attended privately funded schools, many of which charge substantial fees. (. Nearly .G percent more teachers are currently employed in Hargonia9s government0funded schools than had been employed in those schools in the period before the last economic recession. ). %eachers in Hargonia9s government0funded schools are well paid relative to teachers in most privately funded schools in Hargonia, many of which rely heavily on part0time teachers. *. )uring the last economic recession in Hargonia, the government permanently closed a number of the schools that it had funded

#nswer+ ' E planation+ (onclusion+ #ny further recession will not reduce #H#IL#'ILI%; of %*#(>IN< N:'. /hy$ 'ecause of legal requirement+ free school to #LL H (>IL)R*N ,gov funded- W "0% ration will not e6ceed. "o, >ow the #vailability of %eaching 2ob is related to =acts given. 'y "0% ratio, the availability of teaching 2ob can be reduced only if no of student reduced.

:ption #-+ #lways remember. <3#% always tries to confuse you by giving different time period data. If in the main passage we are not tal!ing about different time period then, why do we need now$ # is irrelevant. :ption '-+ .AT children have attended private school recently, so this give us a probability that these children may attend gov0funded school, so instead of decrease, they will help to increase the number of

student. "o it strengthens the argument. :ption (-+ "aying what is number of teachers now as compare to previous recession. ,are we tal!ing about two economic recession or something li!e that- $$ :ption )-+ %his gives us why a teacher should 2oin <ov funded school. :ption *-+ #gain last economic recession, this is Irrelevant. If you see, only :ption ' and ) are tal!ing in same time period as of main passage. ) is tal!ing in some other direction. "o clearly ' is answer here.

1FD.In response to mounting public concern, an airplane manufacturer implemented a program with the well0 publicized goal of reducing by half the total yearly amount of hazardous waste generated by its passenger0 2et division. /hen the program began in 1FF7, the division9s hazardous waste output was FG pounds per production wor!er& last year it was 7G pounds per production wor!er. (learly, therefore, charges that the manufacturer9s program has not met its goal are false. /hich of the following is an assumption on which the argument depends$ #. %he amount of nonhazardous waste generated each year by the passenger02et division has not increased significantly since 1FF7. '. #t least as many passenger 2ets were produced by the division last year as had been produced in 1FF7. (. "ince 1FF7, other divisions in the company have achieved reductions in hazardous waste output that are at least equal to that achieved in the passenger02et division. ). %he average number of wee!ly hours per production wor!er in the passenger02et division was not significantly greater last year than it was in 1FF7. *. %he number of production wor!ers assigned to the passenger02et division was not significantly less in 1FF7 than it was last year.

#nswer+ * E planation+ the production company9s <:#L was to lower the %:%#L #3:@N% of hazardous waste produced. %he *HI)*N(* is stated in terms of the amount 1*R /:RO*R, not the total amount. %his is everything. %herefore, if we can find an answer choice that (:NN*(%" these two concepts ,the total amount of hazardous waste and the amount of hazardous waste per wor!er-, then that9s the correct answer. %his is precisely what answer choice ,e- does. 00 answer choice ,b- is irrelevant, as we have no information about how much waste is produced 1*R N*% 0 not now, not then, never.

1FF.*lectronic computer chips made of tiny silicon wafers now regularly contain millions of electronic switches. @nfortunately, electronic switches that are this small cannot withstand intense radiation. 3icro03echanics plans to produce a chip that, because it uses only microscopic mechanical switches, will be invulnerable to radiation damage. %he switches will, however, be slower than electronic switches and the chip will contain only 1.,GGG switches. =or there to be a mar!et for 3icro03echanics chip as a result of the apparent advantage described above, each of the following would have to be true *E(*1%+ #. %here will be applications in which the speed attainable by an electronic switch is not essential. '. "witches used on electronic chips that contain only 1.,GGG switches are more vulnerable to radiation damage than the switches on 3icro03echanics chip will be. (. %here will be applications for computer chips in environments where the chips may have to survive

intense radiation. ). "ome devices in which computer chips will be used will have other components that will be able to function during or after e6posure to radiation. *. 3anufacturers are able to protect electronic computer chips against e6posure to intense radiation, where this protection is necessary.

#nswer+ * E planation+ usually, when you have an 8e6cept8 problem li!e this one, you9ll have 7 answers that wor!, and 1 that is irrelevant ,usually for some subtle reason-. >ere, though, you have 7 answers that wor!, and 1 that accomplishes e6actly the opposite of what you9re supposed to accomplish. 00 you9re loo!ing for the one choice that is N:% IN =#H:R := %>* N*/ (>I1". %his would be ,e-. If ,e- is true, then the primary )I"#)H#N%#<* of the :%>*R chips 0 the ones that are faster and have more switches 0 is R*3:H*). %his is e6tremely detrimental to the new chips.

.GG. Editor+ #rticles in <ardening 3agazine often spur sales of the plants they describe, particularly among people new to gardening. #ccordingly, we will no longer publish articles or accept advertisements praising the beauty of rare wildflowers. 3ost such plants sold to gardeners have been difficult to propagate under cultivation, so plant sellers often collect them in the wild. :ur new policy is part of our efforts to half this yearly plundering of our native plant populations. /hich of the following, if true, casts the most doubt on the wisdom of the magazine9s new policy as a way of pursuing the intended effect$ ,#- /hen people new to gardening buy plants, they often fail to ta!e adequate care of the plants that they buy and become discouraged from buying those varieties again. ,'- 1lant sellers who sell rare wildflowers have no reasonably ine6pensive alternate way to offer their wares directly to new gardens. ,(- %he demand for rare wildflowers rarely e6ceeds the number of such plants that can be collected in the wild by plant sellers. ,)- %he propagation of rare wildflowers often depends on the plant9s interaction with other organisms in their environment such as plants that create suitable soil conditions or insects and birds that disperse seeds. ,*- Revenues from sales of plants collected in the wild are supporting the discovery of new low0cost techniques enabling rare wildflowers to be readily propagated in nurseries.

#nswer+ * E planation+ #rgument is+ magazine /ont publish article on wildflower 00000J %his act will save wildflowers /hat if publishing articles on wildflowers helps in saving them $ If this is true, argument is wea!ened. ,#s intended effect can be achieved without the stated action-. %his is e6actly what * is saying.

.G1.%he cotton farms of (ountry ? became so productive that the mar!et could not absorb all that they produced. (onsequently, cotton prices fell. %he government tried to boost cotton prices by offering farmers who too! .A percent of their cotton acreage out of production direct support payments up to a specified ma6imum per farm. %he government9s program, if successful, will not be a net burden on the budget. /hich of the following, if

true, is the best basis for an e6planation of how this could be so$ ,#- )epressed cotton prices meant operating losses for cotton farms, and the government lost revenue from ta6es on farm profits. ,'- (otton production in several counties other than ? declined slightly the year that the support0payment program went into effect in ?. ,(- %he first year that the support0payment program was in effect, cotton acreage in ? was AT below its level in the base year for the program. ,)- %he specified ma6imum per farm meant that for very large cotton farms the support payments were less per acre for those acres that were withdrawn from production than they were for smaller farms. ,*- =armers who wished to qualify for support payments could not use the cotton acreage that was withdrawn from production to grow any other crop.

#nswer+ # E planation+ 1araphrasing+ <ov will fund the farmers who will produce only on CAT of their land to boost the cotton prices. =act+ 1- cotton production is very high so cotton prices falling. /e have to balance this+ 'udget 0 fund II 'udget W $$ *ither we can prove fund 00J G or some other factor might be added to budget, so that it will not be a burden on govt. # is much stronger e6planation than ).

If program is not to be a net burden on the budget, <ovt might be getting something which nullifies the effect of direct support payment. # e6plains this by stating that ta6 offsets the money given towards direct support payment.

.G..In the year following an eight0cent increase in the federal ta6 on a pac! of cigarettes, sales of cigarettes fell ten percent. In contrast, in the year prior to the ta6 increase, sales had fallen one percent. %he volume of cigarette sales is therefore strongly related to the after0ta6 price of a pac! of cigarettes. /hich of the following, if true, could most strengthen the argument above$ ,#- )uring the second year after the ta6 increase, cigarette sales increased by a significant amount. ,'- %he information available to consumers on the health ris!s of smo!ing remained largely unchanged in the period before and after the ta6 increase. ,(- 3ost consumers were unaware that the ta6 on cigarettes was going to increase. ,)- )uring the year following the cigarette ta6 increase, many consumers had less income, in inflation0 ad2usted dollars, than they had had in the previous year. ,*- )uring the year after the ta6 increase, there was a greater variety of cigarettes on the mar!ey than there had been during the previous year.

#nswer+ ' E planation+ %his is an absolutely classic type of problem+ it (:N=L#%*" (:RR*L#%I:N /I%> (#@"#%I:N. it ta!es a statistical correlation between cigarette ta6 and cigarette consumption, and postulates that one has a (#@"#L effect on the other. ,here, the ta6 is ta!en to lead to decreased consumption.-

#nythin! disruptin! the $#G+#L relationship between ci!arette ta and ci!arette consumption <<= i"e", any #L*E:N#*'FE ED%L#N#*'-N .-: 0HI *HE *0- #:E $-::EL#*E? 00J will ruin the argument. %herefore, you can "%R*N<%>*N the argument by )I"1*N"IN< with such e6planations. %his is what choice ,b- does. :ne possible alternative e6planation is that consumers may have become more educated about the dangers of cigarettes, leading them to smo!e fewer cigarettes regardless of the ta6. %his choice eliminates that possibility. 'n !eneral, +*#*E&EN*+ -. $#G+E #N? E..E$* are stron!er if -*HE: F#:'#5LE+ #:E 3$-N*:-LLE?3 00 i.e., if N: other variables, apart from the variables under control, are allowed to change. (hoice ,b- !eeps one such variable ,the amount of information available to consumers- controlled.

In the case of choice ,e-, it9s 2ust as plausible that the increased variety of cigarettes caused the decrease in sales, for a variety of reasons ,perhaps consumers were confused or turned off-. %his is 2ust as plausible as your scenario. %he only way to phrase choice ,e- so that it strengthens the argument would be to say that there is the same degree of variety before and after the ta6 increase. i.e., e6perimental control.

.G5.No nation in the world has e6perienced as significant a decline in its ;ucaipa tree population as our nation. ;et only our nation imposes a law prohibiting the use of ;ucaipa tree0bar! oil in cosmetics. %he purpose of this law in the first place was to help maintain the ;ucaipa tree population, at least in this nation. 'ut the law is clearly unnecessary and therefore should be repealed. /hich of the following, if true, would most seriously wea!en the conclusion drawn in the passage$ a. %his nation contains more ;ucaipa trees than any other nation. b. ;ucaipa tree0bar! oil is not used for any consumer goods other than cosmetics. c. %he demand for cosmetics containing ;ucaipa tree0bar! oil is e6pected to decline in the future in other nations while continuing unabated in this nation. d. In other countries, labor used to harvest ;ucaipa trees for cosmetics is less e6pensive than comparable labor in this nation. e. In this nation, some wild animals eat ;ucaipa tree bar!, thereby contributing to their destruction.

#nswer+ ( E planation+ /e are told that the demand for this product will continue @N#'#%*) 0 meaning that there is a demand. %he argument is relatively wea! to begin with. /e are told that the law is unnecessary, based only on the evidence that 1- the law is meant to help protect the trees, and .- the trees are still being lost. %he arguer assumes that this means that the law has no effect. It9s possible, though, that the trees would be lost even 3:R* if not for the law. /e merely need to show that the law does have some effect. ( does this by showing that there is a demand for the trees 0 thus, the law that prevents this type of use ought to help protect them 0 thus wea!ening the argument. 00000 %o repeal a law is to get rid of it 0 to cancel it from the boo!s. %he #R<@3*N% is that the law should be done away with. %o /*#O*N this argument, you should show that the law serves a useful purpose" ;ou are arguing here that 8the law doesn9t do much8. If that9s the case, then you are doing precisely the opposite of what you must do to wea!en the argument.

3a!e sure you !eep these sorts of things straightS It can be difficult at times& you may want to 2ot down notes while you9re doing the problem. .G7. Hunter+ >unters alone are blamed for the decline in <reenroc! National =orest9s deer population over the past ten years. Iet clearly, black bears have also played an important role in this decline . In the past ten years, the forest9s protected blac! bear population has risen sharply, and e6amination of blac! bears found dead in the forest during the deer hunting season showed that a number of them had recently fed on deer. In the hunter9s argument, the boldface portion plays which of the following roles$ #. It is the main conclusion of the argument. '. It is an ob2ection that has been raised against the main conclusion of the argument. (. It is a 2udgment that the argument opposes. ). It is a finding that the argument see!s to e6plain *. It is a biased observation of the hunter

#nswer+ E planation+ 1- ;et clearly, blac! bears have also played an important role in this decline or .- 'oth hunters and blac! bears caused the decline of the deer population If the above is true then # could be correct because of ,1- above. ' is not possible because boldface can only either be %>* (:N(LI"I:N in the case of ,1- above, or in support of the conclusion in the case of ,.- above. ( is wrong for the same reason ) is wrong because the boldface is a claim and not a finding. Rather, the evidence provided, 8In the past ten years, the forest9s protected blac! bear population has risen sharply, and e6amination of blac! bears found dead in the forest during the deer hunting season showed that a number of them had recently fed on deer,8 could be the finding to support the claim in boldface. * is wrong because it is a claim and not an observation 0000 ;et, >owever, 'ut ...can all introduce conclusions. #lthough, )espite, 'ecause.... all introduce premises.

.GA.3ost employees in the computer industry move from company to company, changing 2obs several times in their careers. >owever, "ummit (omputers is !nown throughout the industry for retaining its employees. "ummit credits its success in retaining employees to its informal, nonhierarchical wor! environment. /hich of the following, if true, most strongly supports "ummit s e6planation of its success in retaining employees$ ,#- "ome people employed in the computer industry change 2obs if they become bored with their current pro2ects. ,'- # hierarchical wor! environment hinders the cooperative e6change of ideas that computer industry employees consider necessary for their wor!. ,(- 3any of "ummit s senior employees had previously wor!ed at only one other computer company. ,)- In a nonhierarchical wor! environment, people avoid behavior that might threaten group harmony and thus avoid discussing with their colleagues any dissatisfaction they might have with their 2obs. ,*- %he cost of living near "ummit is relatively low compared to areas in which some other computer companies are located

#nswer+ #

E planation+ this passage is arguing that a non0hierarchical environment is better than a hierarchical one. ;ou can strengthen such a claim in either of . ways+ \ show something better about the N:N0hierarchical environment, :R \ show something worse about the hierarchical environment. %he official answer does the latter of these.

.GB. %aleontolo!ist+ #bout ..D million years ago, many species that lived near the ocean floor suffered substantial population declines. %hese declines coincided with the onset of an ice age. %he notion that cold !illed those bottom0dwelling creatures outright is misguided, however& temperatures near the ocean floor would have changed very little. Nevertheless, the cold probably did cause the population declines, thou!h indirectly. 3any bottom0dwellers depended for food on plan!ton, small organisms that lived close to the surface and san! to the bottom when they died. &ost probably, the plankton suffered a severe population decline as a result of sharply lower temperatures at the surface, depriving many bottom dwellers of food. In the paleontologist9s reasoning, the two portions in boldface play which of the following roles$ #. %he first introduces the hypothesis proposed by the paleontologist& the second is a 2udgment offered in spelling out that hypothesis. '. %he first introduces the hypothesis proposed by the paleontologist& the second is a position that the paleontologist opposes. (. %he first is an e6planation challenged by the paleontologist& the second is an e6planation proposed by the paleontologist ). %he first is a 2udgment advanced in support of a conclusion reached by the paleontologist& the second is that conclusion *. %he first is a generalization put forward by the paleontologist& the second presents certain e6ceptional cases in which that generalization does not hold good #nswer+ # E planation+ In 3:"% 8boldface8 questions, you will be able to answer the question by doing the following+ \ identify the conclusion of the argument ,using a diagram if necessary\ then, for each boldface, tell whether it+ 0 '+ the conclusion 0 +G%%-:*+ the conclusion 0 -%%-+E+ the conclusion in this problem, that9s all you have to do. If you understand the argument at hand, you9ll be able to identify that its conclusion is 8...the cold probably did cause the population declines, though indirectly.8 %herefore, the first bold I" the conclusion. %he second is definitely on the side of the conclusion ,i.e., "@11:R%" the conclusion-, since it provides a reason why the first may be true. %he only answer choice that correctly identifies the first as the conclusion and the second as supporting the conclusion is ,a-.

.GC.In 'erinia, the age at which people could begin to drin! alcohol legally used to be 1D. In 1FFG, in an attempt to reduce alcohol consumption and thereby to reduce alcohol related traffic deaths among 'erinians under .1, the legal drin!ing age was raised to .1. #lcohol0related traffic deaths among people under .1 have decreased significantly since 1FFG. Nevertheless, surveys show that people in that age0group drin! 2ust as much alcohol as they did before 1FFG. /hich of the following, if true of 'erinia, most helps to resolve the apparent discrepancy$ #. =or the population as a whole, annual alcohol consumption is no lower now than it was in 1FFG.

'. #lcohol consumption away from home, for e6ample in bars and restaurants, is much lower among people under .1 than it was in 1FFG. (. %he proportion of people under .1 who own a car is higher now than it was in 1FFG. ). #lcohol consumption is lower among people under .1 than among adults in most other age0groups. *. #lcohol0related traffic deaths among people over .1 have increased slightly since 1FFG.

#nswer+ ' E planation+ *akeaway:< whenever a problem statement references some va!ue term, such as 3the !oal3, 3the discrepancy3, 3the parado 3, 3the conclusion3, etc", you must specify ED#$*LI what this is, 'N #+ &G$H ?E*#'L #+ I-G $#N" If you do this, then it should be much easier to answer the question. 00 in this case, there9s a 8discrepancy8, which means that there are two statements that appear contradictory, but which actually aren9t contradictory. %he main challenge of the problem is to locate those two statements and to pic! a statement that best e6plains how they can both be true at the same time. %he two seemingly contradictory statements are+0 ,1- people under .1 drin! 2ust as much as they used to. ,.- %hese same people aren9t getting into alcohol0related traffic deaths as often as they used to. (hoice ,b- e6plains how these can both be true+ they9re still drin!ing 2ust as much ... but not to places that are away from home. %herefore, not as much need to drive. 000 =act 1+ the age at which people could begin to drin! alcohol legally used to be 1D

fact .+ In 1FFG, in an attempt to reduce alcohol consumption and thereby to reduce alcohol related traffic deaths among 'erinians under .1, the legal drin!ing age was raised to .1 the author here assumes that less alcohol reduces traffic accidents fact 5+ #lcohol0related traffic deaths among people under .1 have decreased significantly since 1FFG. %his is what was e6pected. fact7+ Nevertheless, surveys show that people in that age0group drin! 2ust as much alcohol as they did before 1FFG. >ere comes the contradiction. 1eople under .1 drin! more now but #lcohol0related traffic deaths among people under .1 have decreased.

#. /e are interested in people under .1 rather than all people. *liminate. (. %his answer choice says that there are more drivers under .1 and fact7 says that people under .1 drin! now the same, so this cannot e6plain why now the number of deaths has decreased. *liminate. ). #gain this comparison is out of scope. It doesn9t help to solve the parado6 *. the group of people is out of scope. Remember that our group is under .1

.GD.%rue"ave is a mail0order company that ships electronic products from its warehouses to customers worldwide. %he company s shipping manager is proposing that customer orders be pac!ed with newer, more e6pensive pac!ing materials that virtually eliminate damage during shipping. %he manager argues that overall costs would essentially remain unaffected, since the e6tra cost of the new pac!ing materials roughly

equals the current cost of replacing products returned by customers because they arrived in damaged condition. /hich of the following would it be most important to ascertain in determining whether implementing the shipping manager s proposal would have the argued0for effect on costs$ #. /hether the products shipped by %rue"ave are more vulnerable to incurring damage during shipping than are typical electronic products '. /hether electronic products are damaged more frequently in transit than are most other products shipped by mail0order companies (. /hether a sizable proportion of returned items are returned because of damage already present when those items were pac!ed for shipping ). /hether there are cases in which customers blame themselves for product damage that, though present on arrival of the product, is not discovered until later *. /hether %rue"ave continually monitors the performance of the shipping companies it uses to ship products to its customers

#nswer+ ( E planation+ *akeaway:< if a problem asks 3which of the followin! would be important to know 9 ascertain 9 etc"3, then this is usually equivalent to 3find the assumption3" %his is the case because the 8things that we have to ascertain8 usually have to do with whether the situation will satisfy our assumptions. %his is no different+ the manager states that the added cost of the new fancy pac!ing materials will ma!e up for no more damage claims. %he assumption is that I= we use the advanced pac!aging, then there /ILL '* no more damage claims.

( is correct. /e have to evaluate whether the proposal of the manager would be beneficial. >is proposal is based on the assumption that many products got damaged during the transit and therefore using the better quality pac!ing material we can avoid that damage. /e basically need to evaluate this assumption. Now ( questions that assumption in a fancier way. If a sizable proportion of returned items were damaged not during transit but before pac!ing itself, then new better quality pac!ing cannot do anything to avoid this damage, hence manager9s assumption fall apart.

.GF./hich of the following most logically completes the passage$ :n the whole, scientist do their most creative wor! before age of forty, a tendency that has been ta!en to show that ages carries with it a loss of creative capacity. #n alternate e6planation is that by age forty most scientist have wor!ed in their field for fifteen or more years and that by then they have e6hausted the opportunity for creative wor! in that field. "upporting this e6planation is the finding that444444. #- the average age of recipients of scientific research grants is significantly greater than forty '- a disproportionately large number of the scientist who produce highly creative wor! beyond age forty entered their field at an older age than is common (- many scientist temper their own e6pectations of what they can achieve in their research wor! by their belief that their creativity will decline as they age )- a scientist who are older than forty tend to find more satisfaction in other activities, such as teaching and mentoring, than they do in pursuing their own research

*- there is a similar diminution of creativity with age in nonscientific fields, such as poetry and musical composition

#nswer+ ' E planation+ (onclusion+ "cientists9 (reativity decreases with %ime. In order to support this conclusion #uthor says K3ost scientist by the age of forty have 1A years of e6perience that e6haust the opportunity of creative wor! in the field8. /e have to find something that supports #uthor "aying. a- Irrelevant. b- "cientist who produce creative wor! after the age of forty, entered in the field lately. 00J 3eans by the age of forty, they have not finished the 1A year of e6perience, so they still find opportunities. %his is supporting #uthor. c- "ays scientists do their wor! with a belief that their creativity will decrease with age. 'ut this has nothing to do with #uthor saying. d- "cientist after the age of 7G, find more satisfaction in other activities than research& again if they are not satisfied, this doesn9t mean there is no opportunity for creative wor!. A- Irrelevant, %al!ing about the other fields.

.1G.%he prairie vole, a small North #merican grassland rodent, breeds year0round, and a group of voles living together consists primarily of an e6tended family, often including two or more litters. Holes commonly live in large groups from late autumn through winter& from spring through early autumn, however, most voles live in far smaller groups. %he seasonal variation in group size can probably be e6plained by a seasonal variation in mortality among young voles. /hich of the following, if true, provides the strongest support for the e6planation offered$ #. it is in the spring and early summer that prairie vole communities generally contain the highest proportion of young voles. '. prairie vole populations vary dramatically in size from year to year (. the prairie vole subsists primarily on broad0leaved plants that are abundant only in spring. ). winters in the prairie voles9 habitat are often harsh, with temperatures that drop well below freezing. *. sna!es, a ma2or predator of young prairie voles, are active only from spring through early autumn.

#nswer+ * E planation+ ,a- is the worst answer choice, because it wea!ens the argument. %he conclusion says+ %he seasonal variation in group size can probably be e6plained by a seasonal variation in mortality among young voles. %his is crazy0tal! for 8the groups are smaller in spring and summer because lots of young voles die, for some reason, in the spring and summer.8 If ,a- is true, though, then there is a higher T of young voles during the spring and summer, a statistic that9s in direct contradiction to the thesis of the passage. Not good. 00 ,e- strengthens the correlation by not only providing a reason for the differential in sizes of vole families, but also showing specifically that it9s the young voles that are !illed ,as required by the conclusion-.

.11.%he growing popularity of computer0based activities was widely e6pected to result in a decline in television viewing, since it had been assumed that people lac! sufficient free time to maintain current television0 viewing levels while spending increasing amounts of free time on the computer. %hat assumption, however, is evidently false+ in a recent mail survey concerning media use, a very large ma2ority of respondents who report increasing time spent per wee! using computers report no charge in time spent watching television. /hich of the following would it be most useful to determine in order to evaluate the argument$ #. /hether a large ma2ority of the survey respondents reported watching television regularly '. /hether the amount of time spent watching television is declining among people who report that they rarely or never use computers (. /hether the type of television programs a person watches tends to change as the amount of time spent per wee! using computers increases ). /hether a large ma2ority of the computer owners in the survey reported spending increasing amounts of time per wee! using computers *. /hether the survey respondents reports of time spent using computers included time spent using computers at wor!

#nswer+ * E planation+ first, let9s ma!e sure that we evaluate the question prompt correctly& it tal!s about 8evaluating the argument8. @sually, 8evaluate8 means 8strengthen M wea!en8. /e can9t tell which, but the two are opposite sides of the same coin. ,i.e., if E would strengthen, then not0E will usually wea!en, and vice versa-. :ne reliable way to stren!then 9 weaken an ar!ument is to bolster or sever the (:NN*(%I:N" '*%/**N )I==*R*N% "1*(I=I(". in this case, there are two specifics that are treated as if they were the same, even though they9re totally different+ \ =R** time spent on the computer ,mentioned in the hypothesis of the studyand \ %:%#L time spent on the computer ,reported in the resultsnote that you have to be pretty astute, and paying pretty close attention, to figure this out, since the passage doesn9t actually use the word 8total8. (hoice ,e- correctly points to the issue of this connection ,or lac! thereof-. If the answer to this question is 8yes, the time includes computer time at wor!8, then the connection is severed, and the study9s conclusion doesn9t mean anything. If the answer is the opposite, then the study has indeed found a surprising result.

.1../hich of the following most logically completes the argument$ # new machine for harvesting corn will allow rows to be planted only fifteen inches apart, instead of the usual thirty inches. (orn planted this closely will produce lower yields per plant. Nevertheless, the new machine will allow corn growers to double their profits per acre because 4444444444. ,#- with the closer spacing of the rows, the growing corn plants will quic!ly form a dense canopy of leaves, which will, by shading the ground, minimize the need for costly weed control and irrigation ,'- with the closer spacing of the rows, corn plants will be forced to grow taller because of increased competition for sunlight from neighboring corn plants ,(- with the larger number of plants growing per acre, more fertilizer will be required ,)- with the spacing between rows cut by half, the number of plants grown per acre will almost double ,*- with the closer spacing of the rows, the acreage on which corn is planted will be utilized much more intensively than it was before, requiring more frequent fallow years in which corn fields are left unplanted

#nswer+ # E planation+

) is wrong because it does not ta!e the argument into account and the formula "ales P (osts I 1rofit` %he argument is Kthe new machine will allow corn growers to double their profits per acreL. In general, arguments that discuss profits need to be addressed with the formula of "ales P (osts I 1rofits. #nswer choice # attac!s the KcostL part of the formula. If the farmer lowers the costs, profits will riseS #ll answer choice ) really says is that the number of plants will double. ;ou cannot assume that sales will double with costs only rising marginally to create double the amount of profits. %his might not be the best advice but you can assume ) from the statement P if the spacing is cut in half, the number of plants must double.

#lso, note that choice ,d- contains something that is actually self0evident from the information that9s already in the passage. In other words, if you cut the spacing between rows by 1M., then by definition you9re going to be able to plant about twice as many plants. %herefore, ,d- is simply a repetition of what9s already in the argument, and adds virtually nothing. .15.In parts of the (aribbean, the manatee, an endangered marine mammal, has long been hunted for its meat. >aving noted the manatee hunters e6pert !nowledge of manatees habits, local conservationists are encouraging the hunters to stop hunting and instead to ta!e tourists on boat rides to see manatees. %ourist interest is high, so the plan has promise of achieving the twin goals of giving the former hunters a good income and helping ensure the manatees survival. /hich of the following, if true, raises the most serious doubt about the plan s chance of success$ #. 3any tourists who visit these parts of the (aribbean are uninterested in manatees and would not be willing to pay what the former manatee hunters would have to charge for boat rides to see manatees. '. Recovery of the species would enable some hunting to continue without putting the manatees survival in 2eopardy again. (. In areas where manatees have traditionally been hunted for food, local people could easily replace the manatee meat in their diets with other foods obtained from the sea. ). %here would not be enough former manatee hunters to act as guides for all the tourists who want to see manatees. *. %o maintain their current income, manatee hunters who switched to guiding tourists would have to use far larger boats and ma!e many more trips into the manatees fragile habitat than they currently do.

#nswer+ * E planation+ %he (:N(L@"I:N is 8the plan has promise of achieving the twin goals of giving the former hunters a good income and helping ensure the manatees survival8 therefore, we can /*#O*N %>* (:N(L@"I:N by showing either \ that the plan will N:% give the hunters a good income, or \ that the plan will act to *N)#N<*R the manatees9 survival. 00 the correct answer should be ,e-. If the hunters would have to 8would have to use far larger boats and ma!e many more trips into the manatees fragile habitat than they currently do8, then the manatees9 survival is being placed at increased ris!. Note especially the pointed use of the ad2ective 8fragile8. ,#- is irrelevant. It doesn9t matter that 3#N; tourists are uninterested andMor unwilling to pay for the tours, since the passage has already guaranteed us that 8tourist interest is high8. 3#N; 2ust mean that we can find a bunch of tourists who aren9t interested. %his doesn9t, at all, wea!en the conclusion that tourist interest is high. #nalogy+ if I determine that, in some area, local interest in eating cheeseburgers is high, then I should probably open a burger 2oint ,if there isn9t already one open- in that area. if I find 8many8 people in the area that don9t eat

cheeseburgers, that doesn9t impact my e6isting statement that local interest in eating the burgers is high 0 again, we don9t need everyone to be on board.

.17.%he pharmaceutical industry argues that because new drugs will not be developed unless heavy development costs can be recouped in later sales, the current .G years of protection provided by patents should be e6tended in the case of newly developed drugs. >owever, in other industries new0product development continues despite high development costs, a fact that indicates that the e6tension is unnecessary. /hich of the following, if true, most strongly supports the pharmaceutical industry9s argument against the challenge made above$ ,#- No industries other than the pharmaceutical industry have as!ed for an e6tension of the .G0year limit on patent protection. ,'- (linical trials of new drugs, which occur after the patent is granted and before the new drug can be mar!eted, often now ta!e as long as 1G years to complete. ,(- %here are several industries in which the ratio of research and development costs to revenues is higher than it is in the pharmaceutical industry. ,)- #n e6isting patent for a drug does not legally prevent pharmaceutical companies from bringing to mar!et alternative drugs, provided they are sufficiently dissimilar to the patented drug. ,*- 3uch recent industrial innovation has occurred in products000for e6ample, in the computer and electronics industries000for which patent protection is often very ineffective. #nswer+ ' E planation+ In ,b-, the only thing that we !now =:R "@R* is that the .G0year patent term is effectively only 1G years long, because the companies can9t ma!e any profits off their products until after the conclusion of the clinical trial period. %his fact, which means that the patent period is effectively half as long as it9s supposed to be, would clearly strengthen the industry9s argument for an e6tension of the patent period.

.1A./hen an airplane is ta!en out of service for maintenance, it is often repainted as well, and during the repainting no other maintenance wor! can be done on the plane. In order to reduce maintenance time, airline officials are considering using a new nonto6ic plastic film instead of paint. %he film ta!es 2ust as long to apply as paint does, but many other maintenance tas!s can be carried out at the same time. /hich of the following, if true, is further evidence that using the film will help the airline officials achieve their goal$ ,#- @nli!e paint, the film gives a mil!y tone to certain colors. ,'- #t the end of its useful life, the film can be removed much more quic!ly than paint can. ,(- %he film can be applied only by technicians who have received special training. ,)- %he metal e6teriors of airplanes have to be protected from high temperatures and caustic chemicals such as e6haust gases. ,*- *ven at speeds considerably higher than the normal speed of a passenger 2et, the film remains securely attached.

#nswer+ E planation+ if we ta!e it for granted that the processes of painting the plane and applying the film are parallel 0 i.e., both of them need to be removed 8at the end of their useful life8 0 then this choice represents an additional cost savings, because, prior to applying a new coat, they9ll be able to strip off the film faster than they9ll be able to strip off the paint.

*he other four choices clearly have N-*H'N, to do with time efficiency and faster maintenance . (hoice ,b- is the only one that9s even tangentially related to that idea, so it9s the winner almost by default. Remember, the question is not 8which one of these is ":LI) evidence$8 it9s 2ust 8which one is further evidence$8 %herefore, frustratingly enough, if one choice is wea! evidence, but all four of the other choices are clearly irrelevant, then the 8wea! evidence8 choice wins.

.1B.1olitical advocacy groups have begun to use information services to disseminate information that is then accessed by the public via personal computer. "ince many groups are thus able to bypass traditional news sources, whose reporting is selective, and to present their political views directly to the public, information services present a more balanced picture of the comple6ities of political issues than any traditional news source presents. /hich of the following is an assumption on which the argument above depends$ #. Information services are accessible to enough people to ensure that political advocacy groups can use these services to reach as large a percentage of the public as they could through traditional news sources. '. 1eople could get a thorough understanding of a particular political issue by sorting through information provided by several traditional news sources, each with differing editorial biases. (. Information on political issues disseminated through information services does not come almost entirely from advocacy groups that share a single bias. ). %raditional news sources seldom report the views of political advocacy groups accurately. *. 3ost people who get information on political issues from newspapers and other traditional news sources can readily identify the editorial biases of those sources.

#nswer+ ( E planation+ =or now, I9ll suggest that 8/hich of the following is an assumption on which the argument above depends$8 is stricter than 8which of the following supports the conclusion$8 %here may be several choices that support the conclusion, at least a little bit. Instead, you should translate this question as 8/hich of the following 3@"% be true if you are to have any hope of reaching this conclusion$8 or 8/hich of the following, if *LI3IN#%*), would !ill the conclusion$8 ,(- is correct because if information on political issues disseminated through information services /*R* almost entirely from advocacy groups that share a single bias, then you (:@L) N:% claim that information services present a more balanced picture of political issues than traditional news sources.

.1C.:ne of the limiting factors in human physical performance is the amount of o6ygen that is absorbed by the muscles from the bloodstream. #ccordingly, entrepreneurs have begun selling at gymnasiums and health club bottles of drin!ing water, labeled 8"uper:E;,8 that has e6tra o6ygen dissolved in the water. "uch water would be useless in improving physical performance, however, since the amount of o y!en in the blood who is e ercisin! already more than the muscles can absorb. /hich of the following, if true, would serve the same function in the argument as the statement in boldface$ #- world0class athletes turn in record performances without such water '- frequent physical e6ercise increases the body9s ability to ta!e in and use o6ygen (- the only way to get o6ygen into the bloodstream so that it can be absorbed by the muscles is through the lungs )- lac! of o6ygen is not the only factor limiting human physical performance *- the water lost in e6ercising can be replaced with ordinary tap water

#nswer+ ( E planation+

(. %he main function of the boldface is to highlight the fact that e6tra o6ygen consumed through water would be useless in improving performance. ,#- is irrelevant. %his has nothing whatsoever to do with whether the water will improve performance. #nalogy+ world0class athletes also turn in record performances without using steroids. %his fact clearly doesn9t prove that steroids are useless in improving athletic performances. ,'- is irrelevant. Not only does it have nothing to do with the ways in which water is absorbed, but the 8increase8 is not quantified at all. In fact, ironically, the ,very indirect- effect of ,b- is to contribute to the argument. If frequent e6ercise can increase o6ygen inta!e, then it9s possible that frequent e6ercisers might reach a point at which the e6tra o6ygen in the water would start helping them. ;ou shouldn9t thin! that much, though. %his is a very, very, very distant relationship, and correct answers are always directly related to the passage at hand. .1D.%he recent upheaval in the office0equipment retail business, in which many small firms have gone out of business, has been attributed to the advent of office equipment KsuperstoresL whose high sales volume !eeps their prices low. %his analysis is flawed, however, since even today the superstores control a very small share of the retail mar!et.

/hich of the following, if true, would most wea!en the argument that the analysis is flawed$ ,#- 3ost of the larger customers for office equipment purchase under contract directly from manufacturers and thus do not participate in the retail mar!et. ,'- %he superstores heavy advertising of their low prices has forced prices down throughout the retail mar!et for office supplies. ,(- "ome of the superstores that only recently opened have themselves gone out of business. ,)- 3ost of the office equipment superstores are owned by large retailing chains that also own stores selling other types of goods. ,*- %he growing importance of computers in most offices has changed the !ind of office equipment retailers must stoc!.

#nswer+ ' E planation+ Remember that you must stay within the scope of the argument. ;ou should develop a !een sense for what is, and what isn9t, relevant to the issue,s- at hand. In this problem, the argument is concerned solely with the retail mar!et& non0retail business, such as the direct contracting mentioned in choice ,a-, is irrelevant to the discussion. >ere s the basic s!eleton of the argument+ \ someone has attributed small firms( stru!!lin! in the equipment retail business to the entry of superstores into that retail market" \ %his is flawed because the superstores control a low ; of the market. ;our goal here is to wea!en the argument, which, as always, is done by undermining assumptions. It s actually fairly straightforward to pic! up on the !ey assumption in the argument here, because a connection is made, without any e6plicit 2ustification, between ,a- the superstores9 low mar!et share in the retail business and ,b- their purported inability to affect the bottom line of the other firms in the business. %herefore, the argument ,the original argument, which the narrator says is flawed- ma!es the following assumption+ stores with a small mar!et share can9t possibly affect the other firms in the mar!et. %herefore, we need to find an answer choice that !ives a way in which stores can affect the other firms in the market even if those stores have a small market share. (hoice ,b- does this+ the stores9 heavy advertising is independent of their small mar!et share, so this choice gives a way in which the superstores, despite a small mar!et share, can have a significant impact on the rest of the mar!et.

%o reiterate, choice ,a- is irrelevant, because the argument is concerned solely with the retail mar!et, and so any consideration of the non0retail mar!et is outside the scope of the argument.

.1F.%welve years ago and again five years ago, there were e6tended periods when )arfir Republic9s currency, the pundra, was wea!+ its value was unusually low relative to the world9s most stable currencies. 'oth times a wea! pundra made )arfir9s manufactured products a bargain on the world mar!ets, and )arfir9s e6ports were up substantially. Now some politicians are saying that, in order to cause another similarly sized increase in e6ports, the government should allow the pundra to become wea! again. /hich of the following if true provides the government with the strongest grounds to doubt the politican9s recommendation, if followed, will achieve its aim$ a- "everal of the politicians no recommending that the pundra be allowed to become wea! made that same recommendation before each of the last two periods of currency wea!ness. b- #fter several decades of operating well below its pea! capcity, darfir9s manufacturing sector is now operating at near0pea! levels c- the economy of a country e6periencing a rise in e6ports will become healthier only if the country9s currency is strong or the rise in e6ports is significant. d- those countries whose manufactured products compete with darfir9s on the world mar!et currently all have stable currencies e- a sharp improvement in the efficiency of darfir9s manufacturing plants would ma!e darfir9s products a bargain on the world mar!ets even without wea!ening of the pundra relative to other currencies.

#nswer+ ' E planation+ %he reason for this choice is as even if the currency is further devalued as manufacturing sector is now operating at near0pea! levels, it is not going to increase further production of goods and another similarly sized increase in e6ports will not be possible as supply will be limitedMrestricted due to the same.

notice the following+ 0 ,a- is irrelevant ,politicians9 stances M opinions don9t have a direct bearing on any of the economic indicators in the argument0 we can9t evaluate the effects of ,c- until we !now whether the purported rise in e6ports will be 9significant9 ,circular reasoning 0 we can9t base advocacy forMagainst a policy on its uncertain results0 ,d- irrelevant, as there9s no material difference between this situation and the situation during the first two e6port booms+ notice that, in those cases, the reference currencies were all stable as well 0 ,e- irrelevant+ the question as!s us to undermine the politician9s suggestion, not to suggest an alternative. ,#nalogy+ if you thin! that eating only big macs will help you lose weight, I can9t convince you otherwise by suggesting alternate diets-

=or (, %he recommendation9s aim is to cause a rise in e6ports. (hoice c is concerned with the effects of a rise in e6ports, and really has nothing to do with the cause of that rise. %herefore, we can9t tell whether it will have anything to do with the original aim of the recommendation.

..G./ith a record number of new companies starting up in )erderia and with previously established companies adding many 2obs, a record number of new 2obs were created last year in the )erderian economy. %his year, previously established companies will not be adding as many new 2obs overall as such companies added last year. %herefore, unless a record number of companies start up this year, )erderia will not brea! its record for the new 2obs created. /hich of the following is an assumption on which the argument relies$ #. *ach year, new companies starting up create more new 2obs overall than do previously established companies. '. (ompanies established last year will not add a greater number of 2obs overall this year than they did last year. (. %his year, the new companies starting up will not provide substantially more 2obs per company than did new companies last year. ). %>is year, the overall number of 2obs created by previously established companies will be less than the overall number of 2obs lost at those companies. *. %he number of 2obs created in the )erderian economy last year was substantially larger than the number of 2obs lost last year.

#nswer+ E planation+ when you e6amine questions involving quantitative arguments 0 which this argument definitely is, even though it doesn9t contain actual numbers 0 you should focus especially on the quantitative parts of the argument. "ure enough, in this problem, the issue is to be found in the numbers+ the argument asserts that a record number of new startups must be founded. however, what9s actually needed is a record number of new startup )obs. %herefore, you need an assumption that solidifies the idea that, unless a record number of startups are founded, you won9t get a record number of startup 2obs, either. (hoice c does e6actly that.

(hoice ,b- deals only with companies established last year, whereas the stated premise deals with companies established at any time before this year. %he companies dealt with in choice ,b-, then, are only a small subset of the companies dealt with in the stated premise. >owever, we don9t need to ma!e the assumption in choice ,b-, as its irrelevant one way or the other+ the companies founded last year are merely a subset of 8previously established companies8. therefore, since the stated premise already gives us the overall result for #LL previously founded companies, we have no need to be concerned with the results of smaller components of that population ,such as ,b--.

..1.Hitamin E;_ has long been a favorite among health food enthusiasts. In a recent large study, those who too! large amounts of vitamin E;_ daily for two years showed on average a 7G percent lower ris! of heart disease than did members of a control groups. Researchers corrected for differences in relevant health habits, such as diet.

/hich one of the following inference is most supported by the passage$ ,#- %a!ing large amount of vitamins is probably worth ris!ing the side effects. ,'- %hose who ta!e large doses of vitamin E;_ daily for the ne6t two years will e6hibit on average an increase in the li!elihood of avoiding heart disease. ,(- Li, who has ta!en large amounts of vitamin E;_ daily for the past two years, has a 7G percent lower ris!. ,)- %a!ing large amounts of vitamin E;_ daily over the course of one s adult life should be recommended to most adults.

,*- >ealth food enthusiasts are probably correct in believing that large daily doses of multiple vitamins promote good health.

#nswer+ ' E planation+ since this study is described as well0founded and controlled, the study9s results should be able to be replicated. %herefore, whichever answer choice comes closest to simply replicating the results of the study, /I%>:@% 3#OIN< #N; #))I%I:N#L #""@31%I:N", will be the correct answer. ,a- "ide effects aren9t mentioned at all in the passage, so this requires additional assumptions. ,b- %his basically 2ust says that the results of the study will be repeated 0 correct. ,c- %he 7GT is the average of a large group& large0group averages cannot be imputed to individual members of the group. %his requires additional assumptions ,and assumptions that are dubious, at that-. ,d- %here is a big difference between . years and the entire course of one9s adult life, so this requires lots of additional assumptions. ,e- 9<ood health9 is a much more general category than what9s described in the passage, so this requires considerable additional assumptions.

....:utsourcing is the practice of obtaining from an independent supplier a product or service that a company has previously provided for itself. Hernon, Inc., a small manufacturing company that has in recent years e6perienced a decline in its profits, plans to boost its profits by outsourcing those parts of its business that independent suppliers can provide at lower cost than Hernon can itself. /hich of the following, if true, most strongly supports the prediction that Hernon9s plan will achieve its goal$ ,#- #mong the parts of its business that Hernon does not plan to outsource are some that require standards of accuracy too high for most independent suppliers to provide at lower cost than Hernon can. ,'- Hernon itself acts as an independent supplier of specialized hardware items to certain manufacturers that formerly made those items themselves. ,(- Relatively few manufacturers that start as independent suppliers have been able to e6pand their business and become direct competitors of the companies they once supplied. ,)- Hernon plans to select the independent suppliers it will use on the basis of submitted bids. ,*- #ttending to certain tas!s that Hernon performs relatively inefficiently has ta!en up much of the time and effort of top managers whose time would have been better spent attending to Hernon9s core business.

#nswer+ * E planation+ (hoice ,d- is irrelevant& it merely ma!es a general statement about the way in which vernon will choose the companies to which it will outsource. Note that this choice says nothing about choosing 8optimal8 bids& for all we !now, vernon will actually choose the worst bid. 'ut, much more importantly, the point here is the main theme of the argument, which is the effect of outsourcing on profit. the way in which the outsourcing companies are selected doesn9t really pertain to that theme at all 0 remember that you need an answer choice that shows that -G*+-G:$'N, 0'LL 5E &-:E %:-.'*#5LE *H#N N-* -G*+-G:$'N, . %here is absolutely no connection between choice ,dand this idea. (hoice ,e- requires the assumption that freeing up top managers9 time will somehow contribute to profits 0 '@% remember that you9re loo!ing for the choice that 8most strongly supports8 the argument. In this case, the other four answer choices are completely irrelevant to the issue, which is the effect of outsourcing on profit. (hoice ,e-, on the other hand, unli!e the other choices, really does support the idea that :@%":@R(IN< /ILL '* 3:R* 1R:=I%#'L* %>#N N:% :@%":@R(IN<. %herefore, even though ,e- requires additional assumptions, it9s still the choice that 8most strongly supports8 the argument 0 because it9s the only choice that even could support the argument.

..5.Industrialists from the country )istopia were accused of promoting the )istopian intervention in the #rcadian civil war merely to insure that the industrialists facilities in #rcadia made substantial profits during the war. ;et this cannot be the motive since, as the )istopians foresaw, )istopia s federal e6penses for the interventions were eight billion dollars, whereas, during the war, profits from the )istopian industrialists facilities in #rcadia totaled only four billion dollars. /hich of the following, if true, e6poses a serious flaw in the argument made in the second sentence above$ ,#- )uring the #rcadian war, many )istopian industrialists with facilities located in #rcadia e6perienced a significant rise in productivity in their facilities located in )istopia. ,'- %he largest proportion of )istopia s federal e6penses is borne by those who receive no significant industrial profits. ,(- 3ost )istopian industrialists facilities located in #rcadia are e6pected to maintain the level of profits they achieved during the war. ,)- )istopian industrialists facilities in #rcadia made substantial profits before the events that triggered the civil war. ,*- 3any )istopians e6pressed concern over the suffering that #rcadians underwent during the civil war.

#nswer+ ' E planation+ "pecifically+ \ the profit of U7b went to the industrialists. \ %he cost of UDb was borne by the feds.

%he argument assumes 0 completely without 2ustification 0 that the UDb cost to the feds will somehow cancel out the industrialists9 profit. %here s no reason that this should be the case, or, for that matter, that the costs and revenues of those two entities should have anything to do with one another.

%herefore, to e6pose that flaw, we need an answer choice that demonstrates that the feds9 and industrialists9 ledgers are independent of each other, at least to a large enough degree that the industrialists can still ma!e a tidy profit. (hoice ,b- does this.

..7.#rchaeologists in 3ichigan have e6cavated a Native #merican camp near )umaw (ree!. Radiocarbon dating of animal bones found at the site indicates that the camp dates from sometime between 1BGA and 1CAA. >owever, the camp probably dates to no later than 1B5G, since no *uropean trade goods were found at the site, and *uropean traders were active in the region from the 1B.G9s onward.

/hich of the following, if true, most strengthens the argument$ ,#- )ue to trade among Native #mericans, some *uropean trade goods would have reached the area before the *uropean traders themselves did. ,'- #t all camps in the region that have been reliably dated to the late 1B.G9s, remains of *uropean trade goods have been found. ,(- %he first *uropean trade goods to reach the area would have been considered especially valuable and preserved as much as possible from loss or destruction. ,)- %he first *uropean traders in the area followed soon after the first *uropean e6plorers. ,*- %he site is that of a temporary camp that would have been used seasonally for a few years and then abandoned.

#nswer+ ' E planation+ choice # seems to buttress the argument a bit, by providing additional support for the idea that *uropean good should be at the site. >owever per the directions, you9re loo!ing for the one answer choice that 3:"% strengthens the argument. (hoice b strengthens the argument more than does choice a, because it fills in a badly needed assumption. In particular, the argument moves from a statement that no *uropean goods were found at the site to an inference that those goods were simply never there in the first place. %hat s quite an inductive leap, as not everything that was ever present somewhere leaves a trace& therefore, any choice that fills in that hole will be the best choice to strengthen the argument. %his is precisely what choice b does+ by providing evidence that such traces are, indeed, left behind when the trade goods in question have been present, it fills in the logical hole described above.

-000
:emember+ =illing in a missing assumption is considered better than reinforcing statements that have already been posited when it comes to strengthening an argument.

..A.%iger "har!s are common in the waters surrounding %enare Island. @sually tiger shar!s feed on smaller shar!s, but sometimes they have attached tourists swimming and surfing at %enare9s beaches. %his has hurt %enare9s tourism industry, which is secondary only to its fishing industry in annual revenues. In order to help the economy, therefore, the mayor of the island has proposed an ongoing program to !ill any tiger shar!s within a mile of the beaches. /hich of the following, if true, most strongly calls into question the li!elihood that implementation of the mayor9s proposal will have the desired consequence$ #- *ven if not all the tiger shar!s that come close to the beaches are !illed, the e6istence of the program would reassure tourists. '- 'usiness owners who depend on tourism are willing to pay most of the cost of implementing the program. (- %ourists come to %enare Island for its beaches, even though the island features a number of other tourist attractions. )- %he small shar!s on which tiger shar!s prey feed on fish that are commercially important to the island9s fisheries *- not all tourists who come to %enare Island en2oy swimming or surfing.

#nswer+ ) E planation+ here9s the basic deal+ \ economy has . main pillars+ %:@RI"3 and =I">IN< \ tiger shar!s are hurting %:@RI"3 \ mayor wants to help %:@RI"3 by !illing tiger shar!s if you want to wea!en this argument, you want to show that !illing the tiger shar!s will have a deleterious effect on the other pillar of the economy 0 namely, =I">IN<. ,Note how much more obvious this is if you diagram the passage, as I ve roughly done above-. (hoice d is correct, because it shows that, while !illing tiger shar!s may be good for %:@RI"3, it will be bad for =I">IN< ,which, the passage asserts, is ultimately a more important industry-. wrong answers+ choice a has the opposite effect+ it actually strengthens the mayor9s policy, by assuring that there will be a positive effect even if the program is not carried out to the fullest.

(hoice b is neutral ,transferring U from businesses neither infuses U into the economy nor removes U from the economy-. (hoice c strengthens the policy& because it underscores the idea that the beaches are critical to the economy ,and therefore that removal of the tiger shar!s is of paramount importancechoice e is irrelevant, because the non0beach tourists have no bearing on the issue either way.

..B. $olumnist+ 1eople should avoid using a certain artificial fat that has been touted as a resource for those whose medical advisers have advised them to reduce their fat inta!e. #lthough the artificial fat, which can be used in place of fat in food preparation, has none of the negative health effects of fat, it does have a serious drawbac!+ it absorbs certain essential vitamins, thereby preventing them from being used by the body. In evaluating the columnist9s position, it would be most useful to determine which of the following$ ,#- /hether increasing one9s inta!e of the vitamins can compensate for the effects of the artificial fat ,'- /hether the vitamins that the artificial fat absorbs are present in foods that contain the fat ,(- /hether having an e6tremely low fat inta!e for an e6tended period can endanger the health ,)- /hether there are any foods that cannot be prepared using the artificial fat as a substitute for other fats ,*- /hether people are generally able to detect differences in taste between foods prepared using the artificial fat and foods that are similar e6cept for the use of other fats

#nswer+ # E planation+ ;ou need to boil the columnist9s position down to its essence, which is basically this+ fat absorbs vitamins that are essential. %herefore, body doesn9t get vitamins. %herefore, bad news. %he second 9therefore9 here is unassailable ,it9s definitely bad news if your body doesn9t get vitamins-, so the only thing that might sway the argument in one direction or the other is the first 9therefore9. If we could brea! the connection between absorption of vitamins and robbing the body of vitamins, then we could possibly destroy the argument. (hoice # brea!s the connection, because it introduces the possibility that the body might get the vitamins anyway, despite the absorption of some of those vitamins by the fat. ' is irrelevant, as it doesn9t matter where the vitamins come from ,only whether they9re properly absorbed( is irrelevant, as the possible dangers of low0fat diets don9t affect the above line of reasoning ,the dangers of the artificial fat- at all ) is irrelevant+ which foods can contain the artificial fat has nothing to do with whether it will rob the body of vitamins * is irrelevant+ taste has no bearing on the discussion at hand

..C."ome airlines allegedly reduce fares on certain routes to a level at which they lose money, in order to drive competitors off those routes. >owever, this method of eliminating competition cannot be profitable in the long run. :nce an airline successfully implements this method, any attempt to recoup the earlier losses by charging high fares on that route for an e6tended period would only provide competitors with a better opportunity to undercut the airline9s fares. /hich of the following, if true, most seriously wea!ens the argument$ #. in some countries it is not illegal for a company to drive away competitors by selling a product below cost '. airline e6ecs generally believe that a company that once underpriced its fares to drive away competitors is very li!ely to do so again if new competitors emerge (. as part of promotions designed to attract new customers, airlines sometimes reduce their tic!et prices to

below an economically sustainable level. ). on deciding to stop serving particular routes, most airlines shift resources to other routes rather than reduce the size of their operations. *. when airlines dramatically reduce their fares on a particular route, the total number of air passengers on that route increases greatly.

#nswer+ ' E planation+ the argument rests on the premise that, once the leading airline raises its prices bac! up to 9normal9 higher levels, other airlines will 2ump right bac! into the fray. %herefore, choice b is correct+ it states that other airlines are li!ely to continue to stay away, even after the big mean price0cutting airline raises its prices bac! up. ,If their e6ecutives believe that 9big air9 will simply lower its prices again if they try to wedge bac! into the mar!et, then they9ll stay out.(hoice d is irrelevant, because the passage and its conclusion aren9t at all affected by what the other airlines do if they decide to stop serving some particular route. #ll that matters is that they decide to stop serving the route& the subsequent decisions are immaterial. (hoice e is also irrelevant, as the argument doesn9t turn on what happens during the low0fare period ,its most important premises concern what happens after prices are raised bac!-

..D.%he Rienzi, a passenger ship, san! as a result of a hole in its hull, possibly caused by sabotage. Normally, when a holed ship sin!s as rapidly as the Rienzi did, water does not enter the ship quic!ly enough for the ship to be fully flooded when it reaches the ocean floor. =ull flooding can be achieved, however, by sabotage. #ny ship that sin!s deep into the ocean when not fully flooded will implode. )eep0sea photographs, ta!en of the sun!en Rienzi where it rests on the ocean floor, reveal that the Rienzi did not implode. /hich one of the following must be true on the basis of the information above$ ,#- %he Rienzi was so constructed as to reduce the ris! of sin!ing by impact. ,'- If the Rienzi became fully flooded, it did so only after it reached the ocean floor. ,(- If the Rienzi was not sun! by sabotage, water flooded into it unusually fast. ,)- If the Rienzi had sun! more slowly, it would have imploded. ,*- %he Rienzi was so strongly constructed as to resist imploding under deep0sea pressure.

#nswer+ ( E planation+ %he second0to0last sentence reads+ 3#ny ship that sinks deep into the ocean when not fully flooded will implode"3 %his means that if a ship sin!s deep into the ocean when not fully flooded, it is sufficient to conclude that it will implode. %his yields the following conditional statement+ 'f a ship was not fully flooded when it sank, then it will have imploded" %he contra positive of this conditional statement is+ If a ship did N:% fully implode, then it was N:% fully flooded when it san! deep into the ocean, :r if a ship did N:% fully implode, then it /#" fully flooded when it san! deep into the ocean. %he final sentence tells us that deep0sea photography has 8revealed8 that the Rienzi did not implode.

%herefore, according to the conditional statement above, it is sufficient to conclude that the Rienzi was in fact fully flooded when it san! into the ocean. %he second sentence of the passage tells us that normally these !inds of ships ,i.e., ships that are sin!ing by water flooding in through a hole- are N:% fully flooded when they sin!. 'ut full flooding can be achieved by sabotage ,third sentence-. 'ut because we have deduced that the Rienzi /#" in fact fully flooded, either water flooded it abnormally fast, or else it was sabotaged. ,:r, in other words+ either it was sabotaged or else water flooded it abnormally fast.:r, in other words+ If the Rienzi was not sun! by sabotage, water flooded into it unusually fast. ,(hoice (-

..F. *estluvs Lesson: /hy should we avoid e6treme choices in inference questions$ /ell, the answer to this question has to do with the design of an inference question, and the nature of the process of inferring. In an inference question, all of the information in the passage is necessarily true. %he correct answer is something that the passage proves must also be true. 'ut the more e6treme an answer choice becomes, the less li!ely it is that the passage was strong or relevant enough to prove the choice as being something that is necessarily true. %he only time an e6treme choice will be correct is when that part of the passage that might support the answer choice is equally as e6treme00and that9s fairly rare. #ccordingly, in inference questions you should be partial to tentative choices over e6treme ones. #nd, why should we avoid e6treme choices in necessary assumption questions$ #gain, this has to do with the design of a necessary assumption question, and the concept of necessary assumption. # necessary assumption is something that the arguer9s reasoning depends on, relies on. # necessary assumption is something the author needs in order for the argument to wor!. 'ut it is unli!ely that the author9s argument will be so bold as to depend on an e6treme assumption. %hat9s why we should generally avoid e6treme choices in necessary assumption questions. "o, although it is good advice to avoid e6treme choices in both of these question types, we actually avoid e6treme choices in inference and necessary assumption questions for different reasons. 'ut neither of these reasons applies to sufficient assumption questions or strengthenMwea!en questions. In fact, a sufficient assumption is quite li!ely to be e6treme. %his again has to do with the concept of sufficient assumption. # sufficient assumption is something which, if we plugged it into the argument, the argument would be guaranteed. #nd, the more e6treme a choice is, the more li!ely it is something that will guarantee an argument. ,>owever, on the <3#%, !now that necessary assumption questions are way more common than sufficient assumption questions-. #lso, the more e6treme a choice is, the more li!ely it is something that will strengthen or wea!en a particular argument. >owever, e6treme choices show up fairly rarely in strengthenMwea!en bMc it is usually obvious that that choice would do the 2ob of strengthening or wea!ening. "o, although e6treme choices rarely show up as answer choices in strengthenMwea!en, when an e6treme answer choice does show up in these question types, don9t avoid it 2ust because it is e6treme00it might be the correct answerS 0000 Necessary assumption questions as! for an assumption that the author9s argument depends on or relies on, or for an assumption that is required for the argument. # necessary assumption is something without which the argument certainly fails. # sufficient assumption question can be identified from the absence of necessary language. #lso, they have distinctive phraseology, such as+ 8%he conclusion follows logically if which one of the following is assumed$.8 "o, a sufficient assumption is something with which the argument will be guaranteed ,certainly passes-.

.5G.1rivate industry is trying to attract s!illed research scientists by offering them high salaries. #s a result, most research scientists employed in private industry now earn AG percent more than do comparably s!illed research scientists employed by the government. "o, unless government0employed research scientists are motivated more by a sense of public duty than by their own interests, the government is li!ely to lose its most s!illed research scientists to private industry, since none of these scientists would have problems finding private sector 2obs.

/hich one of the following is an assumption on which the argument depends$ ,#- <overnment research scientists are less li!ely to receive ac!nowledgment for their research contributions than are research scientists in the private sector. ,'- None of the research scientists currently employed by the government earns more than the highest0paid researchers employed in the private sector. ,(- %he government does not employ as many research scientists who are highly s!illed as does any large company in the private sector which employs research scientists. ,)- %he government does not provide its research scientists with unusually good wor!ing conditions or fringe benefits that more than compensate for the lower salaries they receive. ,*- Research scientists employed in the private sector generally wor! longer hours than do researchers employed by the government.

#nswer+ ) E planation+ #0 is not within the scope of the argument& we don9t !now whether the government gives its scientists any recognition besides the less pay scientists receive in the government as compared to scientists in the private sector. 3aybe the government provides support, but the scientists are not satisfied. '0 *6treme word for a necessary assumption question, a red flag that the answer is wrong-. %he argument says that most scientists in the private sector earn more, and this means that most government scientists earn less, but not all. (0 If this was true, then why these scientists are hired by the private sector$ )0 (:RR*(% #N"/*R. %he assumption that the government does not give its scientists with equally important wor!ing conditions that could, in the eyes of the scientists, compensate them for the less pay they receive. *0 :ut of scope. /hether research scientists wor! longer hours than scientists in the government has no bearing on the conclusion. I thin! this could be an answer to strengthenM wea!en question.

.51.=or every AG dogs that contract a certain disease, one will die from it. # vaccine e6ists that is virtually 1GG percent effective in preventing this disease. "ince the ris! of death from complications of vaccination is one death per A,GGG vaccinations, it is therefore safer for a dog to receive the vaccine than not to receive it. /hich one of the following would it be most helpful to !now in order to evaluate the argument$ ,#- the total number of dogs that die each year from all causes ta!en together ,'- whether the vaccine is effective against the disease in household pets other than dogs ,(- the number of dogs that die each year from diseases other than the disease in question ,)- the li!elihood that a dog will contract another disease such as rabies ,*- the li!elihood that an unvaccinated dog will contract the disease in question #nswer+ * E planation+ %he argument is tal!ing and gives information about the following things+ 1. :ne out of every AG dogs dies because of the disease .. Haccine is a virtual 1GGT cure for the disease 5. Haccine has a death rate as well i.e. one death per AGGG vaccines 7. It is safer for a dog to receive a vaccine than not to receive it

"o as you can see ,from above- that the argument gives a lot of info about vaccine and also that one out of AG dogs die if they are not vaccinated. In order to evaluate the argument you would need to !now 0 what is the li!elihood that an unvaccinated dog will contract the disease. +cenario 7 0 Let9s say if 1 in 1GGGG dogs catch this disease, then vaccinating all the dogs is by no means a practical solution because 1 in AGGG dogs can die after vaccination. +cenario 1 0 Let9s say if 1 in 1G dogs contract that disease and you already !now that 1 out of AG dogs will die. %herefore the dogs should be vaccinated. %herefore in order to evaluate the argument you need to !now the li!elihood, percentage or probability of the unvaccinated dogs contracting the disease.

.5.."tudio e6ecutives carefully e6amine how a film performs on its opening wee!end in order to determine whether P and how P to invest more in that film. 3any decisions, such as increasing the number of screens that show the film and e6panding the mar!eting campaign, are best made after reaction can be gathered from audience who actually purchased tic!ets. %herefore, to ma6imize returns on their mar!eting investments, studios should initially release all their films on a small number of screens and with a limited advertising campaign.

%he plan to ma6imize returns by initially releasing films on only a small number of screens and limiting advertising depends on which of the following assumptions$ #- Large mar!eting investments made before the opening wee!end never eventually yields greater profits than small initial mar!eting investments. '- New advertising technique such as web0based viral mar!eting, haven t substantially reduced the average mar!eting cost for films. (- # film s prior performance in noncommercial settings, such as festivals, is not well correlated with how the general public tends to react to than film. )- #cross the movie industry, mar!eting investments do not influence the eventual financial returns of films in predictable way. *- >ow a film performs during its opening wee!end is a strong indicator of the film s financial performance over its lifetime.

#nswer+ * E planation+ /hile ,#- is tempting, and one can ma!e the argument that it is helpful for the argument, the argument does N:% necessarily fall apart if it9s not true. %he reason is because the (:N(L@"I:N is saying that the limited opening plan will produce the ma6imum profits. /hile opening wee!end performance is an indicator of its lifetime performance, it does not tell us anything about how a LI3I%*) opening wee!end performance will fare. ,*- (orrectly ,especially with the strong language, 8never8- eliminates the possibility that a L#R<* opening wee!end will not be as profitable as a LI3I%*) opening wee!end.

.55. +cientists+ #n e6perimental technique for combating severe depression, deep0brain stimulation ,)'"demonstrates much promise for the long0term treatment of chronic depression. In a recent e6periment, electrodes were implanted into the brains of si6 patients who had not responded to any currently approved treatment for depression. /hen an electrical current to the electrodes was switched on, four of the patients reported feeling a dramatic reduction of depressive symptoms. %he depressive symptoms returned when the current was switched off. /hich of the following, if true, best supports the scientist9s claim of the promising potential usage of )'"$ #. %he electrode implanted during deep0brain stimulation can only be activated in a hospital setting. '. %he other two patients reported a slight reduction of depressive symptoms when the current on their electrodes was activated.

(. %he operation to implant the electrodes poses a high ris! of brain hemorrhage, infection or seizure. ). In a subsequent e6periment, a one hour treatment the electrodes produced sustained remission from depression in the four patients for si6 months. *. )eep0brain stimulation relies on the e6pertise of highly s!illed physicians.

#nswer+ ) E planation+ Loo! at the conclusion+ #n e perimental technique for combatin! severe depression, deep<brain stimulation 4?5+6 demonstrates much promise for the lon!<term treatment of chronic depression Notice the !ey word 9long term9. "o we cannot conclude anything from this one e6periment. ": even if the other two patients had remission from depression, we still do not !now whether the effects will last for a 9longer term9. %he proof that the effects are long term is provided in ,)- there is sustained remission from depression for B months which strengthens the conclusion

.57.If, in a tennis tournament, a match reaches a fifth0set tiebrea!, the lower0ran!ed player always loses the tiebrea! ,and, therefore, the match-. If Rafael, the second0ran!ed player, wins a tournament by beating Roger, the top0ran!ed player, then the match must not have included a fifth0set tiebrea!.

/hich of the following arguments most closely mimics the reasoning used in the above argument$ ,#- If a woman with a family history of twins gets pregnant three times, she will have one set of twins. Nennifer, who falls into this category, had two sets of twins, so she must not have gotten pregnant e6actly three times. ,'- If a salesman sells more product than anyone else in a calendar year, then he will earn an all0e6penses0 paid vacation. Noe earned an all0e6pense0paid vacation, so he must have sold more product than anyone else for the year. ,(- # newspaper can charge a AGT premium for ads if its circulation surpasses 1GG,GGG& if the circulation does not pass 1GG,GGG, therefore, the newspaper can t charge any !ind of premium for ads. ,)- If a student is in the top 1GT of her class, she will earn a college scholarship. #nna is not in the top 1GT of her class, so she will not earn a scholarship. ,*- #ll of the players on a football team receive a cash bonus if the team wins the "uper 'owl. If quarterbac! %om 'rady earned a cash bonus last year, he must have been a member of the winning "uper 'owl team. #nswer+ # E planation+ %he logic of the passage follows this pattern+ if #, then '& if not ', then not #.

If, in a tennis tournament, a match reaches a fifth0set tiebrea!, ,#- the lower0ran!ed player always loses the tiebrea! ,'- ,and, therefore, the match-. If Rafael, the second0ran!ed player, wins a tournament by beating Roger, ,Not '- the top0ran!ed player, then the match must not have included a fifth0set tiebrea!. ,Not #-

If a woman with a family history of twins gets pregnant three times, ,#- she will have one set of twins ,'-. Nennifer, who falls into this category, had two sets of twins, ,Not '- so she must not have gotten pregnant e6actly three times ,Not #-. # is the only answer choice that fits this pattern.

.5A.@rban air contains more sulfur dio6ide than does rural air, and plants in cities typically grow more slowly than do plants in rural areas. In an e6periment to see how much of the difference in growth is due to sulfur dio6ide, classes in an urban and a rural school grew plants in greenhouses at their schools and filtered the greenhouse air to eliminate sulfur dio6ide. 1lants in the urban greenhouse grew more slowly than those in the rural greenhouse. /hich of the following, if true, would it be most important to ta!e into account in evaluating the result$ #. %he urban school was located in a part of the city in which levels of sulfur dio6ide in the air were usually far lower than is typical for urban areas. '. #t both schools, the plants in the greenhouses grew much more quic!ly than did plants planted outdoors in plots near the greenhouses. (. %he urban class conducting the e6periment was larger than the rural class conducting the e6periment. ). >eavy vehicular traffic such as is found in cities constantly deposits grime on greenhouse windows, reducing the amount of light that reaches the plants inside. *. 'ecause of the higher levels of sulfur dio6ide in the air at the urban school, the air filters for the urban school9s greenhouse were changed more frequently than were those at the rural school.

#nswer+ ) E planation+ If you loo! at the sentences, we can infer that "ulfur )io6ide is considered as the main culprit for the slow growth of the plants and the e6periments were done to find the e6tent of damage done by "ulfur )io6ide and see how much of a growth difference in growth is brought in. %he results show that the urban plants grew more slowly than rural plants. "o if we analyze the e6perimental results, there is something else other than "ulfur )io6ide that contributed for the slower growth. "o what we need to consider is anything other than "ulfur )io6ide that could hinder the growth of the plants. "o the answer choice should be something that is not related to "ulfur )io6ide, which is causing the slow growth. #- It says "ulfur )io6ide level was lower at the urban school. 'ut still the urban plants grew slower. "o this choice, give support to the argument that there is something else other than "ulfur dio6ide that contributed to the slower growth. 'ut it doesn9t give any hint on what else could have caused the slow growth. '- %his choice compares :pen urban plants vs. <reenhouse @rban plants and similarly for the rural plants. 'ut it doesn9t provide an alternate reason for the slow growth. (- "ize of the urban class is no way related to the slower growth of plants. )- %his gives an alternate reason for the slow growth. @rban greenhouses prevented enough sunlight to reach the plants and hence it caused a slower growth. *- %his statement though says that the filters were replaced frequently, it is difficult to figure out an alternate reason for the slow growth. 0000 \ if there are differences in the amount of light being let through the windows, then the e6perimental control ,identical conditions- is destroyed, and it9s still possible that sulfur dio6ide causes the urbanMrural difference. \ if there are N:% differences in the amount of light, then, since all the sulfur dio6ide is suc!ed out of the greenhouses, the difference 3@"% be caused by other factors. ,d- /ill determine this difference.

.5B.=or years, the debate over public education reform has centered on financing. 3any claim that pouring more money into the public schools will improve student performance. >owever, the only way to fi6 our school systems is to in2ect new ideas and new approaches. %oday the schools are organized to benefit their adult employees rather than the students.

/hich of the following, if true, best wea!ens the argument$ #. "chools that have instituted Knew approachesL attract the best performing students. '. "chools without outside playgrounds have lower levels of student performance than schools that do. (. "tudies show that student performance corresponded most directly with the education of the students families. ). "chool employees, by an overwhelming margin, said that the system performed well, citing superior benefits than those available in comparable private institutions. *. Researchers in education have shown that students from school districts with high per0capita spending tend to receive higher scores on standardized tests.

#nswer+ * E planation+ If higher per capita income for schools leads to higher scores, then we can conclude that schools are not having a wrong approach by focusing on financing. .5C.%he @nited "tates government uses only a household9s cash income before ta6es to determine whether that household falls below the poverty line in a given year& capital gains, non0cash government benefits, and ta6 credits are not included. >owever, yearly cash income is not a fool0proof measure of a given household9s disposable income. =or e6ample, retirees who live off of capital gains from an e6tensive portfolio could earn hundreds of thousands of dollars, yet be classified by the government as living in 8poverty8 because this income is not included in the calculation. /hich of the following, if true, validates the contention that the government9s calculation methods must be altered in order to provide statistics that measure true poverty$ #. =or more than FFT of those classified as living in poverty, yearly cash income comprises the vast ma2ority of each household9s disposable income. '. /hile the government s calculation method indicated a 1..AT poverty rate in .GG5, the same calculation method indicated anywhere from a FT to a 1BT poverty rate during the preceding decade. (. 3ost established research studies conducted by the private sector indicate that the number of people truly living in poverty in the @.". is less than that indicated by the government s calculation method. ). "everal prominent economists endorse an alternate calculation method which incorporates all income, not 2ust cash income, and ad2usts for ta6es paid and other core e6penses. *. %he government s calculation method also erroneously counts those who do not earn income in a given year but who have substantial assets on which to live during that year. #nswer+ ( E planation+ 1rivate research companies have conducted a research that proves that the number of people living below poverty is not correct. * is 2ust an e6tension of the argument. %he fact that the people are erroneously included in the poverty count is already present in the argument.

.5D. ,loria+ %hose who advocate tuition ta6 credits for parents whose children attend private schools maintain that people ma!ing no use of a government service should not be forced to pay for it. ;et those who choose

to buy bottled water rather than drin! water from the local supply are not therefore e6empt from paying ta6es to maintain the local water supply. :o!er+ ;our argument is illogical. (hildren are required by law to attend school. "ince school attendance is a matter not of choice, but of legal requirement, it is unfair for the government to force some parents to pay for it twice.

/hich of the following responses by <loria would best refute Roger s charge that her argument is illogical$ ,#- #lthough drin!ing water is not required by law, it is necessary for all people, and therefore my analogy is appropriate. ,'- %hose who can afford the tuition at a high0priced private school can well bear the same ta6 burden as those whose children attend public schools. ,(- If tuition ta6 credits are granted, the ta6 burden on parents who choose public schools will rise to an intolerable level. ,)- %he law does not say that parents must send their children to private schools, only that the children must attend some !ind of school, whether public or private. ,*- 'oth bottled water and private schools are lu6ury items, and it is unfair that some citizens should be able to afford them while others cannot.

#nswer+ #

.5F.#n electronics company plans to lure first0time buyers this holiday season. :ne aspect of its mar!eting strategy will be to ma!e widely available percent0off coupons for its products that customers can use at any retailer.

/hich of the following, if true, would point to a possible flaw in the company9s plan$ ,#- %he coupons would ma!e it less li!ely that retailers prominently stoc! the company9s offerings in places li!ely to attract first0time buyers. ,'- %he company is supporting the coupon distribution effort with a nationwide television advertising campaign. ,(- %he company9s competitors are not e6pected to offer coupons of any !ind this holiday season. ,)- 3ost retailers are accustomed to accepting percent0off coupons from manufacturers and train their employees accordingly. ,*- Research has shown that first0time buyers of electronics products often choose which brand to purchase based on price.

#nswer+ # E planation+ %his is a flaw question00similar to a wea!en question. %he plan is to attract first0time customers with coupons they can use at any retailer. (onsider each choice+

,#- %his is correct. If the coupons lead retailers to ma!e choices that ma!e it less li!ely first0time customers will buy the company9s products, the coupons will have been counter0productive and the plan will have failed. ,'- %his choice suggests a way in which the company will ma!e the plan more li!ely to succeed. ,(- Li!e ,'-, this is another bit of evidence that suggests the company9s plan is li!ely to succeed. ,)- %his choice addresses a possible problem, but claims that employees will be trained to handle coupons li!e the ones on which the company9s plan hinge. ,*- %his supports the plan9s li!elihood of success. If first0time buyers are influenced by price, a coupon would ma!e them more li!ely to choose the company9s products.

.7G.%he average age of chief e6ecutive officers ,(*: s- in a large sample of companies is AC. %he average age of (*: s in those same companies .G years ago was appro6imately eight years younger. :n the basis of those data, it can be concluded that (*: s in general tend to be older now.

/hich of the following casts the most doubt on the conclusion drawn above$ ,#- %he dates when the (*: s assumed their current positions have not been specified. ,'- No information is given concerning the average number of years that (*: s remain in office. ,(- %he information is based only on companies that have been operating for at least .G years. ,)- :nly appro6imate information is given concerning the average age of the (*: s .G years ago. ,*- Information concerning the e6act number of companies in the sample has not been given.

#nswer+ ( E planation+ ' is incorrect because there isn9t any necessary relationship between someone9s age and how long they have been a (*: in a company. ;ou can be si6ty years old and been (*: for five years& you can be forty years old and been (*: for twenty years. In other words, the scope of the conclusion is the (*:s9 age. %he correct answer must be choice (. %he argument9s conclusion is that (*:9s tend to be older now in general. >owever, because the study only loo!s at companies that were e6isting at least twenty years ago, the study has completely ignored companies that came into being fifteen, ten or five years ago. It could be that, in these companies the (*:s are quite young. In other words, this is a misrepresentative sample. /henever you get a (R about studies, samples, statistics etc, you should always as! yourself+ what is the group in the conclusion, and what is the group in the evidence$

.71.# recent survey of all auto accident victims in )ole (ounty found that, of the severely in2ured drivers and front0seat passengers, DG percent were not wearing seat belts at the time of their accidents. %his indicates

that, by wearing seat belts, drivers and front0seat passengers can greatly reduce their ris! of being severely in2ured if they are in an auto accident.

%he conclusion above is not properly drawn unless which of the following is true$ ,#- :f all the drivers and front0seat passengers in the survey, more than .G percent were wearing seat belts at the time of their accidents. ,'- (onsiderably more than .G percent of drivers and front0seat passengers in )ole (ounty always wear seat belts when traveling by car. ,(- 3ore drivers and front0seat passengers in the survey than rear0seat passengers were very severely in2ured. ,)- 3ore than half of the drivers and front0seat passengers in the survey were not wearing seat belts at the time of their accidents. ,*- 3ost of the auto accidents reported to police in )ole (ounty do not involve any serious in2ury.

#nswer+ # E planation+ %he first sentence reads+ # recent survey of all auto accident victims in )ole (ounty found that, of the severely in2ured drivers and front0seat passengers, DG percent were not wearing seat belts at the time of their accidents. %he implication is that some victims were severely in2ured and other accident victims were in2ured but not severely.

%he conclusion reads+ *his indicates that, by wearin! seat belts, drivers and front<seat passen!ers can !reatly reduce their risk of bein! severely in)ured if they are in an auto accident . "o he is concluding that by wearing a seatbelt your ris! of being severely in2ured from an automobile accident decreases. ,Not that your ris! of being in an accident decreasesS-

In order for him to be right that the ris! of being severely in2ured decreases by wearing a seatbelt, there should be 3:R* non0seatbelt wearers in the severely in2ured group than there are in the Yregular in2ured group. %hat is the fraction of those not wearing seatbelts should increase with the severity of the in2ury. %hen, choice # matches this insight.

.7.. %rofessor #: /e must ma!e a strong moral statement against (ountry E s policies. :nly total divestmenta the sale of all stoc! in companies that have factories or business offices in Eacan do this. %herefore, the university should divest totally. %rofessor 5: :ur aim should be to encourage E to change its policies. 1artial divestment is the best way to achieve this aim. %herefore, the university should sell its stoc! only in companies that either sell goods to E s government, or do the ma2ority of their business in E, or treat their wor!ers in E unfairly.

1rofessor # s and 1rofessor ' s arguments differ in which of the following ways$ ,#- %hey state the same goal but propose different ways of achieving it. ,'- %hey state different goals but propose the same way of achieving them. ,(- %hey state different goals and propose different ways of achieving them. ,)- %hey disagree about whether the university should sell any stoc! at all. ,*- %hey disagree about whether E s policies are ob2ectionable. #nswer+ ( E planation+ ?uestion prompt as!s how the two arguments differ. Loo! at the conclusion for clues. %he two arguments conclude two different methods for resolving the problem. *liminate '. #nswer * and ) is completely off base. Loo! at the evidence to identity the goals in each argument. 1rof. # wants to ma!e a strong moral statement. 1rof ' wants to encourage country E to change policy. 'oth professors have different goals. %he only answer that meets this criterion is solution (. #nswer ( is the correct answer.

.75.%he upcoming presidential election in the /est #frican republic of <anelon is of grave concern to the @.". "tate )epartment. <anelon presently has strong political and military ties to the @nited "tates. >owever, the "ocialist party is widely e6pected to win the election, leading to fears that <anelon will soon brea! away from the pro0#merican bloc and adopt a nonaligned or openly anti0#merican stance.

/hich of the following is an assumption made in the passage above$ ,#- # "ocialist party government in <anelon is more li!ely to oppose the @nited "tates than is a non0 "ocialist party government. ,'- %he people of the @nited "tates recognize their nation s interest in the political stability of /est #frica. ,(- # wea!ening of @.". political ties with <anelon could have serious consequences for @.". relations with other #frican nations. ,)- %he "ocialist party leaders in <anelon believe that their nation s interests would best be served by an alliance with anti0#merican forces. ,*- %he "ocialist party will win the upcoming election in <anelon.

#nswer+ #

E planation+ *liminate * and (. * contains e6treme language. ( is outside of the scope of the passage. #nswer ' mentions the @" people s opinion& however this is outside the scope of the conclusion. #nswer ) is not supported by any information in the passage. %he only answer remaining, #, ta!es the evidence only one step further and lin!s the evidences to the conclusion by stating that a socialist party is more li!ely to oppose the @" than a non socialist party. %he answer is #.

.77.Oitchen magazine plans to license the use of its name by a line of coo!ware. =or a magazine, licensing the use of its name for products involves some danger, since if the products disappoint consumers, the magazine9s reputation suffers, with consequent reductions in circulation and advertising. >owever, e6perts have evaluated the coo!ware and found it superior to all other coo!ware advertised in Oitchen. %herefore, Oitchen can collect its licensing fee without endangering its other revenues.

%he argument above assumes which of the following$ #. No other line of coo!ware is superior to that which will carry the Oitchen name. '. Oitchen will not license the use of its name for any products other than the line of coo!ware. (. 3a!ers of coo!ware will not find Oitchen a less attractive advertising vehicle because the magazine9s name is associated with a competing product. ). (onsumers who are not regular readers of Oitchen magazine will be attracted to the coo!ware by the Oitchen name. *. Oitchen is one of the most prestigious coo!ing0related magazines.

#nswer+ ( E planation+ #+ )oesn t matter even if some other coo!ware is superior to the one using !itchen9s name. In fact I see a scope shift 8other line of coo! ware8 vs 8other line of coo!ware advertised in !itchen8. *liminate. '+ #gain who cares. If it does lic to brand of shaving cream as long as it can get revenues. It may or may not wor!. "o this is not an assumption. *liminate. (+ /ell now if ( is true, then the other ma!ers of coo!ware who have till now advertised in !itchen might stop advertising in Oitchen. %his could lead to loss of revenue. "o ( could be the answer. )+ /hat if this is not true.... no harm done. u will not increase revenue but u mayMmay not lose revenue.. *liminate. *+ /ho cares is !itchen is not the most prestigious mag. *liminate.

.7A.Nunior biomedical researchers have long assumed that their hirings and promotions depend significantly on the amount of their published wor!. 1eople responsible for ma!ing hiring and promotion decisions in the biomedical research field, however, are influenced much more by the overall impact that a candidate9s scientific publications have on his or her field than by the number of those publications. %he information above, if accurate, argues most strongly against which of the following claims$ #. *ven biomedical researchers who are 2ust beginning their careers are e6pected already to have published articles of ma2or significance to the field. '. (ontributions to the field of biomedical research are generally considered to be significant only if the wor! is published. (. %he potential scientific importance of not0yet0published wor! is sometimes ta!en into account in decisions regarding the hiring or promotion of biomedical researchers. ). 1eople responsible for hiring or promoting biomedical researchers can reasonably be e6pected to ma!e a

fair assessment of the overall impact of a candidate9s publications on his or her field. *. 'iomedical researchers can substantially increase their chances of promotion by fragmenting their research findings so that they are published in several 2ournals instead of one.

#nswer+ * E planation+ %he author contends that contrary to the assumption, those who hire and promote people in biomedical research field put more emphasis on the impact a candidate9s scientific publications have on his or her field than the number of those publications. %o argue against this conclusion, we need to show that total number of publications is considered more important, in at least some cases, by those who hire. (hoice * clearly states this+ 'iomedical researchers can substantially increase their chances of promotion by fragmenting their research findings so that they are published in several 2ournals instead of one. I.e. publishing in several 2ournals or increasing the number of publications increases the probability of promotion

.7B.%races of cultivated emmer wheat have been found among the earliest agricultural remains of many archaeological sites in *urope and #sia. %he only place where the wild form of emmer wheat has been found growing is a relatively narrow strip of southwest #sia. "ince the oldest remains of cultivated emmer wheat yet found are from village sites in the same narrow strip, it is clear that emmer wheat was first domesticated somewhere in that strip. /hich of the following, if true, most strengthens the argument$ ,#- %he present0day distribution of another wild wheat, ein!orn, which was also domesticated early in the development of agriculture, covers a much larger area of southwest #sia. ,'- 3odern e6periments show that wild emmer wheat can easily be domesticated so as to yield nearly as well as traditionally domestic strains. ,(- #t the time when emmer wheat was first cultivated, it was the most nutritious of all the varieties of grain that were then cultivated. ,)- In the region containing the strip where wild emmer wheat has been found, climatic conditions have changed very little since before the development of agriculture. ,*- It is very difficult, without genetic testing, to differentiate the wild form of emmer wheat from closely related wild wheat that also grows in southwest #sia.

#nswer+ ) E planation+ the evidence in the argument is based on where this strain of wheat has been found growing, N:/ in modern times ,as you can tell from the present perfect, 8has been found growing8-. If we9re going to argue about the domestication of this wheat, in ancient times, then we need to !now that the same conditions that prevail now also prevailed bac! then. %his is why ,d- strengthens the argument. /ithout ,d-, it9s irrelevant where this wheat grows today.

.7C.1roponents of the electric car maintain that when the technical problems associated with its battery design are solved, such cars will be widely used and, because they are emission0free, will result in an abatement of the environmental degradation caused by auto emissions. 'ut unless we dam more rivers, the electricity to

charge these batteries will come from nuclear or coal0fired power plants. *ach of these three power sources produces considerable environmental damage. %hus, the electric car 4444444.

/hich one of the following most logically completes the argument$ ,#- will have worse environmental consequences than its proponents may believe ,'- will probably remain less popular than other types of cars ,(- requires that purely technical problems be solved before it can succeed ,)- will increase the total level of emissions rather than reduce it ,*- will not produce a net reduction in environmental degradation

#nswer+ # E planation+ %he correct answer must be choice #. %he first sentence gives us the 1:H of the 8proponents8. %hey thin! that the electric car will 8result in an abatement of the environmental degradation caused by auto emissions8. %heir argument can be summed up as+ 8electric car will lead to less auto0emission0induced environmental damage 8. %he ne6t sentence ,i.e., second sentence- begins with the contrast !eyword 8but8. %his tells us the author is out to argue against the proponents00so, at this point, we !now he will generally argue that electric cars are actually not a cure0all. %his sentence and the ne6t one ,third one- must be evidence because the fourth sentence ,the one we have to complete- begins with 8thus8. %o complete the author s 1:H correctly, we should e6amine his evidence. /ell, the third sentence ain t so bad as it 2ust tells us that the electricity that will run these electric cars comes from nuclear or coal0power sources. %he fourth sentence tells us that each of these sources 8produces considerable environmental damage8. 'ut notice the author s evidence ,considerable environmental damage- does not establish whether, overall, a switch would mean more environmental damage. >e never compares levels of environmental damage. "o choice * is wrong, and choice ) is e6treme, and wrong for pretty much the same reason. %he author s main point, then, is that electric cars are not the cure0all their proponents thin! them to be00that there will still be considerable environmental damage even if we switch to them. %he author s 1:H, then, can be summed up as+ 8because the production processes for these electric cars are environmentally damaging, switching to electric cars won t necessarily reduce environmental damage8. (hoice # is a perfect match to this prediction& choose #.

In inference questions ,complete the blan! questions are 2ust inference questions-, the passage and the correct answer are always things that must be true. %herefore, the wrong answers are things that could or must be false. %herefore, you can use denial test+ If the author did not believe in choice #, would his argument ma!e any sense$ /ould his argument still be available to him$ /ell, here, if the author did not believe in choice #00if he believed its9 opposite00it would mean that he thought the proponents are bang0on in their optimistic prediction about electric cars. %hen, what the hec! e6plains the 8but8$ If the author did not believe in choice #, then his statements do not ma!e any sense at all. 3ore technically, if choice # were false, the passage would be falsified, but, in inference questions, we must always treat the passage as necessarily true. %herefore, he must believe in choice #+ it must be true. (hoice * loo!s li!e the 8trap8 answer here. Let9s try denying it. /hen you see 8not8, 2ust deny by removing 8not8. "o, if the author thought that a switch would produce a reduction in environmental damage, do his statements still ma!e at least some sense$ Is his argument still available to him$ ;es, his statements surely still ma!e at least some sense because he said nuclear and coal power is environmentally damaging& he

never compared the specific level of environmental damage from nuclearMcoal power to current cars. 3ore technically, if choice * were false, the passage is not necessarily falsified. %herefore, choice * is not necessarily true+ it could be false. In inference questions, focus on the connections between sentences, and ,especially in the complete0the0 blan! variety-, try to thin! about what the author9s main point is, what he is trying to get at.

.7D. Hospital e ecutive+ #t a recent conference on nonprofit management, several computer e6perts maintained that the most significant threat faced by large institutions such as universities and hospitals is unauthorized access to confidential data. In light of this testimony, we should ma!e the protection of our clients confidentiality our highest priority.

%he hospital e6ecutive s argument is most vulnerable to which one of the following ob2ections$ ,#- %he argument confuses the causes of a problem with the appropriate solutions to that problem. ,'- %he argument relies on the testimony of e6perts whose e6pertise is not shown to be sufficiently broad to support their general claim. ,(- %he argument assumes that a correlation between two phenomena is evidence that one is the cause of the other. ,)- %he argument draws a general conclusion about a group based on data about an unrepresentative sample of that group. ,*- %he argument infers that a property belonging to large institutions belongs to all institutions.

#nswer+ ' E planation+ Is computer experts' e6pertise broad enough to decide what institutions9 highest priority should be$ No, computer e6perts, for e6ample, will not be versed in, say, business strategy, law, etc. It could be that a threat having to do with one of these other aspects constitutes an even greater threat than unauthorized access to confidential data. %he author9s conclusion is a recommendation about what institutions9 highest priority should be. (hoice ) is wrong because the author isn9t even drawing a conclusion about a group.

.7F.*very year, children are born with neural tube defects, which can prevent full formation of the brain or closing of the base of the spine, causing paralysis. # primary cause was found to be a deficiency of folic acid, an essential nutrient, in the diets of pregnant women. %o reduce the incidence of these birth defects, the country of Islandia began to require folic acid supplementation of all wheat flour. %his ensured that everyone received adequate amounts of folic acid in their diets, thereby preventing neural tube defects in children.

*ach of the following, if true, helps to e6plain how the supplementation of wheat flour could have prevented neural tube defects in children *E(*1%+ ,#- 1regnant women whose families did not traditionally eat food containing folic acid found it easier to supplement their diets with the nutrient when it was already contained in a staple such as wheat flour. ,'- #ware that wheat flour had additional health benefits, Islandia residents consumed larger amounts of wheat flour than before the supplementation began.

,(- %he supplementation of folic acid caused wheat flour distributors to more aggressively compete for customers, who they e6pected to see! out their products for health reasons. ,)- 1regnant women who ignored recommended supplements such as folic acid could not avoid including wheat flour in their daily diet. ,*- 1rompted by the country9s emphasis on folic acid supplementation, pregnant women sought out other foods that naturally contain folic acid.

#nswer+ ( E planation+ %his is an e6planation question, and also an 8e6cept8 question, which tends to ma!e things a bit harder. %he passage describes the requirement that wheat flour be supplemented with folic acid so that pregnant women consume more folic acid, thus avoiding birth defects. =our of the choices will describe ways in which the supplementation limits birth defects& we9re loo!ing for the one that does not+ ,#- %his describes a way in which pregnant women who did not consume enough folic acid will start consuming enough. ,'- %his choice suggests that the supplementation had a publicity effect that caused people to consume more folic acid. ,(- %his is correct. %he important thing is that pregnant women consume folic acid, not which wheat flour distributors earn the most business. ,)- %his choice shows that supplementing wheat flour helped reach even those women who would not otherwise ta!e recommendations. ,*- %his is yet another way in which the supplementation plan helped get more folic acid in the diets of pregnant women.

.AG.)uring the nineteenth century, the =rench academy of art was a ma2or financial sponsor of painting and sculpture in =rance& sponsorship by private individuals had decreased dramatically by this time. 'ecause the academy discouraged innovation in the arts, there was little innovation in nineteenth century =rench sculpture. ;et nineteenth century =rench painting showed a remar!able degree of innovation.

/hich one of the following, if true, most helps to e6plain the difference between the amount of innovation in =rench painting and the amount of innovation in =rench sculpture during the nineteenth century$ ,#- In =rance in the nineteenth century, the =rench academy gave more of its financial support to painting than it did to sculpture. ,'- %he =rench academy in the nineteenth century financially supported a greater number of sculptors than painters, but individual painters received more support, on average, than individual sculptors. ,(- 'ecause stone was so much more e6pensive than paint and canvas, far more unsponsored paintings were produced than were unsponsored sculptures in =rance during the nineteenth century. ,)- Hery few of the artists in =rance in the nineteenth century who produced sculptures also produced paintings. ,*- #lthough the academy was the primary sponsor of sculpture and painting, the total amount of financial support that =rench sculptors and painters received from sponsors declined during the nineteenth century.

#nswer+ ( E planation+

1assage says+ )uring the nineteenth century, the =rench academy of art was a ma2or financial sponsor of painting and sculpture in =rance. 'ecause the academy discouraged innovations in arts. =rom those two, conclude that the academy was a sponsor of art, which included both sculpting and painting, and when it discouraged innovations in arts, it did so for both branches of art. %his ma!es sense because later in the passage we find out that sculpting decreased while painting increased, and that ma!es a parado6ical situation, ma!ing the question valid.

.A1.1hilosopher+ #n action is morally right if it would be reasonably e6pected to increase the aggregate well0 being of the people affected by it. #n action is morally wrong if and only if it would be reasonably e6pected to reduce the aggregate wellbeing of the people affected by it. %hus, actions that would be reasonably e6pected to leave unchanged the aggregate well0being of the people affected by them are also right.

%he philosopher s conclusion follows logically if which one of the following is assumed$ ,#- :nly wrong actions would be reasonably e6pected to reduce the aggregate well0being of the people affected by them. ,'- No action is both right and wrong. ,(- #ny action that is not morally wrong is morally right. ,)- %here are actions that would be reasonably e6pected to leave unchanged the aggregate well0being of the people affected by them. ,*- :nly right actions have good consequences.

#nswer+ *6planation+

.A..%op college graduates are having more difficulty demonstrating their superiority to prospective employers than did the top students of twenty years ago when an honors degree was distinction enough. %oday s employers are less impressed with the honors degree. %wenty years ago no more than 1G percent of a given class graduated with honors. %oday, however, because of grade inflation, the honors degree goes to more than AG percent of a graduating class. %herefore, to restore confidence in the degrees they award, colleges must ta!e steps to control grade inflation. /hich one of the following is an assumption that, if true, would support the conclusion in the passage$ ,#- %oday s students are not higher achievers than the students of twenty years ago. ,'- #warding too many honors degrees causes colleges to inflate grades. ,(- %oday s employers rely on honors ran!ing in ma!ing their hiring decisions. ,)- It is not easy for students with low grades to obtain 2obs. ,*- (olleges must ma!e employers aware of the criteria used to determine who receives an honors degree.

#nswer+ #

E planation+ 8grade inflation8 means teachers giving out high grades even though those high grades were not deserved. If teachers give out an 8#8 when the student only deserved a 8'8 then that student9s grade is inflated. #nd if the teacher does this for a lot of students, we would have grade inflation in that teacher9s class. If many of the teachers in the school did this, there would be grade inflation in the school. If many of the teachers in many of the schools across the country did this, there would be grade inflation in the entire country.

.A5.It is now a common complaint that the electronic media have corroded the intellectual s!ills required and fostered by the literary media. 'ut several centuries ago the complaint was that certain intellectual s!ills, such as the powerful memory and e6temporaneous eloquence that were intrinsic to oral culture, were being destroyed by the spread of literacy. "o, what awaits us is probably a mere alteration of the human mind rather than its devolution. %he reference to the complaint of several centuries ago that powerful memory and e6temporaneous eloquence were being destroyed plays which one of the following roles in the argument$ ,#- evidence supporting the claim that the intellectual s!ills fostered by the literary media are being destroyed by the electronic media ,'- an illustration of the general hypothesis being advanced that intellectual abilities are inseparable from the means by which people communicate ,(- an e6ample of a cultural change that did not necessarily have a detrimental effect on the human mind overall ,)- evidence that the claim that the intellectual s!ills required and fostered by the literary media are being lost is unwarranted ,*- possible evidence, mentioned and then dismissed, that might be cited by supporters of the hypothesis being criticized

#nswer+ ( E planation+ "tructure+ #rgument compares the two views 0 one a historical view and the other 0 current view. %he author compares to see the change of culture ,No longer literary media- in the current situation. =inally, the author concludes that there is hardly any detrimental alteration to the human brain. %his is what e6actly this choice says. .A7.# cup of raw mil!, after being heated in a microwave oven to AG degrees (elsius, contains half its initial concentration of a particular enzyme, lysozyme. If, however, the mil! reaches that temperature through e6posure to a conventional heat source of AG degrees (elsius, it will contain nearly all of its initial concentration of the enzyme. %herefore, what destroys the enzyme is not heat but microwaves, which generate heat.

/hich one of the following, if true, most seriously wea!ens the argument$ ,#- >eating raw mil! in a microwave oven to a temperature of 1GG degrees (elsius destroys nearly all of the lysozyme initially present in that mil!. ,'- *nzymes in raw mil! that are destroyed through e6cessive heating can be replaced by adding enzymes that have been e6tracted from other sources.

,(- # liquid e6posed to a conventional heat source of e6actly AG degrees (elsius will reach that temperature more slowly than it would if it were e6posed to a conventional heat source hotter than AG degrees (elsius. ,)- 3il! that has been heated in a microwave oven does not taste noticeably different from mil! that has been briefly heated by e6posure to a conventional heat source. ,*- >eating any liquid by microwave creates small zones within it that are much hotter than the overall temperature that the liquid will ultimately reach.

#nswer+ * E planation+ 3any strengthenMwea!en questions can be viewed as e6plain the phenomenon or cause of the effect arguments. *he phenomenon+ you lose more lysozyme from heating through microwave oven than you do through normal heating. *he e planation+ microwaves ,and not heat- destroy lysozyme. *he assumption+ there are no other e6planations. -ur prediction+ any choice that will suggest some other e6planation. In particular if it suggests that it is heat ,and not microwaves-. (hoice * is a perfect match& it suggests that it is not the microwaves but poc!ets of e6tra0hot heat in microwaves.

.AA.)itrama is a federation made up of three autonomous regions. Oorva. 3itro, and <uadar, @nder the federal revenue0sharing plan, each region receives a share of federal revenues equal to the share of the total population of )itrama residing in that region as shown by a yearly population survey. Last year the percentage of federal revenues Oorva received for its share decreased somewhat even though the population survey on which the revenue0sharing was based showed that Oorva9s population had increased.

If the statements above are true, which one of the following must also have been shown by the population survey on which last year9s revenue0sharing in )irama was based$ ,#- :f the three regions Oorva had the smallest number of residents ,'- %he population of Oorva grew by a smaller percentage than it did in previous years ,(- %he populations of 3itro and <uadar each increased by a percentage that e6ceeded the percentage by which the population of Oorva increased. ,)- :f the three regions. Oorva9s numerical increase in population was the smallest ,*- Oorva9s population grew by a smaller percentage than did the population of at least one of the other two autonomous regions.

#nswer+ *

E planation+ /e !now that 8each region receives a share of federal revenues equal to the share of the total populationL of the )itrama federation. /e also !now that Oorvo9s population increased in number ,pop survey discussed in last sentence- and that, surprisingly, its share went down. %he only way to reconcile these facts is that, collectively, the sum population of the other two regions increased by a greater number than did the population of Oorvo. %his is our !ey deductionMprediction in this inference question. (hoice * matches this. If you were debating between choices # and *, recognize that choice # is e6treme ,8smallest8- while choice * is more tentative ,8...at least one of...8-. In an inference question, if you are struggling between two choices, and one is clearly more tentative ,less e6treme- than another, always go with the more tentative ,less e6treme- one.

.AB.:ne year ago a local government initiated an antismo!ing advertising campaign in local newspapers which it financed by imposing a ta6 on cigarettes of .G cents per pac! :ne year later the number of people in the locality who smo!e cigarettes had declined by 5 percent (learly what was said in the advertisements had an effect although a small one on the number of people in the locality who smo!e cigarettes.

/hich one of the following, if true, most helps to strengthen argument$ ,#- Residents of the locality have not increased their use of other tobacco products such as snuff and chewing tobacco since the campaign went into effect ,'- # substantial number of cigarette smo!ers in the locality who did not quit smo!ing during the campaign now smo!e less than they did before it began ,(- #dmissions to the local hospital for chronic respiratory ailments were down by 1A percent one year after the campaign began ,)- 3erchants in the locality responded to the local ta6 by reducing the price at which they sold cigarettes by .G cents per pac! ,*- "mo!ers in the locality had incomes that on average were .A percent lower than those of nonsmo!ers

#nswer+ ) E planation+ %his is another e6plain the phenomenon or cause of the effect type of argument. %he first and second sentences describe a phenomenon, while the third sentence advances an e6planation for that phenomenon. *he phenomenon+ "mo!ing declined. *he author(s e planation+ %he ads ,and not the ta6esS- caused the decrease in smo!ing. ,8(learly what was said in the advertisements had an effect8.../henever the author ma!es an e6plain0the0phenomenon argument, he is assuming that there are no other e6planations.

%he way we strengthen an argument is by finding an answer choice that bac!s up the assumption. >ow do we bac! up the assumption that there are no other e6planations$ %he most common way it will happen on the <3#% is to loo! for an answer choice that negates an alternative e6planation.

# clear competing alternative e6planation from the stimulus is the ta6 itself. ,%hat is, instead of the ads9 content causing the decline in smo!ing, it could have been the ta60fuelled increase in the costs of the cigarettes that caused the decline in smo!ing-. "o, we would be loo!ing for any choice that says 8the ta6 was not the reason the smo!ing went down8.

(hoice ) is a perfect match to our prediction. It tells us that the merchants absorbed the cigarette ta6& therefore, the consumers who buy the cigarettes did not suffer, did not incur, the ta6 increase. %he fact that they did not have to pay more for the cigarettes and that they still decreased their smo!ing strengthens the author9s argument that what was said in the ads caused the smo!ing decline.

(hoice ' does not strengthen the argument that smo!ing decreased because of the ads9 content. Instead, it simply affirms the fact that smo!ing did decrease. In other words, choice ' does not give us any new information& we already !new from the passage that smo!ing declined. =or, that was the very phenomenon about which the author was advancing an e6planation.

#lthough this loo!s li!e an L"#% question, it is a very fair and even a good representation of how strengtheningMwea!ening commonly wor!s on the <3#%. 3any of the strengthenMwea!en questions on the <3#% are of the 8e6plain the phenomenon8 variety. %he quic!er you are able to spot the form, and the better you !now how to analyze the form, the more efficiently and confidently and easily you can answer a question li!e this. %herefore, according to me, a great idea would be to go through the (R section of the :<, e6plicitly pulling out those stnMw!n questions that use this form.

.AC.=ollowing years of declining advertising sales, the <reenville %imes reorganized its advertising sales force . years ago. 'efore the reorganization, the sales force was organized geographically, with some sales representatives concentrating on city0center businesses and others concentrating on different outlying regions. %he reorganization attempted to increase the sales representatives9 !nowledge of clients9 businesses by having each sales rep. deal with only one type of industry or retailing. #fter the reorg., the advertising sales increased.

In assessing whether the improvement in advertising sales can properly be attributed to the reorganization, it would be helpful to find out each of the following *E(*1%. ,#- . years ago, what portion of <reenville %imes9s total revenue was generated by advertising sales$ ,'- >as the circulation of <reenville %imes increased substantially in the past two years$

,(- >as there been a substantial turnover in the personnel in the advertising sales force over the last . years$ ,)- 'efore the sales reorganization, had the sales representatives found it difficult to !eep up with relevant developments in all types of businesses to which they were assigned$ ,*- >as the economy in <reenville and the surrounding regions been growing rapidly over the last . years$

#nswer+ # E planation+ In evaluate the argument ,or relevant information- questions, the right answer is the one that is directly inside the scope of the argument. "o, it will be something that will have a great effect on the argument. In fact, one way of handling these questions is to treat them as hybrid strengthenMwea!en+ the right answer will be something where if it goes one way it will strengthen the argument, and if it goes the other way it will wea!en the argument.

Let9s loo! at choice *+ ,*- >as the economy in <reenville and the surrounding regions been growing rapidly over the last . years$ /hat if the economy in <reenville and the surrounding regions >#" been growing rapidly over the last . years$ %hen, the argument is clearly wea!ened, as this can be an alternative e6planation ,something other than the reorganization- for why the advertising sales increased+ It suggests the increase in the advertising sales is part of some broader phenomenon rather than being causally related to the company9s reorganization. #nd, what if the economy in <reenville and the surrounding regions >#" N:% been growing rapidly over the last . years$ %hen, the argument is clearly strengthened, as the offered evidence improves in value+ now, the advertising sales have increased IN"1I%* of the fact that sales are dropping everywhere else ,or at least have not been increasing anywhere else-.

"o, by using the Oaplan strategy of treating this as a hybrid strengthenMwea!en, we can see that choice * is clearly relevant to the argument& in evaluating this argument it would be 8helpful to find out8 the answer to the question in choice *. 'ut because this is an *E(*1% question, we eliminate it.

(hoice # is irrelevant because it brings up 8portion8 or the idea of fraction. %he denominator is the company9s total revenues. 'ut we don9t have info about how the company9s total revenue has done over the last . years. Let9s apply the same strategy as above. /hat if the fraction of the total revenue coming from advertising sales has increased$ %his doesn9t mean that advertising sales have actually increased& instead, it could be that total revenue has decreased.

#nd what if the fraction of total revenue coming from advertising sales has decreased over the last . years. #gain, this doesn9t necessarily mean that advertising sales have decreased& instead, it could be that total revenues have decreased.

"o, answering the question in choice # does not help us in better 2udging the significance of the increase in advertising sales. %herefore, there9s no way finding out the answer to the question in choice # would help us in assessing whether the improvement in the advertising sales was or was not due to the reorganization& the argument is unaffected, neither strengthened nor wea!ened under either scenario.

%herefore, choice # is irrelevant to the argument. 'ut because this is an *E(*1% question, we select it. #nd because a test0ta!er well0versed in Oaplan procedures would have characterized the choices and then applied this strategy before going to the answer choices, she could have selected choice # !nowing that it was right not having to worry about evaluating the remaining answer choices, thereby saving her some time and thereby improving her scoreS

.AD.%hree ma2or laundry detergent manufacturers have concentrated their powdered detergents by reducing the proportion of inactive ingredients in the detergent formulas. %he concentrated detergents will be sold in smaller pac!ages. In e6plaining the change, the manufacturers cited the desire to reduce cardboard pac!aging and other production costs. 3ar!et analysts predict that the decision of three manufacturers, who control DG percent of the laundry detergent mar!et will eventually bring about the virtual disappearance of old0style bul!y detergents.

/hich one of the following, if true, most strongly supports the prediction made by the mar!et analysts$ ,#- 3ost smaller manufacturers of laundry detergents will consider it too e6pensive to retool factories for the production of the smaller detergent pac!ages. ,'- 3any consumers will be s!eptical initially that the recommended small amount of concentrated detergent will clean laundry as effectively as the larger amount of the old0style detergent did ,(- "ome analysts believe that consumers will have to pay a greater cost per load of laundry to use the new concentrated detergent than they did to use the old0style detergent ,)- 3a2or supermar!ets have announced that they will not charge the detergent manufacturers less to display their detergents even though the detergents will ta!e up less shelf space ,*- (onsumers are increasingly being persuaded by environmental concerns to buy concentrated detergents when available in order to reduce cardboard waste

#nswer+ * E planation+ # obviously wea!ens the argument9s conclusion+ #+ "maller manufacturers of laundry detergents will consider it too e6pensive to retool factories for the production of the smaller detergent pac!ages. 0000J 3anufacturers other than the big three are not going to switch to concentrated detergents.

.AF.3any cities face the problem of sprawl00unchec!ed development in areas outside of a city center where adequate transportation infrastructure may not yet e6ist. %he city of 3asonville is e6periencing rapid

population growth, which can lead to sprawl. %he city council recently passed a transportation concurrency measure, which requires that roads and other infrastructure be in place before developers can build in an area.

/hich of the following, if true, most seriously undermines the usefulness of the proposal$ ,#- In neighboring )orsetville, a similar measure caused transportation infrastructure costs to nearly double in the first year after its passage. ,'- /here rapid population growth e6ists, sprawl is rarely avoided. ,(- "ome developers include privately funded transportation infrastructure in their plans for new surburban shopping malls and neighborhoods. ,)- %he concurrency measure ensures that any development li!ely to attract more than 1GG new residents would be served by at least one of 3asonville9s bus routes. ,*- %he ma2ority of new areas in which developers build do not attract large populations that require significant transportation infrastructure.

#nswer+ * E planation+ %his is a wea!en question. %he proposal sets out to limit the negative effects of sprawl by requiring that infrastructure is in place before developers can build in an area. (onsider each choice, loo!ing for a reason why the proposal may not have positive results+ ,#- #n increase in cost may not be a good thing, but if the population rapidly increased, it may be well be desirable to spend that much money on transportation. It9s unclear whether this evidence would undermine the proposal. ,'- %his choice is far too general to be correct. /hether or not sprawl is rarely avoided does not indicate whether the proposal will have beneficial effects. ,(- %his choice would seem to reduce the demands of the proposal& however, the word 8some8 suggests that the proposal would still be necessary if such infrastructure were desirable. ,)- %his choice suggests a potential benefit of the proposal. %here9s no drawbac! here. ,*- %his is correct. If the proposal were enacted, there would be an unnecessary financial burden on either the city or on developers, ma!ing it more difficult to develop areas li!e this00areas that do not e6hibit the characteristics of sprawl.

.BG.'ecause ethylene dibromide, a chemical used to fumigate grain, was blamed for the high rate of nerve damage suffered by people who wor! in grain0processing plants, many such plants switched to other chemical fumigants . years ago. "ince then, however, the percentage of wor!ers at these plants who were newly diagnosed with nerve damage has not dropped significantly. %herefore, either ethylene dibromide was wrongly blamed or else the new chemicals also cause nerve damage.

/hich of the following is an assumption on which argument depends$ #. If the new chemicals cause nerve damage, the nerve damage caused would be different from any nerve damage that ethylene dibromide may cause. '. %here are no chemical fumigants that are completely safe for wor!ers in grain0processing plants. (. If ethylene dibromide causes nerve damage, it does not ta!e . years or longer for that damage to become detectable. ). /or!ers at grain0processing plants typically continue to wor! there even after being diagnosed with nerve damage. *. /or!ers at grain0processing plants that still use ethylene dibromide continue to have a high rate of nere damage.

#nswer+ ( E planation+ %he argument can be reduced to this paraphrase+ 8'ecause there has not been a drop in nerve damage in the last two years using the other fumigants, there was nothing wrong with ethyl dibromide.8 ;ou should always try to reduce arguments down to+ 8because 6, y8 in your head so that it ma!es it easier to thin! about them.

Let9s loo! at choice (, and then apply the denial test+ (. If ethylene dibromide causes nerve damage, it does not ta!e . years or longer for that damage to become detectable. "o, does the argument depend on this assumption$ /hat would happen to the argument if we removed ,ie, 8negated8 or 8denied8- this assumption$

(hoice ( denied+ If ethylene dibromide causes nerve damage, it ta!es longer than . years for the damage to become detectable. If that9s true00if it ):*" ta!e longer than two years for ethylene dibromide9s damage to be detectable00then it indicates that his reason00that it9s been two years using the other fumigants and there hasn9t been a drop in nerve damage00no longer establishes that there was nothing wrong with ethylene dibromide+ after all, it9s been only two years and the fact that the incidence of nerve damage hasn9t declined can easily be attributed to the fact that when ethylene dibromide causes nerve damage it ta!es more than two years for the damage to be detectedS

"o, if we remove ,i.e., deny or negate- choice (, then the author9s argument is struc! at its heart. %herefore, the argument9s logical e6istence depends on choice (.

.B1.In Oantovia, physicians income comes from insurance companies, which require physicians to document their decisions in treating patients and to 2ustify deviations from the companies treatment guidelines. %en years ago physicians were allowed more discretion. 3ost physicians believe that the companies requirements now prevent them from spending enough time with patients. ;et the average amount of time a patient spends with a physician during an office visit has actually increased somewhat over the last ten years.

/hich of the following, if true, most helps to resolve the apparent discrepancy between physicians perceptions and the change in the actual time spent$ #. 1atients are more li!ely to be in a hurry nowadays and are less willing to wait a long time to see their physician. '. 1hysicians today typically have a wider range of options in diagnosis and treatment to consider with the patient before prescribing. (. 1hysicians are increasingly li!ely to wor! in group practices, sharing the responsibility of night and wee!end wor!. ). 3ost patients would rather trust their physicians than their insurance companies to ma!e decisions about their treatment. *. "ince the insurance companies pay physicians a set amount for each office visit, it is to physicians financial advantage to see as many 1atients as possible.

#nswer+ ' E planation+ In parado6 questions, loo! for a contrast !eyword li!e 8yet8, 8but8 or 8however8& it will center the parado6. 'efore going to the answer choices, ma!e sure you understand the parado6 ,the surprise- and why it s parado6ical ,why it s surprising-. %he last sentence begins with 8yet8. %hat means that we can understand the parado6 by contrasting the final two sentences against each other. %he parado6 can be summed up as+ 8most physicians believe they have less time per patient yet they are actually spending more time per patient8.

%hen, choice ' clearly resolves+ because they have a so many more treatment options in diagnosis and treatment, naturally, physicians will need more time diagnose and treat a particular patient& so even though they are spending more time they believe ,i.e., feel- as though they are spending less time.

(hoice * is a common trap in parado6 questions& it is called a 1DG or opposite. If physicians want to minimize the time spent per patient ,to ma6imize their dollars-, they will spend less and not more time& but the last sentence tells us that they are actually spending more time per patient. "o, choice * tends to oppose the facts presented in the passage. #lso, if choice * were true, it would not lie well in the physicians9 mouths to say that because of the new requirements they feel as though they have less time& instead, it would be because of their desire to ma6imize dollars.

.B../hich of the following most logically completes the passage$ #ppendicitis ,inflammation of the appendi6- is potentially fatal& consequently, patients with symptoms strongly suggesting appendicitis almost have their appendi6 removed. %he appropriate surgery is low0ris! but performed unnecessarily in about .G percent of all cases. # newly developed internal scan for appendicitis is highly accurate, producing two misdiagnoses for every FD correct diagnoses. (learly, using this test, doctors can largely avoid unnecessary removals of the appendi6 without, however, performing any fewer necessary ones than before, since 44444. #. the patients who are correctly diagnosed with this test as not having appendicitis invariably have medical conditions that are much less serious than appendicitis '. the misdiagnoses produced by this test are always instances of attributing appendicitis to someone who does not, in fact, have it (. all of the patients who are diagnosed with this test as having appendicitis do, in fact, have appendicitis ). every patient who is diagnosed with this test as having appendicitis has more than one of the symptoms generally associated with appendicitis *. the only patients who are misdiagnosed using this test are patients who lac! one or more of the symptoms that are generally associated with appendicitis

#nswer+ ' E planation+ (omplete the passage questions are li!e inference questions. ;ou will most li!ely have to put in the correct conclusion or else ,as here- a correct piece of evidence that would logically support the conclusion. #s with other inference questions, the best approach is to follow the gist of the passage, ma!e a deduction and scan for a match. >ere, the author is arguing that the scan will obviate the need for surgeons to perform unnecessary appendectomies. Really$ Let9s e6amine what we have learned about the scan+ the misdiagnosis rate is .T. 'ut are the misdiagnoses false positives or false negatives$ # false positive is where the scan says you have appendicitis when you don9t while a false negative is when the scan fails to pic! up that a patient has appendicitis when in fact they do. =alse positives are not ris!y here bMc if a person who doesn9t have appendicitis gets hisMher appendi6 removed, its no big deal. :n the other hand, false negatives are very ris!y bMc if a person who has appendicitis doesn9t get hisMher appendi6 removed, then they may die. "o, in order for the author9s conclusion to be correct, the !ind of misdiagnoses the scan ma!es must be false positives and not false negatives. %hat would be the piece of evidence that would support the conclusion ,we !now we want to put in some evidence here bMc the passage ends with 8since9-. %hat is our deduction, and now we scan for a match....choice '.

.B5./hich of the following most logically completes the argument$ # certain cultivated herb is one of a group of closely related plants that thrive in soil with high concentrations of metals that are to6ic to most other plants. #gronomists studying the herb have discovered that it produces large amounts of histidine, an amino acid that, in test0tube solutions, renders these metals chemically inert. 1ossibly, therefore, the herb s high histidine production is what allows it to grow in metal0rich soils, a hypothesis that would gain support if 444444.

#. histidine is found in all parts of the plant s roots, stem, leaves, and flowers '. the herb s high level of histidine production is found to be associated with an unusually low level of production of other amino acids (. others of the closely related group of plants are also found to produce histidine in large quantities ). cultivation of the herb in soil with high concentrations of the metals will, over an e6tended period, ma!e

the soil suitable for plants to which the metals are to6ic *. the concentration of histidine in the growing herb declines as the plant approaches maturity

#nswer+ ( E planation+ %his is a classic causeMeffect scenerio, repeated often on the <3#%. In this case, the possible cause is >istidine, leading to the effect of survival of the plant. 'ut there is another !ey clue in this stimulus00 that this herb is one of a family of herbs. %hat is a shift in scope, which indicates an assumption in the argument. %he correct answer, we can predict, must therefore establish whether histidine is a possible e6planation for not only the one herb but also its cousins, or whether something else about the family of plants aside from histidine is a li!ely cause. =or that reason, ( will help us. If all of the family produces histidine, and all live in metal0rich soil, our e6planation is highly plausible. >owever, if some of the species are living in the same to6ic soil without histidine, the claim that this one plant requires the amino acid becomes suspect.

.B7.%he pace of new technology brings a constant stream of new devices to the mar!et and many of them en2oy commercial success. 'ut analysts warn that announcing new technology too soon after the introduction of a successful device can bac!fire+ consumers may resent feeling pressured to spend money to replace a device they have 2ust purchased, even if the new technology is clearly superior. %he result is that consumers either do not buy the old device in anticipation of the new one, or they do not buy the new device out of resentment over having already spent their money on the old one. "o if a company wishes to introduce a new device, it should wait until purchases of the old device have begun to decline. /hich of the following, if true, would best support the claims made above$ #. New technology often becomes less e6pensive after an initial surge in sales. '. 3edia outlets such as television and magazines often report on the planned introduction of new devices while sales of old devices are still strong. (. (onsumers are usually able to determine whether new technology is superior to current technology. ). "urveys have shown that consumers prefer to ma!e only one or two technology purchases per year. *. (onsumers tend to be loyal to technology companies whose products they en2oy using.

#nswer+ ' E planation+ If it s true that media outlets often report on the planned introduction of the new device while sales for the old device are strong, then the idea that consumers will halt their purchase of the current device in anticipation of the new device is strengthened since the consumers are now aware of the upcoming device. #nd the argument that a company should wait until purchases of the old device begin to decline is thereby strengthened since the company9s revenues will be hurt if consumers halt their purchase of the device that is currently in the mar!et. (hoice # brings up a fact that is nice for the consumers. 'ut it doesn9t really effect the company9s strategic planning as to when they should introduce the new device. =or choice )+ first of all, we don9t !now specific timelines here, so we can9t attach any significance to 8per year8. If the technology in question is something that is bought in 50A year cycles, then choice ) is pretty much irrelevant. #lso it says 8one or two8 instead of 8one8.

.BA.%o meet growing energy needs in Ibernia, companies built more energy plants that burn oil, coal, and natural gas. %o limit carbon dio6ide emissions and encourage the development of 8green8 energy, the Ibernian governmental recently imposed a carbon ta6 that directly affects the newly built plants. /e should therefore e6pect to see more money and innovation in the development of alternative energy and a decrease in the country9s reliance on fossil fuels.

/hich of the following is an assumption on which the argument depends$ ,#- %he plants that burn oil, coal, and natural gas can easily be converted so that they use 8green8 energy. ,'- %he carbon ta6 will lead to the closure of many of the plants that burn oil, coal, and natural gas. ,(- Ibernia9s energy needs will continue to grow even if the carbon ta6 results in higher energy costs. ,)- Ibernia9s reliance on fossil fuels is li!ely to cause the country to fall behind its neighbors in efforts to develop 8green8 energy. ,*- %he cost of developing alternative energy is less than the cost to e6isting energy plants of the government9s carbon ta6.

#nswer+ * E planation+ %his is an assumption question. %he argument concludes that, as a result of ta6es on newly built plants, companies will put more money into developing alternative energy sources. %he underlying assumption is that the ta6 is substantial enough so that developing new technologies is a cheaper alternative to stic!ing with the newly built plants and paying the ta6es. If you identify the assumption, you don9t need to analyze each choice, as choice ,*- neatly wraps up the lin! between the two sentences in the passage. ,*- is the correct choice.

.BB.%he stated goal of the government s funding program for the arts is to encourage the creation of wor!s of artistic e6cellence. "enator 'eton claims, however, that a government0funded artwor! can never reflect the independent artistic conscience of the artist because artists, li!e anyone else who accepts financial support, will inevitably try to please those who control the distribution of that support. "enator 'eton concludes that government funding of the arts not only is a burden on ta6payers but also cannot lead to the creation of wor!s of true artistic e6cellence. /hich one of the following is an assumption on which "enator 'eton s argument is based$ ,#- 3ost ta6payers have little or no interest in the creation of wor!s of true artistic e6cellence. ,'- <overnment funding of the arts is more generous than other financial support most artists receive. ,(- )istribution of government funds for the arts is based on a broad agreement as to what constitutes artistic e6cellence. ,)- :nce an artist has produced wor!s of true artistic e6cellence, he or she will never accept government funding. ,*- # contemporary wor! of art that does not reflect the independent artistic conscience of the artist cannot be a wor! of true artistic e6cellence.

#nswer+ *

E planation+ %he argument can be reduced to this paraphrase+ 8because government0funded artwor! can9t reflect independent artistic conscience, government funding cannot lead to creation of wor!s with artistic e6cellence8 or 8because no ind. art consc., no artistic e6cellence.8 %his is actually a common form of argument+ 8because no #, no '8. In this !ind of argument, the arguer is assuming that # is necessary for '. >ere, the 8#8 is independent artistic conscience and the ' is ,trueartistic e6cellence. "o, the author is assuming that independent artistic conscience is necessary for ,trueartistic e6cellence. If we went into the answer choices with this prediction and then scanned for a match, choice * clearly matches.

.BC.%he local agricultural official gave the fruit growers of the )istrict 1G =armers9 (ooperative a new pesticide that they applied for a period of three years in their pear orchards in place of the pesticide they had formerly applied. )uring those three years, the 1roportion of pears lost to insects was significantly iess0than it had been during the previous three0year period. :n the basis of these results, the official concluded that the new pesticide was more effective than the old pesticide, at least in the short term, in limiting the loss of certain fruit to insects.

%he official9s conclusion is most strongly supported if which one of the following groups of trees did not show a reduction in losses of fruit to insects$ ,#- 1each trees grown in the district that were treated with the new pesticide instead of the old 1esticide ,'- 1each trees grown in the district that were treated with the new pesticide in addition to the old 1esticide ,(- 1ear trees grown in the district those were treated with the old pesticide instead of the new pesticide ,)- 1ear trees grown in a neighboring district that were treated with neither the old nor the new pesticide ,*- 1ear trees grown in the district that were treated with the new pesticide instead of the old 1esticide

#nswer+ ( E planation+ =irst of all, the scope of the argument is pear trees and not peach trees, so we can eliminate choices # and ' immediately ,i.e., without having to read them-. /e are loo!ing to bolster the argument that it was the new pesticide that was the cause of the trees being less vulnerable to insect damage. %he stem is specific here+ we need to figure out how to strengthen the argument by thin!ing about a group of trees that are still vulnerable to insect damage. /ell, if we want to strengthen the idea that the new pesticides are the thing ma!ing pear trees less vulnerable to insect damage, then a choice that shows that without the pesticides the trees are still vulnerable to insects would do the tric!00that9s choice (.

.BD.%wenty percent of the stores in 3organville9s downtown shopping district will fail within five years because they will be competing directly with the "avemart discount department store newly opened in *ast 3organville. %he downtown shopping district has lost business at this rate before and has always completey rebounded. (onfidence that it will rebound again from the losses it is now about to suffer is ill founded, however, because 444444444444444444444 #- the stores li!ely to be put out of business by direct competition from "avemart are the downton shopping district9s anchor stores, on whose ability to draw shoppers many of the other downtown stores depend

'- the bus line that has long connected the downtown area of 3organville with *ast 3organville has a tradition of carrying shoppers who reside in *ast 3organville into downtown 3organville to shop (- when the downtown shopping district has rebounded before, the business premises of a failed business were typically ta!en over by a business of the same !ind as had been there before )- "avemart9s business plan for the *ast 3organville store is based on earning low profits, if any, during the first A years of store9s e6istence *- it is conceivable that the downtown shopping district could shrin! substantially without collapsing altogether

#nswer+ # E planation+ /e need to find an answer choice that wor!s as a reason supporting the argument that this time the downtown district will not be able to rebound. If choice ) is true00if the new competitor9s business plan ,i.e., "avemart9s business plan- is based on ma!ing low profits in the first five years00then it suggests that the downtown district /ILL be able to rebound. "o, choice ) does the opposite of what we want. :n the other hand, if choice # is true, then there is a unique reason for believing that this time the )owntown district won9t be able to rebound.

.BF.%hough suc!ing zinc lozenges has been promoted as a treatment for the common cold, research has revealed no consistent effect. Recently, however, a zinc gel applied nasally has been shown to greatly reduce the duration of colds. "ince the gel contains zinc in the same form and concentration as the lozenges, the greater effectiveness of the gel must be due to the fact that cold viruses tend to concentrate in the nose, not the mouth.

/hich of the following, if true, most seriously wea!ens the argument$ # *6perimental sub2ects who used the zinc gel not only had colds of shorter duration but also had less severe symptoms than did those who used a gel that did not contain zinc. ' %he mechanism by which zinc affects the viruses that cause the common cold has not been conclusively established. ( %o ma!e them palatable, zinc lozenges generally contain other ingredients, such as citric acid, that can interfere with the chemical activity of zinc. ) No zinc0based cold remedy can have any effect unless it is ta!en or applied within 7D hours of the initial onset of cold symptoms. * )rug0company researchers e6perimenting with a nasal spray based on zinc have found that it has much the same effect on colds as the gel does.

#nswer+ ( E planation+

0hat is the phenomenonC %hat in one mode of administration ,lozenges-, zinc produces no consistent beneficial effect while in another mode of administration ,gel-, it does. 0hat is the e planationC %hat something about the cold virus ,its congregation in the nose- ma!es gel more effective than lozenges. 0hat is the author assumin!C %hat there are no other e6planations for this phenomenon. ,in all 8e6plain the phenomenon8 arguments, this is the central assumption-. How do we weakenC %he way we wea!en is by finding an answer choice that attac!s the assumption. >ow do we attac! the assumption that 8there are no other e6planations8$ "imple+ by finding an answer choice that suggests there is an alternative e6planation. >ere, we would be loo!ing for something that suggests that something else ,something besides the cold virus9 congregation in the nose- e6plains why zinc gels are more effective than zinc lozenges. 'ecause choice ( tells us that zinc lozenges generally contain other ingredients, it could be that the difference in consistent beneficial effect between the modes of administration ,lozenge vs gel- is due to the fact that the gel contains these other ingredients. #nd, it doesn9t matter that these other ingredients interfere with zinc+ the gels might be more effective because of the other ingredients such as citric acid& that is, the zinc gels9 greater effectiveness might not have anything to do with zinc at all. In that case, choice ( clearly wea!ens the argument.

.CG.1sychologists who wish to have one of their boo! review nominated for the prestigious 'oatwright 1sychology Review award should not submit boo! review articles that review more than three boo!s at a time. %his is because editors for the 'oatwright 1sychology Review will not publish a boo! review article if it is too lengthy and cumbersome to read. In their submission guidelines, the editors e6plicitly state that review articles that cover more than three boo!s at a time are considered too lengthy and cumbersome to read.

/hich of the following statements represents an assumption upon which the argument relies$ #- %he boo! reviews articles that covers the most boo!s must be the lengthiest and most cumbersome article to read. '- If a boo! review article is published in the 'oatwright 1sychology Review, that article will receive the prestigious 'oatwright 1sychology Review award. (- #ll articles published in the 'oatwright 1sychology Review must be limited to a certain length specified by the editors. )- %he 'oatwright 1sychology Review editors generally prefer boo! review articles that cover one boo! rather than boo!s. *- %o be nominated for the 'oatwright 1sychology Review award, a psychologist9s boo! review article must be published in the 'oatwright 1sychology Review.

#nswer+ * E planation+ $onclusion+ 1sychologists who wish to have one of their boo! review nominated for the prestigious 'oatwright 1sychology Review award should not submit boo! review articles that review more than three boo!s at a time.

%remise 7+ editors for the 'oatwright 1sychology Review will not publish a boo! review article if it is too lengthy and cumbersome to read. %remise 1+ %he editors e6plicitly state that review articles that cover more than three boo!s at a time are considered too lengthy and cumbersome to read.

'oo! to be nominated for review000"hould not e6ceed more than 5 boo!s at time. %o publish a boo! review00000should not be too lengthy %oo lengthy0000are the ones that cover more than 5 boo!s. (onnecting all above three, we see that In order to be nominated for the review award, article must be published. * (orrectly states this assumption.

.C1.%he population of desert tortoises in %argland s Red )esert has declined, partly because they are captured for sale as pets and partly because people riding all0terrain vehicles have damaged their habitat. %argland plans to halt this population decline by bloc!ing the current access routes into the desert and announcing new regulations to allow access only on foot. %argland s officials predict that these measures will be adequate, since it is difficult to collect the tortoises without a vehicle.

/hich of the following would it be most important to establish in order to evaluate the officials prediction$ #. /hether possessing the tortoises as pets remains legally permissible in %argland '. /hether %argland is able to enforce the regulations with respect to all0terrain vehicle entry at points other than the current access routes (. /hether the Red )esert tortoises are most active during the day or at night ). /hether people who travel on foot in the Red )esert often encounter the tortoises *. /hether the %argland authorities held public hearings before restricting entry by vehicle into the Red )esert

#nswer+ ' E planation+ Relevant information questions are really hybrid strengthenMwea!en questions. %he correct answer will be a question with . possible answers+ one answer will strengthen the argument, the other will wea!en it. %he wrong choices will all be outside the scope, i.e. the answers to their questions are all irrelevant. >ere, ,'- is the only relevant question. If %argland can enforce the regs for all0terrain vehicles, then it will be harder to capture the tortoises and the plan should wor!. If %argland can9t enforce the regs for all0terrain vehicles, then people can still #%H into the desert to pic! up the poor critters and the plan will li!ely fail. #ccordingly, whether %argland can enforce the regs for #%Hs is relevant and ,'- is correct. *estLuvs E planation:

:ne potential point of confusion is that choice ' discusses 8entry points other than the current access routes8 while the argument discusses bloc!ing current access routes. %his does not at all ma!e choice ' outside the scope. Instead, this is actually very relevant to determining whether the plan will wor!+ %he prediction that the plan will wor! simply assumes that people won9t be able to circumvent the currently bloc!ed off routes by entering through entry points other than the current access routes. If people are able to come in through entry points other than the current access routes, then the plan won9t li!ely wor!, and the prediction is wea!ened. #nd, if people aren't able to come in through entry points other than the current access routes, the plan is more li!ely to wor!, and the prediction that it will wor! is strengthened. %hus, choice ' is relevant.

.C..# ma2or chemical spill occurred five years ago at 'a!er s 'each, the world s sole nesting ground for 3erric! sea turtles, and prevented nearly all the eggs laid that year from hatching. ;et the number of adult female 3erric!s returning to lay their eggs at 'a!er s 'each has actually increased somewhat since five years ago. (learly, environmentalists prediction that the world s 3erric! population would decline as a result of the spill has proven unfounded.

/hich of the following, if true, most seriously undermines the argument offered in refutation of the environmentalists prediction$ #. %he chemical spill five years ago occurred at a time when there were neither 3erric! sea turtles nor 3erric! sea turtle eggs on 'a!er s 'each. '. =emale 3erric! sea turtles begin returning to 'a!er s 'each to lay their eggs when they are ten years old. (. @nder normal conditions, only a small proportion of hatchling female 3erric! sea turtles survive in the ocean until adulthood and return to lay their eggs at 'a!er s 'each. ). *nvironmental pressures unrelated to the chemical spill have caused a significant decline in the population of one of the several species of sea birds that prey on 3erric! sea turtle eggs. *. #fter the chemical spill, an environmental group re2ected a proposal to increase the 3erric! sea turtle population by transferring eggs from 'a!er s 'each to nearby beaches that had not been affected by the spill.

#nswer+ ' E planation+ %he refutation of the prediction that turtle population wasn9t negatively impacted is based off the current number of turtles returning to lay eggs. If ' is true, then the current number of returning is irrelevant& we9re concerned about the number of turtles who will return 1G years after the spill, which is still A years off in the future. "ince ' ma!es us question the relevance of the refutation9s evidence, it wea!ens the refutation.

.C5.1arland s alligator population has been declining in recent years, primarily because of hunting. #lligators prey heavily on a species of freshwater fish that is highly valued as food by 1arlanders, who had hoped that the decline in the alligator population would lead to an increase in the numbers of these fish available for human consumption. ;et the population of this fish species has also declined, even though the annual number caught for human consumption has not increased. /hich of the following, if true, most helps to e6plain the decline in the population of the fish species$ #. %he decline in the alligator population has meant that fishers can wor! in some parts of la!es and rivers that were formerly too dangerous.

'. :ver the last few years, 1arland s commercial fishing enterprises have increased the number of fishing boats they use. (. 3any 1arlanders who hunt alligators do so because of the high mar!et price of alligator s!ins, not because of the threat alligators pose to the fish population. ). )uring 1arland s dry season, holes dug by alligators remain filled with water long enough to provide a safe place for the eggs of this fish species to hatch. *. In several neighboring countries through which 1arland s rivers also flow, alligators are at ris! of e6tinction as a result of e6tensive hunting.

#nswer+ ) E planation+ 'ecause it uses the word 8e6plain8, we can tell from the question stem ,which we should always read first- that this is a parado6 question ,the stimulus does not present an argument& instead the stimulus presents . or more facts out of whose combination arises a parado6-. In parado6 questions, we loo! for a contrast !eyword such as 8yet8, 8but8, 8however8 etc. "uch a word will center the parado6& oftentimes by putting the fact that comes before 8yet8 together with the fact that comes after 8yet8 we will grip the parado6. #nother term for 8parado68 is 2ust 8surprise8.

If we loo! at the stimulus, we see the first word of the last sentence is 8yet8. %herefore, we can comprehend the parado6 by contrasting the fact that comes before 8yet8 against the fact that comes after 8yet8. %he parado6 is+ the alligators eat the fish and yet when the 1arlanders !ill the alligators there are fewer ,not more- fishS %his is indeed surprising. Now, we go to the answer choices searching for one that will resolve the parado6, relieve our surprise.

(hoice ) resolves the parado6+ it tells us that, by !illing off the alligators, the 1arlanders are actually decreasing the li!elihood that the fish eggs will hatch. ,%he alligators dig holes that allow the fish eggs to thrive& ironically, the alligators help to sustain the fish population, even though they themselves eat the fish-.

.C7.%echnology that enables drivers to pay tolls automatically, using a small device mounted to their cars, has become increasingly popular. *conomists compared the toll rates in places using automated collection with those that collect tolls manually. %hey found that automated collection is associated with toll rate increases .G to 7G percent higher than in places using manual collection. %his difference may be due to the costs associated with the technology. >owever, another study showed that manually collected tolls increased the least during election years, while automatic tolls showed no such pattern.

/hich of the following hypotheses is best supported by the statements given$ ,#- )espite lower increases, manual toll collection is much more e6pensive than is automated collection. ,'- Rates where tolls are manually collected are less politically motivated than rates where tolls are automatically collected. ,(- 1oliticians see!ing reelection try to !eep manually collected tolls low in order to garner favor with their constituents, while they do not ma!e the same effort with automatically collected tolls.

,)- In the years following election years, manually collected tolls increase more than do automatically collected tolls. ,*- #s voters grow accustomed to automatically collected tolls, they will associate those rates with their elected officials, so politicians see!ing reelection will attempt to limit those rate increases, as well.

#nswer+ ( E planation+ %his is an inference question. %here are two !ey findings in the passage+ that automated toll rates are higher than manually collected toll rates, and that manually collected tolls do not increase as much during election years as other years. %he inference ,or conclusion- in the correct choice will li!ely have something to do with one of those. (onsider each one+ ,#- %his contradicts the claim in the passage, and there is no evidence to support it. ,'- If anything, this is opposite of what we9re loo!ing for. %he final sentence of the passage suggests that manually collected toll rates may be politically motivated, while automatic tolls are not. %his statement claims the opposite. ,(- %his is correct. It is the most li!ely e6planation of the election0year distinction, and the distinction in election years between manually collected tolls and automated tolls. ,)- %his may or may not be true. /e !now that manual toll rates increase more in non0election years than in election years, but not how those rates compare to automated tolls in the same years. ,*- %his statement may be true, but it goes far beyond the claims made in the passage. %hus it is not a reasonable inference to ma!e. .CA."unflowers growing in pots were placed, with their roots submerged, in the pond contaminated with radioactive elements. %he sunflowers !ept growing& in the process, they absorbed radioactive elements. /ithin twelve days, DA percent of the radioactive elements were removed from the water, which is no less than can be accomplished with the much more e6pensive conventional filtration techniques. "cientists therefore propose using sunflowers for decontamination wherever there are radioactively contaminated ponds. /hich of the following, if true, points to a limitation on the applicability of the proposed method of decontamination$ #. "ome plants other than sunflowers can also remove radioactive elements from water. '. %he water in many ponds contaminated with radioactive elements is so cold that it would !ill sunflowers whose roots were submerged in it. (. "unflowers that grow with their roots submerged in water grow less well than sunflowers growing under optimal conditions on dry land. ). :nly species of sunflowers with large leaves can have their roots submerged in water and still !eep growing. *. In ponds in which the circulation of the water is artificially increased, sunflowers absorb radioactive elements far faster than they do in other ponds.

#nswer+ ' E planation+ %he question as!s us for a fact that would point to a limitation in the applicability of the method. (hoice ) limits the method to using sunflowers with large leaves. 'ut choice ) neither suggests nor establishes that sunflowers with large leaves are lac!ing in supply. #lso, choice ) does not suggest that sunflowers in general are lac!ing in supply. %hus, choice ) does not necessarily point to a limitation in the applicability of the method. :n the other hand, choice ' necessarily points to a limitation in the applicability of the method. %hus, choice

' satisfies the conditions of the question while choice ), along with the rest of the choices, do not. %hus, there is only one answer that necessarily satisfies the conditions of the question.

.CB.Investigators concluded that human failure was not responsible for the fatal airplane crash last #ugust, and since that time new and more stringent rules for identifying and reporting mechanical problems have been in effect. %hat accounts for the fact that reports of airplane mechanical problems have increased in frequency by AG percent since last #ugust. /hich one of the following is an assumption underlying the argument in the passage$ ,#- #irplane travel is still relatively safe, despite the increase in reported mechanical problems. ,'- 3echanical problems in airplanes have increased dramatically since last #ugust. ,(- 3echanical problems in airplanes have not increased by AG percent since last #ugust. ,)- #irlines are less reluctant to report mechanical problems than they previously were. ,*- 3echanical problems in airplanes have become easier to detect since last #ugust.

#nswer+ ( E planation+ *he phenomenon+ reports of problems have increased. *he e planation+ more stringent rules for identifying and reporting mechanical problems. *he assumption+ no other e6planations. (hoice (+ if you deny it, the argument clearly falls apart. If we deny (hoice ( we have+ mechanical problems have increased. If mechanical problems have increased then it suggests that the increase in reporting of mechanical problems is due to the fact there #R* more mechanical problems ,rather than being due to the fact that reporting procedures are now more stringent.- %hus, the author9s argument depends on choice (. (hoice ) uses the word 8reluctance8. >ere9s a tip+ choices that tal! about people9s motivations, biases, preferences, etc are almost always wrong ,unless the argument is specifically about someone9s motivations, biases, preferences, etc.-

.CC.In #ustralia the population that is of driving age has grown larger over the last five years, but the annual number of traffic fatalities has declined9 %his leads to the conclusion that, overall, the driving0age population of #ustralia consists of more s!illful drivers now than five years ago.

*ach of the statements below, if true, wea!ens the argument *E(*1%+ ,#- %hree years ago, a mandatory seat0belt law went into effect throughout #ustralia ,'- =ive years ago, #ustralia began a ma2or road repair pro2ect. ,(- 'ecause of increases in the price of fuel, #ustralians on average drive less each year than in the preceding year. ,)- %he number of hospital emergency facilities in #ustralia has doubled in the last five years9 ,*- In response to an increase in traffic fatalities, #ustralia instituted a program of mandatory driver education five years ago.

#nswer+ * E planation+ %his is another e6plain the phenomenon or cause of the effect argument ,these !inds of arguments are very common in strengthenMwea!en questions-.

*he phenomenon+ more drivers but fewer fatalities. *he author(s e planation+ drivers are now more s!illful. *he assumption+ there are no other e6planations ,this is always the assumption in e6plain the phenomenon arguments+ How to weakenC 'y finding a choice that attac!s this assumption000 a choice will attac! this assumption by opening up the possibility of an alternative e6planation. %his is an *E(*1% question, so we need to eliminate four choices. (hoices #, ', and ( clearly point to alternative e6planations. (hoice ) is tric!ier but it is telling us that instead of drivers being more s!illful, the victim of any particular car accident is more li!ely to be rescued ,because there are more emergency facilities-. %hus, choice ) also points to an alternative e6planation. (hoice * doesn9t point to an alternative e6planation. Instead it supports the author9s e6planation. If there are more mandatory driver education programs than there used to be, then it bolsters his argument that drivers are now more s!illed.

.CD. #rchaeolo!ist+ # s!eleton of a North #merican mastodon that became e6tinct at the pea! of the Ice #ge was recently discovered. It contains a human0made pro2ectile dissimilar to any found in that part of *urasia closest to North #merica. %hus, since *urasians did not settle in North #merica until shortly before the pea! of the Ice #ge, the first *urasian settlers in North #merica probably came from a more distant part of *urasia.

/hich one of the following, if true, most seriously wea!ens the archaeologist s argument$ ,#- %he pro2ectile found in the mastodon does not resemble any that were used in *urasia before or during the Ice #ge. ,'- %he people who occupied the *urasian area closest to North #merica remained nomadic throughout the Ice #ge. ,(- %he s!eleton of a bear from the same place and time as the mastodon s!eleton contains a similar pro2ectile. ,)- :ther North #merican artifacts from the pea! of the Ice #ge are similar to ones from the same time found in more distant parts of *urasia. ,*- (limatic conditions in North #merica 2ust before the Ice #ge were more conducive to human habitation than were those in the part of *urasia closest to North #merica at that time.

#nswer+ # E planation+ 'ecause the assumption bridges the gap between the ideas in the evidence and the idea in the conclusion, the classic Oaplan method involves loo!ing for ideas that are differentially present in conclusion and evidence. Notice a big idea in the evidence is this whole Kpro2ectileL business+ 8it is a simple premise and conclusion structure. 1remise+ pro)ectile did not resemble to any pro)ectile found in the part of *urasia closest to #merica. 'ut notice this idea or term 8pro2ectile8 is N:% in the conclusion+

$onclusion+ =irst *urasian settlers in North #merica came from distant part of *urasia. %hus, the arguer is definitely assuming something about pro2ectiles+

#ssumption+ 1ro2ectile found resembled the pro2ectile in distant part of *urasia. /e wea!en arguments by finding a choice that attac!s the assumption. In order to attac! the assumption, an answer choice has to be within the scope of the argument ,since the assumption is always within the scope of the argument-. #ccordingly, the correct answer almost certainly has to have the term Kpro2ectileL in it. # quic! scan of the answer choices reveals that only choices # and ( have the term Kpro2ectileL in them. #nd choice ( is definitely outside the scope, so choose #...doneS

.CF.#ccording to a recent research study, more than FGT percent of graduates of private high schools in a certain county continue their education in college. 'y contrast, only BAT of graduates of public high schools subsequently pursue college education. %herefore, if parents in the county wish to increase the li!elihood that their children will attend college, they should send them to private rather than public schools.

/hich of the following statements would most seriously wea!en the argument above$ #- <raduates of private schools typically score higher on standardized tests and other tests of academic achievement. '- /hile private schools are typically very e6pensive, attendance of public school is free for the residents of the county. (- In comparison with graduates of private schools, a substantially greater proportion of public school graduates receive need0based financial aid for their college education. )- In comparison with private schools, public schools provide more opportunities for student involvement in sports and other athletic activities, which almost always increase the li!elihood of students9 acceptance to colleges. *- "ince most public schools are located in rural areas of the county populated primarily by farmers, nearly 5GT of students from public high schools choose to pursue farming occupations rather than apply to colleges.

#nswer+ * E planation+ /ell, even if choice ) is true, we still !now it9s also true that only BAT of students from public schools go on to college ,while FGT from private schools do.- (hoice ) does not negate this fact. Let9s say choice ) weren9t true ,denial test-+ if public schools didn9t provide these e6tra opportunities, then, the fraction of students from public schools going on to college would surely be less than BAT, maybe around 7G or AG percent. 'ut with these e6tra opportunities that choice ) discusses, this percentage goes up to BAT. 'ut BAT is still less than FGT. ,%his is what 3<3#%9s e6planation means when it says that the study has already factored this in."o, choice ) is a common !ind of wrong answer in wea!en questions. It9s an answer choice that tempts you to put less weight on evidence discussed in the passage. 'ut evidence in the passage is always true. "o in wea!en, avoid choices that you thin! are contradicting stated evidence. In order to wea!en an argument, we need to attac! its reasoning, not the facts on which the argument is built.

(hoice *, in contrast to choice ), attac!s the author9s reasoning+ %he author assumes that the reason fewer public school !ids go on to college is because of the inferior education that students receive at public schools. (hoice * effectively attac!s this assumption by establishing that many public school !ids don9t go on to college because of choice ,rather than an inferior education-.

.DG.'ecause of rising costs, @nited "hipping (ompany raised its rates by ten percent last year. 3any of its customers bal!ed at the increase, however, and turned to a rival company whose rates were lower. In response, @nited "hipping (ompany began offering a pac!age of services for a single rate that, though high, is still lower than the combined rates of the individual services. :fficers of the company claim that this move will recoup the company9s lost profits.

/hich of the following, if true, provides the best basis for the officers9 claim$ #- @nited "hipping will aggressively advertise the new pac!age of services. '- @nited "hipping9s rival does not offer some of the services included in the pac!age. (- 3ar!eting studies have shown that many of @nited "hipping9s former customers would prefer a combined rate for their various shipping services. )- @nited "hipping does not already offer all the services separately. *- No other shipping company offers the same pac!age of services as @nited "hipping.

#nswer+ ( E planation+ %he argument isn9t that by providing new services, @nited "hipping will draw customers away from its competitors. Instead, the argument is that by pac!aging up multiple services that customers of other companies are already paying for individually, @nited "hipping can offer up an on0the0whole cheaper value proposition. ,%his is also why, incidentally, choice ( is correct-.

.D1. Joolo!ist+ 3eer!ats are desert mammals that live in groups and are believed to e6hibit altruistic behavior. /hile the group is foraging, a single meer!at will stand guard, !eeping watch for predators and giving loud warning cries when danger appears. >owever, this behavior is not true altruism+ 3eer!ats on guard duty do not face a greater ris! of being !illed, and solitary meer!ats will also stand guard.

/hich of the following, if true, most seriously undermines the reasoning in the zoologist9s argument$ ,#- "ome meer!at populations are nomadic and rarely return to the same place to find food. ,'- %he degree to which an activity may lead to immediate death is not the only determinant of altruism. ,(- "everal desert species prey on meer!ats, and meer!ats are most at ris! when they are foraging for food, even if one of the group stands guard. ,)- # solitary meer!at who stands guard may be part of a group e6cept for when it forages for food.

,*- /hen meer!ats stand guard, they forego opportunities to forage for food even though others in the group may not share food with them.

#nswer+ * E planation+ %he argument in question is that meer!ats are not acting in an altruistic manner when they stand on guard duty, because guard duty does not put them at greater ris! of being !illed. %he underlying assumption is that the ris! of being !illed on guard duty is the only way in which this behavior could be considered altruistic. #n alternative reason to consider the behavior altruistic would provide a wea!ener. (onsider each choice+ ,#- %his is irrelevant, as it has nothing to do with the behavior of meer!ats on guard duty. ,'- In general terms, this addresses the assumption, but it doesn9t give us any concrete reason why the meer!at may be altruistic. ,(- %his choice suggests that foraging for food is ris!y for meer!ats, but as the passage claims, guard duty does not pose a greater ris!. ,)- %his is irrelevant. /e are concerned with the possible altruism of meer!ats. %his choice only tells us when meer!ats affiliate with others. ,*- %his is correct. It describes a ris! to meer!ats who stand guard, in that by standing guard, they may miss out on opportunities to eat for themselves. %hat9s a danger apart from the ris! of being !illed by a predator. .D..@nprecedented industrial growth in the country of Remo has created serious environmental problems because factories there lac! adequate pollution0control systems. Remo is developing a clean growth plan that includes environmental regulations that will require the installation of such systems. "ince no companies in Remo currently produce pollution0control systems, the plan, if implemented, will create significant opportunities for foreign e6porters to mar!et pollution0control systems.

/hich of the following, if true, most seriously wea!ens the argument$ #. %he clean growth plan will provide ta6 incentives for local businesses to develop and manufacture pollution0control devices. '. =oreign e6porters would provide factory0trained technicians to maintain the pollution0control systems sold to Remo. (. Industrial lobbyists sponsored by local businesses in Remo are trying to prevent the implementation of the government regulations. ). %he regulations that Remo plans to implement are much less strict than those in neighboring nations. *. 1ollution in Remo has caused serious health problems for wor!ers, contributing to a significant increase in the number of wor!days lost to illness.

#nswer+ #

E planation+ Remember to focus on the correct answer. It is a mista!e to thin! that you have to give every answer choice attention, and that you have to figure out why every wrong answer is wrong. /e get rewarded for announcing the correct answer00not for articulating why the wrong answers are wrong.

(hoice ( is wrong because the author9s argument only comes into effect I= the plan is implemented ,last sentence of the argument-. (hoice ) is an irrelevant comparison. /e don9t !now the situation in neighboring countries, so we can9t 2udge the relevance of fact ). #lso, again, the author9s argument only comes into play I= the plan is implemented.

Das könnte Ihnen auch gefallen